SlideShare a Scribd company logo
1 of 86
Download to read offline
ข้อสอบศูนย์ประเมิน
ความรู้ความสามารถ
ของผู้ประกอบวิชาชีพ
เวชกรรม (ศรว.)
ขั้นตอนที่ 2 ครั้งที่ 1
วันที่ 9 มีนาคม 2551
สวัสดีครับ เพื่อนๆ พี่ๆ น้องๆ
ข้อสอบ ศรว. ฉบับนี้เป็นฉบับ
update ล่าสุด เมื่อเดือน
กันยายน 2551 โดยเป็นข้อสอบ
จากพี่ๆ มหาวิทยาลัยต่างๆ 4
สถาบันคือ ขอนแก่น
ธรรมศาสตร์ เชียงใหม่ และ
จุฬานะครับ
ข้อสอบฉบับนี้ จะไม่มีโอกาส
สำาเร็จได้เลย ถ้าปราศจาก
แรงงานและแรงใจของพี่ๆ
คณะต่างๆ ที่รวบรวมข้อสอบ
เอาไว้ จึงขอเอ่ยนามขอบคุณไว้
ที่นี้ด้วยครับ
ขอขอบคุณ
- พี่กิ๊ก, ก้อย, ปุย, สา,
นูดเดิล, แบงค์ MD 31
KKU
- พี่ๆ มช. ที่ช่วยๆกันจำา
ข้อสอบ
- พี่ๆ Med 14 THAMMASAT
University
- พี่ๆ จุฬาโดยเฉพาะเฮีย
peck และเจ๊กุ๊ก
สุดท้ายนี้ ขอขอบคุณเพื่อนๆ
ทุกคนไม่ว่าจะเป็นจุฬา วราลี
กรวิกและเพื่อนๆคนอื่นที่สละ
แรงกายมารวบรวมข้อสอบใน
ครั้งนี้ด้วยครับ
ป.ล. การรวบรวมข้อสอบครั้งนี้
อาจไม่ใช่ข้อสอบชุดที่สมบูรณ์
ที่สุดและยังเป็น version แรก
เท่านั้น ถ้าคณะใดรวบรวม
ข้อสอบเสร็จแล้วก็รบกวนส่งมา
อีกนะครับ ยินดีจัดทำาให้เสมอ
ครับ ถึงแม้ข้อสอบชุดนี้จะ
คลอดออกมาช้าไปหน่อย แต่ก็
น่าจะทันอยู่นะ ^_^
Medicine
1. ผู้ป่ วยหญิงไทยอายุ 32 ปี
ปวดข้อเป็นๆ หายๆ และ
มีผื่นขึ้นที่หน้า ผลการ
ตรวจทางห้องปฏิบัติการ
พบ Urine protein 1+, red
blood cell cast ก า ร ส่ ง
ตรวจเพิ่มเติมในข้อใดต่อ
ไปนี้ specific สำาหรับการ
วินิจฉัยในผู้ป่ วยรายนี้มาก
ที่สุด
a. Rheumatoid factor
b. ANA
c. Anti-histone
antibody
d. Anti ds-DNA
antibody
2. ผู้ป่วยหญิงอายุ 35 ปี มีไข้
ตำ่าๆ ปวดข้อเข่าและข้อ
มือมา 4 วันก่อนมาโรง
พยาบาล มาพบแพทย์
เจาะนำ้าข้อเข่าตรวจพบ
wbc = 89000, PMN =
90%, G/S: gram negative
diplococci intracellular
การรักษาข้อใดต่อไปนี้
เหมาะสมที่สุด
a. Cloxacillin
b. Gantamicin
c. Ceftriazone
d. Doxycycline
e. Metronidazole
3. ผู้ป่ วยชายวัยกลางคน ชัก
แ บ บ Generalized Tonic
Clonic ป ฏิ เ ส ธ ป ร ะ วั ติ
อุบัติเหตุหรือการใช้สาร
เสพย์ติด ผู้ป่ วยไม่เคยมี
อาการชักแบบนี้มาก่อน
ไ ม่มีไ ข้ จากการต ร วจ
ร่างกายเบื้องต้นไม่พบ
neurological deficit ใ ด ๆ
แ ต่ พ บ ว่ า มี papilledema
ผ ล CT scan: multiple
calcified cystic lesion
เชื้อก่อโรคในข้อใดต่อไปนี้
น่าจะทำาให้เกิดอาการใน
ชายผู้นี้มากที่สุด
a. Taenia solium
b. Taenia saginata
c. Toxocara
d. Toxoplasma=HIVinf
ect มาด้วยชัก
e. Gnathostoma
4. ผู้ป่ วยชายอายุ 55 ปี ไอ
แห้งๆ บางครั้งมีเลือดปน
ออกมาด้วย นำ้าหนักตัวลด
ลง 4 กิโลกรัมใน 1 เดือน
ผลการตรวจ AFB ได้ผล
negative 3 ครั้งติดต่อกัน
ผ ล CXR พ บ ว่ า มี
reticulonodular infiltration
with thickening cavity
การปฏิบัติในข้อใดต่อไปนี้
น่าจะเหมาะสมกับผู้ป่ วย
รายนี้มากที่สุด
a. ร อ ผ ล culture for
TB
b. Start anti-TB drug
<diag tb
จ า ก film แ ล ะ
clinical>
c. Bronchoscopy
d. Aspiration
e. PCR for TB
5. ผู้ป่ วยชายอายุ 18 ปี ไป
เยี่ยมสถานรับเลี้ยงเด็กมา
เมื่อ 2 สัปดาห์ก่อนมาโรง
พยาบาล ต่อมามีอาการ
อ่อนแรงของแขนขา เป็ น
ม า ก ขึ้น เ รื่อ ย ๆ แ พ ท ย์
ทำาการตรวจร่างกายพบว่า
มี Upper motor power
4/5, lower 1/5, absent
DTR ข้อใดคือการวินิจฉัย
ในผู้ป่ วยรายนี้ที่เหมาะสม
ที่สุด
a. Transverse myelitis
b. GBS
c. MG
d. Neurosyphilis
6. ผู้ชายอายุ 30 ปี เหนื่อย
มา 1 เดือน แพทย์ทำาการ
ต ร ว จ ร่ า ง ก า ย พ บ ว่ า มี
neck vein engorgement,
Hepatomegaly ฟั ง ป อ ด
ปกติ ผลจากการตรวจ
EKG พบว่ามี diffuse ST-
T change, low voltage
ข้อใดคือการวินิจฉัยในผู้
ป่วยรายนี้ที่เหมาะสมที่สุด
a. Restrictive
pericarditis แ ต่
เ ป ลี่ ย น เ ป็ น
constrictive
b. Beri beri
c. Cardiac tamponade
d. Dilated
cardiomyopathy
7. เด็กชายอายุ 9 ขวบ มา
โรงพยาบาล ด้ว ยไ ข้สูง
หายใจหอบเหนื่อย ไอ
แ พ ท ย์ ทำา ก า ร ต ร ว จ
ร่างกายพบว่ามี BT= 39.0
o
C, PR= 120 /min, RR =
40 /min, crepitation both
lungs, decreased breath
sound left lung ผู้ ป่ ว ย มี
แผลตุ่มหนองและ cellulitis
ที่ขาซ้าย เชื้อก่อโรคในข้อ
ใดต่อไปนี้น่าจะทำาให้เกิด
อาการในชายผู้นี้มากที่สุด
a. Salmonella spp.
b. S. aureus
c. H. influenzae
d. Mycoplasma
pneumonia
e. Mycoplasma
tuberculosis
8. ผู้ชายอายุ 50 ปี กินเหล้า
มานาน 5 วัน มาด้วยปวด
ท้อ ง ลิ้น ปี่ ร้าวไปหลัง
นอนงอตัวดีขึ้น แพทย์
ทำาการตรวจร่างกายพบ
ว่ า มี tenderness and
guarding at epigastrium,
liver dullness +ve ข้อใด
คือการวินิจฉัยในผู้ป่ วย
รายนี้ที่เหมาะสมที่สุด
a. acute pancreatitis
9. มี rash เป็ นแบบ vesicle
ตามแนวซี่โครง มีแสบๆ
เ จ็ บ ๆ ข้ อ ใ ด คื อ ก า ร
วินิจฉัยในผู้ป่ วยรายนี้ที่
เหมาะสมที่สุด
a. VZV
10. ผู้ป่ วยหญิงอายุ 40 ปี มี
อาการปวดบวมต้นขาซ้าย
และมีไข้ แพทย์ทำา การ
ตรวจร่างกายพบว่ามี BT
= 39
o
C, erythema and
tender at left upper thigh
เจาะแล้วได้หนอง เชื้อก่อ
โรคในข้อใดต่อไปนี้น่าจะ
ทำาให้เกิดอาการใน ผู้ป่ วย
รายนี้มากที่สุด
a. S. aureus
b. S. pyogenes
เพราะได้หนอง
c. H. influenzae
d. P. aeruginosa
11. ผู้ป่ วยชายไทย สูงอายุ
ประวัติโรคประจำาตัวเดิม
เ ป็ น DM, HT ครั้งนี้ม า
ด้วยอาการชาและอ่อน
แ ร ง 1 ข้ า ง น า น 10
นาที จากนั้น หายเป็ น
ปกติ ข้อใดคือการวินิจฉัย
ในผู้ป่ วยรายนี้ที่เหมาะสม
ที่สุด
a. TIA
b. Temporal lobe
ischemia
c. Cerebella ischemia
d. Bell’s palsy
12. ผู้ป่ วยชายอายุ 18 ปี ก่อน
หน้านี้สุขภาพร่างกายแข็ง
แรงดีไม่มีโรคประจำา ตัว
ค รั้ ง นี้มี อ า ก า ร Upper
respiratory tract infection
ผู้ป่ วยมาด้วยไข้ ไอ หอบ
ผลจากการตรวจ CXR
พ บ ว่ า มี Interstitial
infiltration ยาปฏิชีวนะใด
เหมาะสมที่สุดในผู้ป่ วย
รายนี้
a. Ampicillin
b. Ciprofloxacin
c. Roxithromycin
d. Gentamicin
e. Augmentin
Atypicalpneumonia=Inter
stitial infiltration
Typical จะเป็น alveolar
13. ใ น ก า ร รั บ น้ อ ง ที่
มหาวิทยาลัยแห่งหนึ่ง รุ่น
พี่ได้ให้รุ่นน้องให้กินนำ้า
เปล่าปริมาณมาก หลัง
จากนั้นผู้ป่ วยมีอาการซึม
ปวดศีรษะและ คลื่นไส้
อาเจียน ข้อใดต่อไปนี้น่า
จะทำาให้เกิดอาการใน ผู้
ป่ วยรายนี้มากที่สุด (หลัง
จากนำา ข้อมูลจากแต่ละ
สถาบันมารวมกันแล้ว พบ
ว่ามีจำานวนตัวเลือกเกิน 5
ตัวเลือก)
a. Hypokalemia
b. Hyperkalemia
c. Hyponatremia
d. Hypernatremia
e. Hypocalcemia
f. Acute gastric
dilation
g. SIADH
14. สารพิษที่พบในสีทาบ้าน
ทำา ใ ห้ เ กิ ด อ า ก า ร
peripheral neuropathy
(wrist drop) แ ละ ซี ด คื อ
อะไร-ตะกั่ว
15. ผู้ ป่ ว ย CKD มี K = 7
EKG change จงบอกการ
รักษาที่สำา คัญที่สุดในผู้
ป่ วยรายนี้
Cacium
gluconate
Beta 2
agonist
16. ผู้ป่วยชายอายุ 40 ปี มีไข้
และปวดเมื่อยตามเนื้อตัว
มา 3 วัน ผู้ป่ วยให้ประวัติ
ว่าที่ละแวกบ้านมีนำ้ าำ
และผู้ป่ วยต้องลุยนำ้าออก
จากบ้านทุกวัน แพทย์
ทำาการตรวจร่างกายพบว่า
มี BP = 120/80 mmHg,
PR = 110/min, RR = 24
/min, BT = 39
o
C, mild
jaundice, ling clear, liver
2 cm BRCM ข้ อ ใ ด คื อ
การวินิจฉัยในผู้ป่ วยรายนี้
ที่เหมาะสมที่สุด
a. Enteric fever
b. Dengue fever
c. Leptospirosis
d. Scrub typhus
e. Murine typhus
17. ผู้ป่ วยหญิงอายุ 50 ปี มา
ด้วยอาการปวดแขนขา
ขึ้นบันไดลำาบาก ลุกจาก
เก้าอี้ลำา บาก 4 สัปดาห์
ก่ อ น ม า โ ร ง พ ย า บ า ล
แ พ ท ย์ ทำา ก า ร ต ร ว จ
ร่ า ง ก า ย พ บ ว่ า มี
tenderness of quadriceps
muscle, purple-red
discoloration over the
upper forehead, eyelids
and cheeks ข้อใดคือการ
วินิจฉัยในผู้ป่ วยรายนี้ที่
เหมาะสมที่สุด
a. Polymyositis
b. Dermatomyositis—
proximal mus
weak
c. Psoriasis
d. Mixed connective
tissue disease
e. SLE
18. ผู้ป่ วยชายอายุ 40 ปี มี
อาการไอแห้งๆ มา 1
เดือนก่อนมาโรงพยาบาล
อ า ก า ร เ ป็ น ม า ก ต อ น
อากาศหนาว และตอน
เย็น ไอมีเสมหะสีขาว ไม่มี
เลือดปน ตรวจร่างกาย
ปกติ ผลการตรวจ CXR
พบว่า WNL การปฏิบัติใน
ข้อใดต่อไปนี้น่าจะเหมาะ
สมกับผู้ป่วยรายนี้มากที่สุด
a. Skin test
b. CT chest
c. Sputum AFB
d. Bronchoscopy
e. PFT
19. ผู้ป่ วยชายอายุ 50 ปี กิน
เหล้าขาวมานาน 25 ปี
ครั้งนี้มีอาการเหนื่อย ขา
บวม 1 สัปดาห์ก่อนมา
โ ร ง พ ย า บ า ล แ พ ท ย์
ทำาการตรวจร่างกายพบว่า
มี BP = 120/70 mmHg,
fine crepitation both
lower lungs, PMI 6
th
ICS 1 cm lateral to
MCL, S3 gallop ก า ร
รักษาที่เหมาะสมที่สุดในผู้
ป่วยรายนี้คือข้อใด
a. Digoxin
b. Thiamine<cardiac
beriberi>
c. Atropine
20. ผู้ป่ วยชายหลังจากกลับ
จากนำ้าตก มีไข้และแผล
ดังแสดงในรูป เชื้อก่อ
โรคในข้อใดต่อไปนี้น่าจะ
ทำาให้เกิดอาการใน ผู้ป่ วย
รายนี้มากที่สุด
a. Chigger mite
b. Louse
c. Aedes egypti
d. Tick
e. Flea
21. ผู้ป่ วยชายไทย มาโรง
พยาบาลด้วยอาการชาที่
บ ริ เ ว ณ ต้ น ค อ แ พ ท ย์
ทำาการตรวจร่างกายพบว่า
มี BP 60/30 mmHg ไม่มี
sacral reflex ข้อใดคือการ
วินิจฉัยในผู้ป่ วยรายนี้ที่
เหมาะสมที่สุด
a. Cardiogenic shock
b. Spinal shock
c. Neurogenic shock
d. Hypovolemic shock
e. Obstructive shock
22. วัยรุ่นชายอายุ 18 ปี แข็ง
แ ร ง ดี มี อ า ก า ร ห อ บ
หายใจลำาบาก ตอนออก
กำาลังกาย และตอนกลาง
คืน มีอาการ 2-3 ครั้งใน
เ ดื อ น ที่ผ่ า น ม า แ พ ท ย์
ทำาการตรวจร่างกายเบื้อง
ต้นพบว่าไม่มีความผิดปรก
ติใดๆ การรักษาที่เหมาะ
สมที่สุดในผู้ป่ วยรายนี้คือ
ข้อใด
a. inhale salbutamol
prn + inhale
corticosteroid
b. alpha-agonist as
need
c. steroid ตลอด
d. beta-agonist as
need + steroid
ตลอด
e. Oral bronchodilator
f. Salbutamol
inhalation as
needed
g. Long acting beta2
agonist +
corticosteroid
h. Inhaled
corticosteroid
23. Motor Cycle Accident มี
lesion ที่ T2 ค า ส า ย
Foley Catheter มีอาการ
แน่นหน้าอก เหงื่อออก
มากขึ้น BP 180/110 RR
เพิ่ม ควรทำาอย่างไร
a. off F/C
b. Hydralazine
c. ลุกนั่งปลดเสื้อผ้า
d. นอนหัวตำ่าวัด BP ซำ้า
e. ADVICE
24. ห ญิ ง ไ ท ย 70 ปี no
Underlying Disease
สบายดีมาตลอด มีอาการ
แขนซ้ายอ่อนแรง 5 นาที
EKG เป็น AF ใช้ยาอะไร
ป้ องกันอาการอ่อนแรง
a. ASA
b. Warfarin
c. Clopidogrel
25. ผู้หญิง อายุ .PE: HT +
Cushing appearance
Lab เจออะไร
a. Increase cortisol
level
26. ผู้ ห ญิ ง subacute
thyroiditis ให้ยาใดเพื่อลด
อาการ
a. ASA+propanplol
27. T4 ปกติ TSH ปกติ Dx?
a. Simple Goiter
b. Iodine def
c. Thyroid CA
d. Subacute thyroiditis
e. Hashimoto
Thyroiditis
28. ผู้หญิงอายุ 28 ปี ฉี่ไม่
อ อ ก ต้ อ ง ใ ส่ ส า ย ส ว น
ปั ส ส า วะ 2 ค รั้ง ต่อ มา
ขาอ่อนแรง ชาถึงระดับ
ส ะ ดื อ PE : DTR 3+ ,
loss of sphincter tone ผู้
ป่วยมี lesion อยู่ที่ไหน
a. parasagital sinus
b. Basal pons
c. Thoracic area
d. Lumbosacral area
e. Clonus medularis
29. ผู้ ป่ ว ย ช า ย ไ ท ย เ ป็ น
bronchial asthma พ่ น ย า
ไ ม่ ดี ขึ้ น lung : expirat
Wheezing ก า ร รั ก ษ า ที่
เหมาะสมที่สุดในผู้ป่ วย
รายนี้คือข้อใด
a. iv. Dexametazone
30. EKG : bradycardia ต่ อ
มาตรวจ vital sign พบว่า
ผู้ป่ วยมีความดันโลหิตตก
(80/50 mmHg, HR = 30)
การรักษาที่เหมาะสมที่สุด
ในผู้ป่วยรายนี้คือข้อใด
a. Atropine
b. dobutamine
c. pacemaker
31. ผู้ ป่ ว ย ห ญิ ง อ า ยุ 27 ปี
เจ็บคอมา 2 สัปดาห์ PE:
white patch เต็มคอและ
ลิ้น ถามว่าจะส่งตรวจ
อะไรเพื่อให้ได้เหตุผลของ
โรคที่ดีที่สุด
a. anti-HIV
32. ผู้ป่ วยชาย ให้ EKG อ่าน
เอง (เป็ น Mobitz II) จะ
ให้ยาอะไร
a. Atropine
b. Lidociane
c. Digitalis
d. Cardioversion
33. ผู้ ป่ ว ย ห ญิ ง อ า ยุ 68 ปี
แ พ ท ย์ ทำา ก า ร ต ร ว จ
ร่างกายเบื้องต้นทุกอย่าง
ปกติดี จะให้วัคซีนใด
a. Rabies vaccine
b. Tetanus viccine
c. HBV vaccine
d. Influenza vaccine
e. HPV vaccine
34. ผู้ป่ วยหญิงอายุ 18 ปี มี
อาการเจ็บหน้าอก โน้มตัว
ไปข้างหน้าอาการดีขึ้น
PE: lung clear, มี เ สี ย ง
rub ที่ chest คิดว่าเป็ น
อะไร
a. Endocarditis
b. Pancarditis
c. Myocarditis
d. Pneumonia
35. ผู้ป่วยชายอายุ 16 ปี เป็น
homozygous Beta-thal
ต้องรับเลือดเป็นประจำา มี
อาการอ่อนเพลียและบวม
มา 1 เดือน แพทย์ทำาการ
ตรวจร่างกายเบื้องต้น BP
drop, irregular heart rate
& rhythm,
hepatosplenomegaly ถาม
ว่ า ผู้ ป่ ว ย มี heart
complication จากอะไร
a. cardiac siderosis
b. Cardiac
hypertrophy
c. Immune
cardiomyositis
d. Extramedullary
hematopoiesis at
heart
36. ผู้ ป่ ว ย ช า ย อ า ยุ 20 ปี
Motorcycle Accident ทำา
CT พ บ crescent shape
at frontal region ในผู้ป่ วย
รายนี้น่าจะคิดถึงโรคใด
มากที่สุด
a. Acute epidural
hemorrhage
b. Acute subdural
hemorrhage
c. Subacute epidural
hemorrhage
37. clinical rheumatoid
arthritis ยาใดลดปวดเร็ว
สุด
a. Ibuprofen
b. Paracetamol
c. MTX
d. Sulfazalazine
e. Chloroquine
38. ผู้ป่ วยสูงอายุมีโรคประจำา
ตั ว เ ป็ น DM with HT มี
แ พ ท ย์ ทำา ก า ร ต ร ว จ
ร่ า ง ก า ย เ บื้ อ ง ต้ น
Costovertebral angle
tenderness ก าร รัก ษ า ที่
เหมาะสมที่สุดในผู้ป่ วย
รายนี้คือข้อใด
a. Ceftriaxone IV
b. Ceftriaxone กิน
c. Ofloxacin กิน
39. ชายอายุ 22 ปี หมดสติ
ในห้องนำ้า ไม่มีไข้ ตรวจ
ร่างกายพบ PR ตำ่า RR
ตำ่า miosis การรักษาที่
เหมาะสมที่สุดในผู้ป่ วย
รายนี้คือข้อใด
a. Atropine
b. Naloxone
c. Diazepam
40. ผู้ป่ วยหญิง 40 ปี หนัก
80 kg สูง 160 cm. เพิ่ง
Dx. DM, FBG 180 ก า ร
รักษาที่เหมาะสมที่สุดในผู้
ป่วยรายนี้คือข้อใด
a. ควบคุมอาหาร
b. Metformin
c. Acarbose
d. Glibenclamide
41. ผู้ป่ วยหญิง 25 ปี ใจสั่น
กินจุ นน.ลด มา 1 ปี ,
PE: mild exophthalmos,
generalized enlarged
thyroid both lobe, ….
(Graves’), T4..,T3 400,
TSH <0.001 การรักษาที่
เหมาะสมที่สุดในผู้ป่ วย
รายนี้คือข้อใด
a. Propanolol
b. Lugol solution
c. PTU
d. Radioactive iodine
e. Subtotal
thyroidectomy
42. ชาย 32 ปี มีอาการใจสั่น
นำ้าหนักลด BP 150/100
mmHg PR 120/min PE:
neuro sensory normal,
muscle weak gr. 1/5
both upper and lower,
reflex 2+ all
Na 136 mmol/l K
2.5 mmol/l
Cl 102 mmol/l
CO2 22 mmol/l
ต้องส่งตรวจอะไรเพื่อ
เป็นการหาสาเหตุของอาการ
อ่อนแรง
a. Free T4
b. Plasma glucose
c. CT brain
d. CSF exam
43. ผู้ป่ วยชายมีอาการใจสั่น
มี muscle weakness
ตรวจ DTR 1+, กล้ามเนื้อ
แขนขาอ่อนแรง 1+, ลุก
ไม่ขึ้น K 2.5 ถามว่าตรวจ
อะไรเพิ่มเติม
a. Free T4
b.
44. หญิง 15 ปี มี webbed
neck ไม่มีเต้านม ข้อใดถูก
a. Insulin ตำ่า
b. GH ตำ่า
c. Estrogen สูง
d. Gonadrotropin สูง
e. Parathyroid
hormone สูง
45. ชาย 30 ปี ไอมา 1
เดือน เสมหะขาวมีเลือด
ปนบางครั้ง ไข้ตำ่าๆ นำ้า
หนักลด 3 kg, PE:
normal, CXR: Reticulo-
patchy infiltration with
thick wall cavity, AFB
sputum: neg X 3 ครั้ง
ควรทำาอะไรต่อ?
a. Bronchoscopy
b. FNA
c. Anti-TB drug
d. รอผล culture for
TB
46. ชายไทย 30 ปี อาชีพขับ
รถ bus มีไข้ตำ่า ไอแห้ง
นำ้าหนักตัวลดลง, ตรวจ
พบ AFB 3+, แพทย์ให้ยา
TB ควรเขียนให้ลาหยุด
งานนานเท่าใด
a. 2 wk.
b. 1 mo.
c. 2 mo.
d. ทีละ 1 wk จนกว่า
AFB จะ neg
e. ไม่จำาเป็นต้องหยุด
งาน
47. ช า ย อ า ยุ 40 ปี โ ร ค
ป ร ะ จำา ตั ว เ ป็ น Asthma
เป็ นหวัด 2 วัน PTA พ่น
ย า แ ล้ ว อ า ก า ร ไ ม่ ดี ขึ้ น
แ พ ท ย์ ทำา ก า ร ต ร ว จ
ร่างกายเบื้องต้น T 37.4
o
C BP 110/90 mmHg PR
100 bpm RR 28 /min
Lung: wheezing at both
lower lung with
accessory muscle used
การรักษาที่เหมาะสมที่สุด
ในผู้ป่วยรายนี้คือข้อใด
a. Terbutaline Sc
b. IV steroid
c. IV dexamethasone
d. IV Theophyline
e. NB salbutamol
48. ผู้ ป่ ว ย ห ญิ ง ไ ท ย มี
hyperkeratosis
onycholysis การรักษาที่
เหมาะสมที่สุดในผู้ป่ วย
รายนี้คือข้อใด
a. Topical
cotrimoxazole
b. Topical
itraconazole
c. Topical
ketokonazole
d. Oral itraconazole
49. ผู้ป่วยหญิง มาด้วยอาการ
ผื่น มีสะเก็ด เป็นทั่วศีรษะ
ข้อเข่า ข้อศอก เกาแล้วมี
ผื่นขึ้น แกะสะเก็ดออกมี
เลือดออกเล็กน้อย ในผู้
ป่วยรายนี้น่าจะคิดถึงโรค
ใดมากที่สุด
a. Psoriasis
b. Lichen planus
c. Discoid lupus
erythematosus
50. Pt 50 yr มาด้วย แพทย์
ทำาการตรวจร่างกายเบื้อง
ต้นตัวเหลืองตาเหลือง 1
wk, Cholesterol 350, TG
250 อาจจะพบอาการ
แสดงทางผิวหนังอะไร
a. Eczema
b. Lipoma
c. Xanthelasma
d. Seborrheic
dermatitis
51. หญิง 15 ปี มาด้วย
อาการปวดศีรษะข้างเดียว
มา 4 เดือน มีอาการ
เดือนละ 2-3 ครั้ง จงให้
Prophylaxis
a. Propanolol
b. Sumatriptan คือ
ทันที acute
c. Flunazine
d. A,B
e. A,C
52. ผู้ป่วยชาย 30 yr ปวด
ศีรษะมากทันที เวียนหัว
คลื่นไส้อาเจียน
drowsiness, BP 150/110,
PR 80, RR 22, stiffness
of neck, Babinski’s sign
- neg, Retinal a. -
absence of pulse, no
papilledema ในผู้ป่วยราย
นี้น่าจะคิดถึงโรคใดมาก
ที่สุด
a. Migraine
b. Viral encephalitis
c. Subarachnoid
hemorrhage
d. Bacterial meningitis
53. หญิง 35 yr หนังตาตกมา
4 wk เป็นมากขึ้นตอ
นบ่ายๆ ตรวจตาซ้ายพบ
ptosis มี mild proximal
muscle weakness,
normal DTR, normal
sensory จง Dx
a. Polymyositis
b. Horner’s syndrome
c. Hyperthyroidism
d. Myasthenia gravis
e. CN III palsy
54. ผู้ป่วยชายอายุ 21 yr
อ่อนแรงขาทั้ง 2 ข้าง ต่อ
มาอ่อนแรงไปทั้งตัว มา 3
วัน, Total
ophthalmoplegia, bilateral
facial palsy, paradoxical
breathing, generalized
decrease motor power gr
II-III/V, areflexia, loss of
vibration sense all toe
and fingers ข้อใดคือการ
รักษาที่เหมาะสมที่สุด
diab GBS
a. Azatiopine
b. Interferon beta 1A
c. Cyclophosphamide
d. Methylprednisolone
e. Intravenous
Immunoglobulin
55. ผู้ป่วยหญิง 70 yr มีแขน
ขาซ้ายอ่อนแรง และมี
EKG ดังนี้ (มี ST
elevate)ให้ยาใดป้ องกัน
อาการดังกล่าว
a. Aspirin
b. Enoxaparin
c. Clopidogrel
d. Warfarin
e. Dipyramidamole
56. Male วัยกลางคน มี
อาการปวดบวมเข่าและนิ้ว
โป้ งเท้าขวา เจาะข้อได้
WBC 20,000 และ
Needle shaped crystal
จะเจอ X-ray finding
อะไร
a. Osteoporosis
b. Osteophyte
c. Chondrocalcinosis
d. Bone ankylosis
e. Lumpy soft tissue
57. ผู้ป่วยชายอายุ 50 ปี
ปวดเข่ามา 2 วัน มีไข้
ปวดบวมเข่าขวา ขยับข้อ
เข่าไม่ได้ เนื่องจากมี
อาการปวดมาก PE: BT
38.5
o
C, Rt. knee swelling
and marked tender. Lab
ใดจะมีประโยชน์มากที่สุด
a. Serum uric acid
b. Rheumatoid factor
c. Joint fluid analysis
d. ESR
e. Knee plain film X-
ray
58. ผู้ ป่ ว ย ช า ย อ า ยุ 70 ปี
ปวดเข่าขวามา 3 วัน ได้
ย า diclofenac ไ ม่ ดี ขึ้ น
เ จ า ะ เ ข่ า ไ ด้ turbid
yellow, WBC 20,000
(PMN 50 %, L 20 %)
ESR 50 % gram stain
negative (X-ray : no joint
space narrowing
,sclerotic rim? no
fracture, no osteolytic
lesion)ในผู้ป่ วยรายนี้น่าจะ
คิดถึงโรคใดมากที่สุด
a. septic arthritis
b. Pseugogout
c. Gout
d. Rheumatoid
arthritis
e. Osteoarthritis
59. ชายอายุ 49 ปี มีไข้ ไม่มี
หนาวสั่น เจ็บที่ชายโครง
ข้างขวาร้าวไปที่ไหล่ขวา 3
วันก่อนมาโรงพยาบาล
อุจจาระปกติ ไม่เหลือง มี
เบื่ออาหาร นำ้าหนักลด
PE: Rt lobe liver
enlarged with mild
tenderness, HBsAg
positive
LAB: LFT: TB/DB
= 1.0/0.2
AST = 60
ALT = 70
ALP = 112
Total protein = 5.0
Albumin = 2.3
U/S : Hyperechoic mass
at right lobe liver diameter 6
cm, no intrahepatic duct
dilatation
ผลการวินิจฉัยเป็นอะไร
a. Metastatic CA
b. Amoebic liver
abscess
c. Hepatocellular
carcinoma
d. Peripheral
Cholangiocarcinom
a
e. Postnecrotic
cirrhosis with
hepatic adenoma
60. ชายวัย 52 ปี มีอาการ
เจ็บใต้ชายโครงซ้าย
เฉียบพลัน หลังจาก
อาเจียนมาก PE :
profused sweating,
decrease breath sound
Lt. lung, Abd.: tender
with guarding Dx?
a. Hiatal hernia
b. Ruptured
esophagus
c. Mallory-Weiss
syndrome
d. Tension
pneumothorax
e. Acute MI
61. ชายอายุ 40 ปี ดื่มเหล้า
3-4 ครั้ง/wk มีปวดถ่วงๆ
ท้องน้อย อุจจาระมีเลือด
ปน แต่เลือดไม่ปนเป็น
เนื้อเดียวกับอุจจาระ ข้อ
ใดเป็นคำาแนะนำาเพื่อลด
อาการถ่ายเป็นเลือด
a. เพิ่มอาหารไขมัน
b. เพิ่มอาหารโปรตีน
สูง
c. เพิ่มอาหาร fiber
สูง
d. ลดเครื่องดื่ม
Alcohol
e. ฝึกเบ่งถ่ายอุจจาระ
ให้ถูกวิธีทุกวัน
62. ผู้ป่วยอายุ 45 ปี ดื่มเหล้า
ขาววันละ 1 ขวด มา 25
ปี ถ่ายเหลาวมา 3-4
ครั้ง/d เป็นเวลา 2 วัน มา
ด้วยอาการสับสน จำาญาติ
ไม่ได้ ตรวจร่างกาย V/S
ปกติ พบ scaling lesion
บริเวณหน้า คอ และ
แขน 2 ข้าง ถาม Dx
a. Pellagra B3 def
dementia diarrhea
dermatitis
b. Subdural
hematoma
c. Wernicke’s
encephalopathy
d. Electrolyte
imbalance
e. Delirium tremens
63. ผู้ป่วยชาย อายุ 33 ปี
dyspnea, engorged neck
vein, no murmur, EKG:
low voltage, generalized
ST change with inverted
T ในผู้ป่วยรายนี้น่าจะ
คิดถึงโรคใดมากที่สุด
a. Constrictive
pericarditis
b. Cardiac tamponade
c. RBBB
d. Lateral wall
infarction
e. Pulmonary
embolism
64. ผู้ป่ วยชายอายุ 18 ปี มี
อาการเหนื่อยขึ้นมาทันที
ไอเป็นฟองสีชมพู แพทย์
ทำาการตรวจร่างกายตรวจ
ร่ า ง ก า ย พ บ ว่ า มี pink
frothy sputum, crepitation
both lower lungs, BP
160/100 mmHg ไ ด้ ใ ห้
furosemide และ O2 ไป
การรักษาเพิ่มเติมที่เหมาะ
สมที่สุดในผู้ป่ วยรายนี้คือ
ข้อใด
a. Digoxin
b. Propanolol
c. Dobutamine
d. Nitropusside
e. Morphine
f. Mg
65. ผู้ป่วยหญิงอายุ 32 ปี
Underlying Rheumatic
heart disease with Mitral
valve stenosis ขณะนี้ได้
ยา Digoxin(0.25mg) 1
tab OD , Furosemide(40
mg) 1 tab OD โดย 3 วัน
ก่อนมา รพ. ผู้ป่วยมี
อาการเหนื่อยมาก คลื่นไส้
อาเจียน เบื่ออาหาร เห็น
แสงสีผิดปกติ PE: BP
100/70 mmHg, HR 50
bpm, Lung fine
crepitation both lung,
Heart loud S1, diastolic
murmur at apex อะไร
เป็นสาเหตุที่ทำาให้เกิด
อาการมากขึ้น
a. Hypokalemia
b. Hyponatremia
c. Drug interaction
d. Volume depletion
e. Metabolic alkalosis
66. ผู้ ป่ ว ย ห ญิ ง อ า ยุ 32 ปี
Underlying Rheumatic
heart disease with Mitral
valve stenosis ขณะนี้ได้
ย า Digoxin(0.25mg) 1
tab OD , Furosemide(40
mg) 1 tab OD โดย 3 วัน
ก่ อ น ม า ร พ . ผู้ ป่ ว ย มี
อาการเหนื่อยมาก คลื่นไส้
อาเจียน เบื่ออาหาร เห็น
แ ส ง สี ผิ ด ป ก ติ PE: BP
100/70 mmHg, HR 50
bpm, Lung fine
crepitation both lung,
Heart loud S1, diastolic
murmur at apex ก า ร
รักษาที่เหมาะสมที่สุดในผู้
ป่วยรายนี้คือข้อใด
a. เพิ่มยา Atropine
b. เพิ่มยา beta
Blocker
c. หยุดทานยา
Digoxin
d. เพิ่มยา Steroid
e. เพิ่มยา Furosemide
67. ผู้หญิงอายุ 60 ปี สูบบุหรี่
มา 30 ปี ไอมาก X-ray
พบ Right upper lung
mass และมี Na ในเลือด
ตำ่า (urine osm 50) เกิด
จากอะไร
a. SIADH
b. Polydipsia
68. patient CHF … Bx:
lymphocyte infiltration,
most common virus?
a. Adenovirus
b. Coxsackie B
c. EBV
d. CMV
69. ชายอายุ 30 ปี นำ้าท่วม
บ้าน มาด้วยปวดกล้าม
เนื้อ ไข้ Dx?
a. Leptospirosis
70.ชาย 65 ปี U/D AAA มีจำ้า
เลือดตามตัว PE: multiple
ecchymoses, อื่นๆปกติ,
CBC: Hb 10, WBC
7,200 (N 75, L 25), Plt
30,000, PT,PTT,TT :
prolong, fibrinogen ↓, จะ
ตรวจอะไรเป็นประโยชน์
ที่สุดในการบอกสาเหตุ ?
a. D-dimer
b. Euglobulin lysis
time
c. Blood smear
71. ลูกคนแรกเป็น HbH จง
หา Typing ของแม่และพ่อ
Hb MCV
พ่อ 13 76
แม่ 13 87
a. พ่อเป็น alpha
alpha, - -
b. แม่เป็น alpha -,
alpha -
72. กินเหล้าเถื่อน แล้วมี
อาการอาเจียน หายใจ
หอบ Na? K?
a. Methanol
b. Ethylene glycol
c. Salicylate
d. Paraquat
73. ผู้ชายอายุ 20 ปี ถูกสุนัข
หน้าโรงหนังกัด แผล
ขนาด 2 x 1 cm หลัง
จากทำาแผลให้ TT แล้ว
จะป้ องกัน Rabies ยังไง
a. ให้ Rabies vaccine
b. ให้ Rabies Ig
c. แนะนำาให้สังเกต
อาการสุนัขอีก 1
สัปดาห์
d. ตัดหัวสุนัขส่งตรวจ
e. ให้ Rabies vaccine
และ Rabies Ig
74. ผู้ป่ วยชายหมดสติมาที่
ER ผลการทำา EKG เป็ น
ดังรูป (VF)การรักษ าที่
เหมาะสมที่สุดในผู้ป่ วย
รายนี้คือข้อใด
a. Defibrillation
b. Atropine injection
c. Adenosine injection
d. Adrenaline injection
75. หญิงอายุ 35 ปี มาที่ ER
ประวัติว่ากินส้มตำาปู
ปลาร้า พริกขี้หนู 10 เม็ด
หลังจากนั้น 2 ชั่วโมง มี
อาการชารอบปาก น้อง
สาวที่ทานด้วยกันก็มี
อาการเช่นเดียวกับผู้ป่วย
PE: BT 37
o
C , BP
140/90, HR 120, RR
14/min with shallow
breathing, motor
gradeII/V, reflex 1+ ถาม
ว่าเกิดจากพิษอะไร
a. Saxitoxin
b. Tetradotoxin
c. Botulinum toxin
76. ผู้ ป่ ว ย ใ น ICU เ ป็ น
Subarachnoid
Hemorrhage ใส่ ET tube
แ ล ะ Foley’s cath 1 wk
ต่อมาปัสสาวะมีสีขุ่นย้อม
เป็น candida albican การ
รักษาที่เหมาะสมที่สุดในผู้
ป่วยรายนี้คือข้อใด
a. Itraconazole
b. Clotrimazole
c. Fluconazole
d. Amphoterin B
e. ถอดสาย Foley’s
cath
77. ทหารเกณฑ์ออกฝึกมี
อาการหน้ามืด เป็นลม
และมีอาการชักแพทย์
ทำาการตรวจร่างกายเบื้อง
ต้น BT 41.1
O
C , BP
80/60 mmHg , PR
140 /min RR 25/min ในผู้
ป่วยรายนี้น่าจะคิดถึงโรค
ใดมากที่สุด
a. Heat stroke
b. Subdural
hematoma
c. Dehydrate
d. Encephalitis
78. ผู้ป่วยชาย 75 ปี อัมพาต
bed sore ไข้ ซึม ฉี่ไม่
ออก 2 วัน Stupor , mild
pale , no Jx , no
hepatosplenomegaly ,
Hb 9 , MCV 98 , WBC
14000 , N90 , L10 , Plt
20000 , PT18 , aPTT 36
, มี PBS เป็น MAHA
Blood picture ในผู้ป่วย
รายนี้น่าจะคิดถึงโรคใด
มากที่สุด
a. primary fibrinolysis
b. Vit K def
c. Thalassemia with
sepsis
d. DIC
e. TTP
(MAHA พบได้ในโรคต่อไปนี้
นะครับ TTP, HUS, Malignant
HT, Prostatic heart vale,
DIC,etc)
79. ผู้ป่ วยชายไทยรายหนึ่ง
ตรวจร่างกายประจำาปี มี
ผลเลือดดังต่อไปนี้
TG = 210, LDL =
200, HDL = 30
ยาในข้อใดน่าจะเหมาะ
สมกับผู้ป่ วยรายนี้มากที่สุด
ที่สุด
a. Gemfibrozil
b. Simvastatin
c. Fibric acid
d. Niacin
e. Cholestyramine
f. Fenofibrate
80. ผู้ป่วยชายอายุ 56 ปี มา
ด้วยเจ็บแน่นหน้าอกด้าน
ซ้าย 2 ชม เป็น DM &
HT มา 10 yrs ให้ EKG
มาเป็น STEMI , V1-V4
หลังจากให้ ASA ไปแล้ว
การรักษาใด เหมาะสม
ที่สุด
a. Enoxaparin
b. Warfarin
c. Streptokinase
d. Balloon
e. PCI
81. ชาย 56 ปี เป็นโรคตับ
แข็งมา 2 ปี ถ่ายเหลว 3-
4 ครั้งต่อวัน มา 3 วัน
ก่อนมารพ. วันนี้ซึมลง
BT= 37.3 PR=98 RR=18
BP104/80 stupor , mild
jaundice , mild ascites ,
spider nevi + palmar
erythema จะส่ง
Investigation ใดเพื่อช่วย
ในการวินิจฉัยมากที่สุด
a. CBC
b. Stool exam
c. Coagulogram
d. LFT
e. Abdominal
paracentesis
82. ผู้ป่วยชายอายุ 42 ปี สูบ
บุหรี่ 2 ซองต่อปี มานาน
20 ปี ตรวจพบ cyanosis
at finger tips , radial
pulse 1+,brachial pulse
normal Dx?
a. Raynaud’s disease
b. Takayasu’s
disease
c. Polyarteritis
nodosa
d. Thromboangiitis
obliterans
e. Systemic lupus
erythematosus
83. ผู้ป่วยชายอายุ 60 ปี
ปวดบวมแดงร้อนเข่าขวา
มา 1 สัปดาห์ มีไข้ ตรวจ
เข่าขวาพบ บวม อุ่น มีนำ้า
ในข้อ พบ WBC 50,000 ,
PMN 80%, sugar 22
(plasma glucose 105) ไม่
พบ crystal , ทำา G/S &
AFB negative การรักษา
ที่ เหมาะสม คือ
a. Colchicine
b. Ceftriaxone
c. Indomethacin
d. Intraarticular
steroid injection
e. Anti-TB drug
84. เด็กหญิง 6 ขวบ มีจำ้า
เลือดตามตัวเป็น ๆ
หายๆ มา 1 เดือน PE:
generalized ecchymosis
CBC:Hb 12,Hct
36,WBC=8,500,PMN=55
%,Lym=30%,Eo=15%,Plt=
154,000 with pale stain
ตรวจ LAB อะไรผิดปกติ
a. PT
b. aPTT
c. Bleeding time
d. VCT
e. TT
85. ผู้ป่วยหญิงอายุ 50 ปี
ปวดแขนขามา 4 สัปดาห์
นั่งแล้วลุกขึ้นยืนลำาบาก
ขึ้นลงบันไดลำาบาก ตรวจ
ร่างกายพบ purple-red
discoloration ที่หน้าผาก
แก้มและเปลือกตา purple
nodule ที่ศอกและเข่า คิด
ว่าคนนี้เป็นอะไร
a. Psoriasis
b. Dermatomyositis
c. Discoid LE
d. SLE
e. Mixed connective
tissue
86. ชายอายุ 30 ปี ท้องเสีย
3 เดือน Cachexia,
muscular wasting รูปไข่
พยาธิคล้ายถั่วลิสง
(Capillaria philipinensis)
ควรมีวิธีการป้ องกัน
อย่างไร
a. งดกินปู
b. งดกินหอย
c. งดกินผักสด
d. งดกินปลานำ้าจืด
ดิบๆ
e. งดเดินเท้าเปล่าตาม
ทุ่งนา
f. งดดื่มนำ้าที่ยังไม่ได้
ผ่านการต้มสุก
87. หญิงอายุ 42 ปี เดินทาง
มาจากเชียงใหม่ มาตรวจ
FU มะเร็งเต้านมที่
รพ.กรุงเทพฯ ขณะกำาลัง
ลงจากรถทัวร์ มีอาการ
เจ็บอกด้ายขวา และ
หายใจหอบเหนื่อย PE:
PR 128 /min RR 30 /min
lung-clear, Rt leg-Swollen
& tenderness เข้ากับโรค
ใดมากที่สุด
a. Pleuodynia
b. Pneumonia
c. Pulmonary
embolism
d. Acute coronary
syndrome
e. Dissected aortic
aneurism
88. ชายอายุ 18 ปี ไข้สูง เจ็บ
คอ 3 day ซีด เหลือง
moderate pale, mild
Jaundice, liver 2 cm
below costal margin,
spleen 3 cm below
costal margin, Hb 7.9,
Hct 28, MCV 65, WBC
9500, N 80%, L 20%,
Plt 450000 ในผู้ป่วยรายนี้
น่าจะคิดถึงโรคใดมาก
ที่สุด
a. AIHA
b. HbH with crisis
c. PNH
d. G6PD
e. DIC
89. ชายอายุ 50 ปี ปวดเข่า
ขวามาก มีไข้ P.E. : Rt.
Knee swelling , Tender
ปวดมาก เดินไม่ได้ ต้อง
อะไรต่อ
a. X-ray
b. Synovial analysis
90. ผู้ป่วย Cirrhosis มีท้อง
เสีย ถ่ายเหลว 3-4 ครั้ง
ต่อวัน มา 3 วัน มี
อาการซึม ตรวจพบ Sign
ของ Chronic liver
disease และ Moderate
ascites ต้อง investigation
ใดเพิ่มเพื่อช่วย Dx
a. Abdominal
paracentesis
b. Stool exam
c. LFT
d. CBC
91. ชาย อายุ … ปี มีไข้เจ็บ
คอ P.E. : Tonsil
enlargement , greyish
patch at pharynx and
tonsils gl. , anterior
cervical L.N. enlargement
both side , ตับม้ามไม่โต
ในผู้ป่วยรายนี้น่าจะคิดถึง
โรคใดมากที่สุ
a. Diptheria
92. ชายอายุ 23 ปี มาด้วยเจ็บ
อก หอบเหนื่อย นอนราบ
ไม่ได้ Dx : CHF ต่อมา
เสียชีวิต ตรวจศพพบ
Lymphocyte infiltrate ที่
กล้ามเนื้อหัวใจ ถามหา
เชื้อ
a. Influenza
b. Adeno virus
c. Coxakie B
d. EBV
93. ผู้ป่วยชายอายุ 45 ปี
ปวดหัวมาก 1 วัน โดยมี
อาการมาประมาณ 1-2
เดือน ใจสั่นเป็นพักๆ วัน
ละ 1-2 ครั้ง เคยไปพบ
แพทย์ตรวจพบว่าความดัน
200/120 mmHg ได้ยามา
ทานแต่ทานไม่สมำ่าเสมอ
ตรวจความดันได้ 210/130
mmHg PR 120 /min RR
22/min ตรวจตา Fundus
พบ Flame shape
hemorrhage with early
papilledema นอกจาก
Furosemide แล้วจะให้ยา
ลดความดัน
a. Enalapril
b. Propanolol
c. Prazosin
d. Nitroprusside
e. Spironolactone
94. ชาย 45 ปี กินเหล้ามา
นาน มาร.พ.ด้วยเลือด
ออกใน stomach ขณะนี้
เลือดหยุดแล้ว ตอนนี้มี
อาการกระสับกระส่าย
กระวนกระวาย มี
restlessness , sweating ,
BP = 150/100 , PR =
120 , Tremor both hands
จะ Treatment อย่างไร
a. Chlorpromazine
IM
b. Diazepam IV
c. Haloperidol IM
d. Thiamine IM
95. ชายอายุ 60 ปี มีตุ่มนำ้า
ขึ้นเป็นกลุ่มบริเวณหลังไป
ถึงใต้ชายโครง ปวดแสบ
ร้อน ถามว่าเกิดจากโรค
อะไร
a. Cellulitis
b. Contact dermatitis
c. Dermatitis
dermatiformis
d. Herpes simplex
infection
e. Vericella zoster
infection
96. ผู้ป่วยมี anaphylaxis
shock ต้อง treatment
อย่างไร
a. IM adrenaline ดี
กว่า Subcutaneous
adrenaline confirm
97. ผู้ป่วยหญิงอายุ 50 ปี
ขณะดูโทรทัศน์ ขณะดู
โทรทัศน์ที่โซฟา มีอาการ
ปวดท้ายทอย ลุกเดินขึ้น
มีเดินเซขวา PE : pupil
1mm แขนขาอ่อนแรง
ถามหา lesion
a. Pontine
hemorrhage
b. Cellebellar
hemorrhage
c. Subarchnoid
hemorrhage
d. Basal ganglion
hemorrhage
e. Intraventricular
hemorrhage
98. ผู้ป่วยชาย 30 ปี มาด้วย
ผื่นขาว ขอบเขตไม่ชัดเจน
บริเวณซีกเดียวของ
ร่างกายลามช้าๆ ไม่คัน
แต่มีอาการชาบริเวณผื่น
ในผู้ป่วยรายนี้น่าจะคิดถึง
โรคใดมากที่สุด
a. Vitiligo
b. Pityriasis versicolor
c. Tinea corporis
d. Leprosy
e. Syphilis
99. หญิงอายุ 45 ปี มาด้วย
เหนื่อย 2 เดือน ตรวจ
ร่างกาย PR เร็ว มี
heave, totally irregular
มี pitting edema , JVP
up to mandible , tapping
apex , tapping
parasternum, loud P2
(แต่มีอีกสถาบันหนึ่งบอก
ว่าเป็น loud S1 ???),
opening snap , มี
Diastolic rumbling
murmur ที่ apex ในผู้
ป่วยรายนี้น่าจะคิดถึงโรค
ใดมากที่สุด
a. Rheumatic carditis
b. Mitral valve
prolapsed
c. Infective
endocarditis
d. Mitral annulus
calcification
e. Rupture chordate
tendinae
100. ชายวัยกลางคน มี
ไข้ ไอ เหนื่อย ลม
หายใจมีกลิ่นเหม็น
CXR :cavitary lesion with
air fluid level ในผู้ป่วย
รายนี้น่าจะคิดถึงโรคใด
มากที่สุด
a. Lung abscess
101. ผู้ป่ วย CRF ,urine
ออกน้อย, BP สูง , K 7 ,
EKG change การรักษาที่
เหมาะสมที่สุดในผู้ป่ วย
รายนี้คือข้อใด
a. 10% calcium
gluconate
102. Pt. มี ไ ข้ ตำ่ า ไ อ
เรื้อรัง นำ้าหนักลด CXR
:cavitary lesion +
infiltration at RUL , AFB
negative 3 ค รั้ ง ก า ร
รักษาที่เหมาะสมที่สุดในผู้
ป่วยรายนี้คือข้อใด
a. Tx. TB เลย
103. Pt. มีอาการปวดหัว
เป็นๆหายๆ มี HT ตรวจ
พบ Na 140, K 3.0,
สาเหตุของ HT คืออะไร
a. Pheochromocytoma
b. Primary
hyperaldosteronism
c. Renal a. stenosis
104. Pt. ,มี tinia ungium
การรักษาที่เหมาะสมที่สุด
ในผู้ป่วยรายนี้คือข้อใด
a. Topical
ketoconazole
b. Oral ketoconazole
c. Oral itraconazole
d. Topical
itraconazole
Topical
105. ผู้ป่ วยได้ penicillin
มานาน ต่อมา diarrhea
ต ร ว จ proctoscope พ บ
ลักษณะ เข้าได้กับ AAC
การรักษาที่เหมาะสมที่สุด
ในผู้ป่วยรายนี้คือข้อใด
a. Metronidazole
106. pt. post arrest มี
EKG ให้ดูเป็ น VT การ
รักษาที่เหมาะสมที่สุดในผู้
ป่วยรายนี้คือข้อใด
a. Defibrilation
107. pt pale + jx ตับ
ม้ามไม่โต จะส่งตรวจ
อะไร:
a. Hb typing
b. inclusion body
c. BMA
d. G-6 PD screening
e. Coomb’s test
108. ผู้ป่ วย Clinical DKA
การรักษาที่เหมาะสมที่สุด
ในผู้ป่วยรายนี้คือข้อใด
a. i.v. insulin
b. i.v. fluid
109. ผู้ป่ วย clinical DIC
การรักษาที่เหมาะสมที่สุด
ในผู้ป่วยรายนี้คือข้อใด
a. ATB
b. Plt.
c. FFP
110. รูป EKG เป็น PVC
3 ตัวติดกัน ในผู้ป่วยรายนี้
น่าจะคิดถึงโรคใดมาก
ที่สุด
a. Ventricular
tachycardia
111. รู ป EKG เ ป็ น
complete heart block
การรักษาที่เหมาะสมที่สุด
ในผู้ป่วยรายนี้คือข้อใด
a. Atropine
Pediatrics
1. เด็กอายุ 5 ปี มาด้วยไข้ เจ็บ
คอ ตรวจร่างกายพบ cervical
lymphadenopathy,
Splenomegaly and gray patch
at tonsil เชื้อใดต่อไปนี้เป็นเชื้อ
ก่อโรคในผู้ป่วยรายนี้
A) EBV
B) Diptheria
C) GAS
D) Measles
2. เด็กหญิงอายุ 6 ปี มีผื่นขาว
ขุยเล็กๆ ที่ใบหน้า เห็นชัดเวลา
ว่ายนำ้า เป็ นมานาน 3 เดือน
ไม่คัน PE: Multiple ill-defined,
fine scaly patch,
hypopigmented macule and
patch ที่ห น้ า แ ล ะ ค อ KOH:
negative จงให้การวินิจฉัย
A) Vitiligo
B) Pityriasis alba
C) Pityriasis vesicolor
D) Tinea facialis
E) Contact leukoderm
3. ผู้ ป่ ว ย เ ด็ ก ช า ย อ า ยุ 3 ปี
สบายดีมาตลอด หลังเล่นกับพี่
มีไอ Lung: rhonchi right lung
ควรส่ง investigation อะไร
A) AP, lateral
B) PA, lateral
C) Inspiration and
exspiration
D) Right lateral
decubitus-3yr can’t do C
E) Left lateral decubitus
4. Term newborn BW = 4000
g มารดาเป็ น DM ชักเกร็งทั้ง
ตัว สั่น และมือเท้าเย็น Blood
sugar = 26 mg% ให้การรักษา
ด้ ว ย ส า ร นำ้า ช นิ ด แ ล ะ อั ต ร า
เท่าใดจึงจะเหมาะสมที่สุด
A) Oral 10% D/W 10
ml/kg
B) IV 10% D/W 2
ml/kg+maintain GFR6-
8mg/kg/min
C) IV 10% D/W 4 ml/kg
D) IV fluid with GPR 4-
6 mg/kg/min
E) IV fluid with GPR 6-
8 mg/kg/min
F) IV hydrocortisone 10
mg/kg/day และนมแม่
5. เด็กอายุ 4 ปี มาด้วยไข้และ
ซึม BP = 90/60 mmHg, PR=
120 /min ตรวจร่างกายพบจุด
ตามตัว Stiff neck positive เชื้อ
ที่เป็ นสาเหตุการเกิดโรคในผู้
ป่วยรายนี้มากที่สุดคือ
Neisseria meningitis
6. เด็กหญิงไม่มีประจำา เดือน
ต ร ว จ ร่ า ง ก า ย พ บ webbed
neck, wide nipple distance
and low hair line ค ว า ม ผิ ด
ป ก ติ ข อ ง ผู้ ป่ ว ย ร า ย นี้อ ยู่ ที่
ฮอร์โมนชนิดใด
A) Decrease Insulin
B) Decrease GH
C) Increase estrogen
D) Increase GnRH
E) Decrease PTH
7. เด็กหญิงอายุ 14 ปี ไม่มี
ประจำา เดือนมานาน 8 เดือน
ไม่มีก้อนที่คอ ไม่มีใจสั่น/ เหงื่อ
ออกมากผิดปกติ ปฏิเสธนำ้านม
ไหลผิดปกติ ไม่ได้ลดนำ้าหนัก
หรือออกกำา ลังอย่างหักโหม
ตรวจร่างกายไม่พบความผิดปก
ติ ใ ด ๆ ทำา progesterone
challenge test: negative แ ล ะ
estrogen-progesterone
challenge test: negative ก า ร
วินิจฉัยใดเหมาะสมที่สุด
A) Ovarian failure
B) PCOS
C) Exercise amenorrhea
D) Asherman’s
syndrome
8. เด็กอายุ 5 ปี ปวดท้อง ปวด
ข้อ และพบผื่นขึ้นที่ขาทั้งสอง
ข้าง การส่งตรวจในข้อใดต่อไป
นี้มีประโยชน์ในการประเมิน
prognosis มากที่สุด
A) CBC
B) ESR
C) Skin biopsy
D) UA
9. ปวดข้อและพบผื่นกดเจ็บสี
แดงที่หน้าแข้งทั้งสองข้าง จง
ให้การวินิจฉัย
Erythema nodusum asso
TB
10. NB 35 wk, 2700 g แ ม่
Blood group O, Rh positive
นำ้าเดิน 4 ชั่วโมง หลังคลอด
เ ห ลื อ ง Hct 40% TB = 10
เหลืองจากข้อใดต่อไปนี้เป็นไป
ได้มากที่สุด
A) Sepsis
B) Prematurity
C) Thalassemia
D) G6PD deficiency
E) ABO incompatibility
11. เด็กอายุ 2 ปี สุขภาพแข็ง
แรงดี เมื่ออายุ 1 ปี อยู่สถานรับ
เลี้ยง และเป็ นหวัดบ่อย 1-2
ครั้งต่อสัปดาห์ ครั้งละ 3-5 วัน
ควรแนะนำาอย่างไร
A) Pneumococcal
vaccine
B) ติดเครื่องฟอกอากาศ
ที่สถานเลี้ยงเด็ก
C) ใ ห้ กิ น antihistamine
ป้ องกันอาการ
D) ทำา skin
test
E) ไม่ทำาอะไร
เป็น normal ของเด็ก
12. เด็กอายุ 2 ปี ไข้สูง 1 wk
เจ็บคอ หายใจลำาบาก ตรวจ
ร่ า ง ก า ย พ บ retraction,
greenish patch at posterior
pharynx and tonsils จงให้การ
วินิจฉัย
A) Croup
B) Diptheria
C) Candidiasis
D) Streptococcus
tonsillitis
E) Infectious
mononucleosis
13. เด็กอายุ 6 ปี U/D Asthma
ไข้ ไอ นำ้ามูก 2d หอบมากขึ้น
PE: ไข้ คอแดง suprasternal,
intercostal, subcostal
retraction, inspiratory and
exspiratory wheezing, RR
40/min ใ ห้ salbutamol 3
dose ทุก 20 min หอบน้อยลง
แต่ยังมี exp. wheezing อยู่ ใน
ขณะนั้นให้การรักษาอะไรเพิ่มดี
ที่สุด
A) Oral mucolytic
B) Oral antibiotic
C) IV Aminophylline
D) IV dexamethasone
E) NB. Ipratropium
bromide
14. เด็ก 10 ปี U/D Bthal/HbH
ต้องรับเลือดทุกเดือน แพทย์
พิจารณาทำา splenectomy ก่อน
ที่จะ Sx ควรได้รับ Vaccine ใด
เพื่อ prophylaxis
A. Typhoid vaccine
B. Influenza vaccine
C. Pneumococcal
vaccine
D. Meningococal vaccine
15. ผู้ป่วยหญิง 13 yr คอโต
แบบ generalize มา 3 mo
ไม่มีอาการอื่น FT4 1.25, TSH
3.5 Dx
1. Simple goiter
2. Hashimoto thyroiditis
3. Subacute thyroiditis
4. Iodine deficiency
5. CA thyroid
16. หญิง 2 ปี ขาอ่อนแรง เดิน
ไม่ไหว PE: mild pale, no Jx
มี nodule ที่หัว มีก้อนที่ท้อง
ขนาด 6x8 cm cross midline
hard consistency Dx
A) Neuroblasoma ป ว ด
ท้อง
B) Wilm’s tumor มี
hematuria มีก้อน
C) ANLL
D) Non-Hodkin
lymphoma
E) Retroperitoneal
teratoma
17 เด็กหญิงอายุ 4 ปี มี RUQ
pain with mass urine:RBC 10-
20,WBC 1-2 จงให้การวินิจฉัย
โรค
a. Wilm’s tumor
b. Neuroblastoma
c. Lymphoma
d. Hydronephrosis
e. ..............
18. เด็กชาย 3 ปี ไข้ ซีด เหลือง
reticulocyte 7% ให้ตรวจอะไร
ต่อ
A) G6PD screening
19. เด็กชาย 9 ปี เรียนป.3
เขียนหนังสือไม่ค่อยได้ ต้อง
สะกดทีละคำา IQ test ได้ 109
คะแนนการอ่าน 79 การเขียน
80 การคำานวณ 97 ถาม
เด็กเป็นอะไร
A) dyslexia
B) dyscalculia
C) mental retardation
D) Attention deficit
disorder
E) Autism
20. เด็กอายุ 3 ขวบ ไข้มา 3
วัน แล้วซีด PE : Moderately
pale , mild icteric sclera , No
hepatosplenomegaly + ตรวจ
ร่างกายอย่างอื่นปกติ Lab :
Hb 7.7 , WBC 4,800 , Lym
เด่น ,plt 380,000 , RC 7.5 ,
MCV 87 ต้องตรวจอะไรเพิ่ม
A. Coomb’s test
B. Inclusion test
C. Hemoglobin typing
D. G6PD screening
E. Bone marrow
aspiration
21. เด็กกินปลาทอด มีผื่นขึ้น
ตามตัว หายใจมีเสียง wheeze
ประมาณ anaphylaxis ถามว่า
first med คืออะไร
ตอบ Adrenaline (Epinephrine)
22. เด็ก 1 ปี มีไข้ซึม 1 วัน มี
ไ ข้ V/S T 39.8 , PR 140,
RR 40, BP 90/60, CBC :
Hb 11, WBC 20,000 N 93% ,
Plt 50,000 Coagulogram :
prolong , Petechiae and
Necrotic purpura at legs ถาม
ว่า การ manage ที่สำาคัญ คือ
A) fluid therapy
B) antimicrobial
C) Plt
concentration
D) FFP
E) vasopressin
23. เด็กคลอดที่บ้าน กินแต่นม
แ ม่ มี ecchymosis ต า ม ตั ว
ตอบ Vit K deficiency
ผู้ป่วยเด็ก มีหนังตาบวม ตัว
บวมทั่วทั้งตัวมา 2 เดือน ตรวจ
ร่างกายมี Ascites UA :
Proteinuria 3+ , RBC 0-1/HPF
, Fine granular cast 0-1 , Fat
oval cast 0-1 อะไรทำาให้เกิด
การบวม
A. Decrease Plasma
oncotic pressure
B. Increase Plasma
Hydrostatic pressure
C. Oncotic pressure ใน
cavity เพิ่มขึ้น
D. Hydrostatic pressure ใน
cavity ลดลง
E. Increase Vascular
permeability
24. เด็กหญิงอายุ 12 ปี เป็นลม
หมดสติ 1 นาที หลังฟื้ นปกติ
ไ ม่ เ ค ย เ ป็ น แ บ บ นี้ม า ก่ อ น
management อะไรต่อ
A. EKG
B. Blood glucose
C. 24 – hr holter
monitoring
D. EEG
E. No further
investigation
25. ผู้ป่วยหญิงเป็น TB AFB
2+ CXR พบ cavity มีลูกอายุ
6 เดือน ตรวจ PPD 4 mm
ถามว่าจะ manage ลูกอย่างไร
26. เด็กชาย 8 ปี มีปวดท้อง
คลื่นไส้อาเจียน ปัสสาวะบ่อย
มา 3 วัน มีไข้ BP 90/70, PR
120, RR 30, poor perfusion,
Lab: Na 145 Cl 110
HCO3 10, urine ketone 4+
Management ที่ต้องทำาเป็น
อันดับแรกคืออะไร
1. Insulin
2. 0.9%NaCl
3. Potassium
4. HCO3
27. ผู้ป่วยเด็กอายุ 8 ปี มีค่า
PT, PTT prolong เจาะเลือด
พบ Platelet 50,000 ตรวจ
ร่างกายพบ purpura, petechiae
จะ management อย่างไร
A) FFP
B) Cryo
C) Platelet conc.
28. ผู้ป่วย newborn มีประวัติ
แม่ตั้งครรภ์ 41 wk with
meconium stain AF + fetal
distress จะทำาอย่างไร
A) เช็ดตัว + ดีดเท้า
B) ET tube + suction
29. เด็ก term อายุ 2 เดือน
คลอดที่บ้าน BW 4,600 gm
PE: BT = 37, PR = 150 AF
bulging, mod pale, no jx,
CBC: WBC 8,500 platelet
190,000 คิดว่าเป็นอะไร
A) sepsis
B) SDH
C) Idiopathic vitamin K
deficiency of infant
30. ผู้ป่วย newborn ตรวจ
ร่างกายพบ heart murmur,
cataract, ตับม้ามโต คิดว่า
congenital infection เชื้อใด
(Rubella)
31. ผู้ป่วยเด็กอายุ 6 เดือน มี
อาการเขียวมาตั้งแต่อายุ 4
เดือน ครั้งนี้มีเขียวมากขึ้นเมื่อ
ร้องไห้ PE: tachycardia,
tachypnea, pansystolic
murmur 2/6 at left parasternal
border, cyanosis คิดว่าเป็น
ภาวะใด
1. Hypoxic spell
2. Asthma
3. Pulmonary...
32. ผู้ป่วยเด็กอายุ 2 ปี 4 วัน
PTA มีไข้ 38 องศา PE:
drooling กลืนลำาบาก, posterior
pharynx บวม Dx
A) Acute tracheitis
B) Acute epiglottitis
C) Viral croup
D) Laryngitis
E) Retropharyngeal
abscess
33. ผู้ป่วยเด็กอายุ 3 ปี มารับ
วัคซีน เคยรับวัคซีนครบจนถึง
อายุ 1 ปี จากนั้นไม่ได้รับวัคซีน
อีกเลย จะให้วัคซีนใดใน visit
นี้
A) OPV, DPT
B) Hib
C) MMR, OPV, DPT
D) HBV
E) OPV, DPT, JE
34. ผู้ป่วย newborn preterm
GA 32 wk นำ้าหนักแรกคลอด
1,500 gm หลังคลอด 12
ชั่วโมงต่อมามีอาการหายใจ
ลำาบาก ถามว่าขาดสารตัวใด
ในปอด
A. Spyringomyelin
B. Lacitin
C. Phosphatidyl inositol
D. Phosphatidyl glycerol
35. ผู้ป่วยเด็กอายุ 4 ปี มีไข้
39 C PE: milky patch at
tonsil, cervical LN โต 1-2
node ขนาด 2 cm 2 ข้าง,
นอนกรน หายใจลำาบาก liver
2 cm BRCM, spleen 2 cm
BLCM จงให้การวินิจฉัย
1) Infectious
mononucleosis
2) TB lymphadenitis
3) Streptococcal
pharyngitis
4) Dipthelia
5) Lymphoma
36. ผู้ป่วยเด็กอายุ 4 ปี ปู่เป็น
TB ตรวจร่างกายไม่พบ BCG
scar CXR ปรกติ ทำา
tuberculin test 13 mm จะทำา
อย่างไรต่อไป
A) INH prophylaxis
B) start ยา TB
C) repeat tuberculin test
อีกที 3 เดือนต่อมา
D) ฉีด BCG
E) AFB from
gastric lavage
37. ผู้ป่วยเด็กอายุ 9 เดือน
กินนมแม่ มีไข้ตำ่า ๆ ได้รับยา
para, amoxy มา 3 วัน ต่อมา
ถ่ายเหลวเป็นนำ้า PE: sunken
eyeball, dry lips, perianal
redness ท่านคิดว่าเกิดจาก
สาเหตุใดมากที่สุด
A) Salmonella
B) Shigella
C) Rotavirus
D) Lactose deficiency
E) Antibiotic related
diarrhea
38.เด็กชาย 1 ขวบ มีไอ มี
นำ้ามูก barking cough,
inspiratory stridor, expiratory
rhonchi จะทำาอย่างไรนอกจาก
ให้ IV
1) Ampicilin IV
2) Cloxacilin IV
3) พ่น beta-agonist
4) พ่น adrenaline
5) ETT
39. เด็กอายุ 3 ปี ไอแบบ
Barking cough จง Mx
1. Adrenaline NB stat
2. Salbutamol NB stat
3. Dexamethasone IV
4. Prednisolone po
40. เด็กอายุ 7 วัน ไข้ ซึม ร้อง
เวลาจับเต้านมขวา มีนำ้านมไหล
ตั้งแต่อายุ 2 เดือน มารดาบีบ
ออกทุกวัน , PE: lethargy,
moderate jaundice,
erythematous indurated mass
at Rt. Breast, ควรให้ ATB ?
1. Ceftriaxone
2. Ampicillin +Gentamicin
3. Cloxacillin + Gentamicin
4. Fortum + Amikacin
5. Fortum + Vancomycin
41. ผู้ป่วยเด็กชาย ได้ยา
ampicillin มา 14 วัน หลังจาก
นั้นมีไข้ ท้องเสีย ถ่ายเหลว ทำา
endoscope พบว่ามี hyperemia
Tx?
1. Metronidazole
2. Ceftriaxone
42. เด็กชาย อายุ 8 ปี เมื่อ
ก่อนแข็งแรงดี มีประวัติเป็น
หวัด 1 PTA, มีจำ้าเลือดตามตัว
และเลือดออกตามไรฟัน 2 wk
PTA, vital sign ปกติ, PE :
Oozing per gum, petichiae &
ecchymosis ที่ trunk &
extremities, no
lymphadenopathy, no
hepatosplenomegaly, Dx?
1. Hemophilia
2. Acute leukemia
3. DHF
4. ITP
5. APDE
43. Female Infant 5 hr
APGAR 8,9 ที่ 1,5 min BW
4,200 gm. คลอดมามีอาการ
ชักเกร็ง ตัวยาว 52 cm. Hc 35
cm. The most appropriate
investigation for Dx this
Patience’s Disease is?
1. Plasma Glucose
2. Urine Ketone
3. Serum Ammonia
44.(ชุด A) Case preterm 32
wk. BW 1,500 g ดูหายใจ
เร็วๆ ถามว่าเกิดจากภาวะ
อะไร
1. Respiratory distress
syndrome
เด็กชายอายุ 4 ปี จมนำ้าในสระ
ว่ายนำ้า หลังจาก resuscitation
แล้วหายใจได้เอง แต่ยังไม่ค่อย
รู้สึกตัว นำาส่ง ER P.E. : T =
36.5 , PR = 140 , RR =
55 ,BP = 90/60 , drowsy ,
mild cyanosis , diffuse fine
crepitation and expiratory
wheezing , capillary refill 4
sec , O2 sat 85% On O2
mask with bag 5 LPM ไอมี
frothy pink จะทำาอะไร
A. IV dopamine
B. Nebulized adrenaline
C. Nebulized
bronchodilator
D. On O2 mask with bag
10 LPM
E. Positive pressure
ventilation
45. เด็ก 11 ปี , 1 เดือนก่อน
ไม่สบาย มีจำ้าเลือดตามตัว,
CBC : Plt ตำ่า อย่างเดียว ,Dx?
1. ITP
46. เด็กชาย อายุ 15 ปี ไป
สถานกักกันมา 2 ปี ตรวจ
ร่างกาย พบ Upper/Lower
motor weak gr IV/II DTR no
impaired pain and touch,
glottic stroking pattern
1. Beri Beri
2. GBS
47. เด็กอายุ 7 ปี มีอาการคัน
ตาเป็นๆหายๆ มา 1 ปี ตรวจ
ตา พบ Giant papillae ลักษณะ
เป็น Cubblestone การวินิจฉัย
ที่เป็นไปได้มากที่สุด
1. Trachoma
2. Inclusion Conjunctivitis
3. Hay fever Conjunctivitis
4. Vernal
karatoconjunctivitis
5. Epidemic Conjunctivitis
48. เด็กมาด้วยอาการ
Drooling, miosis, dysphoresis,
profuse sweating ถามว่า เกิด
จากสารใด
1. Ethanol
2. Organophosphate
3. Opioid
4. Ephedrine
5. Cocaine
49. ทารกอายุ 6 ชั่วโมง แรก
คลอดไม่ดูดนม หายใจหอบ
เหนื่อย ฟัง Breath sound ได้
ข้างขวาข้างเดียว เสียงหัวใจอยู่
ทางขวา X-ray พบ heart ที่
Rt.chest ท้องยุบ จง Dx.
1. Situs inversus
2. TE fistula
3. Diaphragmatic hernia
50. เด็กหญิงอายุ 10 ปี เป็นโรค
ผนังกั้นหัวใจรั่วแต่กำาเนิด เคย
เข้ารับการรักษาด้วยหัวใจล้ม
เหลวหลายๆครั้งตอนเล็กๆ
เมื่ออายุ 3 ปี อาการเหนื่อย
น้อยลง ตอนนี้มาด้วยเหนื่อย
มากขึ้นมา 2 เดือน นอนราบได้
PE: central cyanosis, clubbing
finger, Loud P2, diastolic
rumbing murmur 3/6 at Lt
upper parasternal border lung:
clear ข้อใด น่าจะเป็นสาเหตุ
ของการเหนื่อยในเด็กรายนี้
1. Pulmonary embolism
2. Aortic regurgitation
3. Hypoxic spell
4. Eisenmenger complex
5. Congestive heart failure
51. เด็กชายอายุ 12 ปี มาด้วย
อาการเป็นลม ล้มหมดสติไป
1 นาที ตื่นมาปกติดี เกิดขณะ
เข้าแถวตอนเช้า ตรวจร่างกาย
พบว่าปกติ, neurologic exam
normal ต้องทำาอะไรต่อหรือไม่
1. EKG
2. Blood sugar
3. 24hr Holter monitor
4. EEG
5. ไม่ต้องทำาอะไรเพิ่มเติม
52. เด็กอายุ 7 วัน นำ้าหนักแรก
คลอด 2,800 gm กินนมแล้ว
อาเจียนเป็นนำ้าสีเหลือง หลัง
จากนั้นซึม ดูดนมน้อยลง นำ้า
หนักลดเหลือ 2,400 gm จง
Dx. เบื้องต้น
1. Duodenal atresia
2. Pyloric stenosis
3. Inborn error metabolism
4. Sepsis
5. Hirchsprung disease
53. ผู้ป่วยเด็กหญิงอายุ 9 ปี มี
อาการหงุดหงิดง่าย การเรียน
ตกตำ่าลง PE: Thyroid diffuse
enlarged, exophthalmos,
smooth skin. PR120 FT4 เพิ่ม
TSH ลด Antityroglobulin
1:1600 จะรักษาอย่างไร
1. Iodized oil
2. Iodized salt
3. Lugol solution
4. Methimazole
54. เด็ก 6 yr ได้ vaccine ครบ
ตอน 5 yr ถูกแมวข่วนขนาด 2
cm ต้องได้รับ vaccine อะไร
บ้าง
1. Rabies vaccine
2. TT
3. Rabies vaccine +
Tetanus toxoid
4. Rabies vaccine และ
Rabies Ig
5. TT + Rabies vaccine
และ Rabies Ig
55. เด็ก 7 ปี Dx ว่าเป็น
Chickenpox ควรแนะนำาแม่ว่า
จะให้เด็กไป รร ได้เมื่อไร
1. หลังจากไข้ลด
2. เมื่อแผลตกสะเก็ดหมด
3. เมื่อตุ่มนำ้าใสเม็ดสุดท้าย
ลอก
4. หลังกิน Acyclovir 48 hr
5. ไปได้เลย แต่ต้องสวมเสื้อ
ให้มิดชิด
56. เด็ก 9 ปี มีอาการปวดหัว
ปวดท้องก่อนไป รร ไม่อยากไป
รร กลัวมีเหตุการณ์ลักพาตัว
แม่บอกว่ามีฝันร้ายตอนกลาง
คืน ไม่ไป รร 3 wk ถาม Dx
1. Generalized anxiety
2. Separation anxiety
3. Simple phobia
57. เด็ก หายใจหอบเหนื่อย มี
กลิ่นเหม็น hyperpnea, +ve
urine ketone 4+ Dx อะไร
1. DKA
2. Sepsis
3. meningitis
58. ผู้ป่วยเด็กชายอายุ 9 mo
มาด้วย otitis media เคยเป็น
โรคติดเชื้อเป็นๆหายๆ ตั้งแต่ 3
mo ก่อน พวก pneumonia,
S.pneumo, Hib, ถามว่าน่ามี
defect ที่ใด
1. B cell
2. T cell
3. Phagocytic defect
4. Complement
59. เด็ก 4 yr ไข้สูง 1 d มีผื่น
แดงตามตัว วันนี้ซึมลงจึงมา
รพ. PE: BP 90/60, RR 40,
PR 120, stiff neck +ve,
Purpura??? ถาม Dx
1. Salmonella
2. N. meningitides
3. Listeria monocytogenes
4. S. pneumonia
5. H. influenza
60. เด็ก 10 mo มีไข้ ไอ นำ้า
มุก มา 3 mo วันนี้ไอมาก
หายใจหอบเหนื่อย มี
inspiratory stridor, chest
drawning ภายหลังให้
epinephrine มีหอบน้อยลง
epinephrine มีผลอย่างไร
1. ขยายหลอดลม
2. ขับเสมหะ
3. ขับนำ้าที่ค้างในปอด
4. ลดบวมเยื่อบุทางเดิน
หายใจ
61. มารดาเป็น DM เด็กเกิดมา
มีอาการสั่น มี sign ของ
hypoglycemia DTX 30 Hct 63
ถาม Management
1. Early feeding
2. Bolus 10%D/W 2ml/kg
3. Plasma exchange
62. เด็กชายอายุ 4 mo ซึมลง
WBC 18000 (N 80%), Hct
6.9%, Dx อะไร
1. Subdural hematoma
63. เด็กอายุ 4-5 ขวบ มีนำ้ามูก
ไหลกลิ่นเหม็น รักษาด้วย
Amoxy 2 วัน อาการไม่ดีขึ้น
มาพบแพทย์ตรวจพบหนองใน
ช่องจมูกมาก ทำาอะไรต่อไป
a. ส่ง X-ray PNS
b. ตรวจด้วย nasal
speculum ซำ้าอีกครั้ง
c. Amoxicillin/Clavulinic
acid
d. ใช้ Amoxycillin เดิมต่อ
ไป
e. Consult โสต ศอ นอ
สิกแพทย์
64. เด็กในข้อใดมีพัฒนาการช้า
a. อายุ 10 เดือน ตั้งไข่
ได้
b. อายุ 12 เดือน เดินได้
c. อายุ 2 ปี เดินก้าวขึ้น
บันได้สลับข้างได้
d. อายุ 3 ปี พูดคำาโดด
ได้ วลี 10-20 คำา
e. อายุ 5 ปี วาดรูป
สามเหลี่ยมได้
65. เด็กหญิง เป็น UTI รักษา
ด้วย Ampicillin ครบ 14 วัน
จากนั้นปวดท้อง ไข้ขึ้น ตัว
เหลือง อุจจาระมี WBC=1-
5,RBC>100 ส่องกล้อง scope ดู
เจอ Patch? ให้ยาอะไร
a. Cotrimoxazole
b. Norfloxacin
c. Metronidazole
66. เด็กผู้ชาย มีอาการเหมือน
Cushing syndrome จะตรวจ
อะไรผิดปกติ
A. Hypocalcemia
B. Hyperkalemia
C. Increased cortisol level
D. Advance bone age
E. Increased renin plasma
activity
67.เด็กหญิงอายุ 4 yr ซีด 2
wk ก่อนหน้านี้ปกติ PE: Pallor,
petichiae บริเวณแขนขา CBC
Hb 7, Hct 22, WBC 3200(N
35, E 3, L 62), Platelet
26000, MCV 90 จะส่ง
Investigation อะไรเพื่อการ
วินิจฉัย
A. Serum ferritin
B. Bleeding time
C. Reticulocyte count
D. Screening coagulogram
E. Bone marrow aspiration
68. เด็กหญิง 9 เดือน กินนม
แล้วสำาลักบ่อย เป็นปอดบวม 3
ครั้ง BW 7 kg PR 120 lung:
wheezing both lung, CXR:
perihilar infiltration, PPD 4
mm หลังได้ brochodilator แล้ว
ดีขึ้น Dx
A. Retian FB
B. Diaphragmatic hernia
C. TB
D. GERD
E. Asthma
69. เด็กมีไข้ มีผื่นตาม
ตัว(Purpura fulminan) Stiff
neck : positive ถามว่าเกิดจาก
เชื้ออะไร
- N.meningitidis
70. เด็ก asthma มีอาการ
เฉพาะเวลาออกกำาลังกายและ
ตอนกลางคืน 2-3 ครั้งต่อเดือน
ถามว่าต้องให้ยาอะไร
A. Oral salbutamol
B. Inhale sulbutamol
C. Inhale steroid
D. Inhale ipratropium +
Salbutamol
71. เด็กอายุ 6 เดือนกระตุก
ตอนเช้าเป็นมา 2 สัปดาห์ โดย
เริ่มจากผงกศีรษะก่อนแล้ว
ทำาท่ามือโอบกอด จง
วินิจฉัย---infantile spasm
72. เด็กชาย อายุ 10 ปี MCA
มาที่ ER ด้วยอาการปวดบวม
และขยับข้อศอกขวาไม่ได้
PE: swelling at Rt elbow,
normal triangle landmark of
Rt elbow, notmal
neurovascular จง Dx
A. Elbow dislocation
B. Displaced olecranon
fracture
C. Displaced supracondylar
fracture
D. Displaced lateral
condylar fracture
E. Displaced medial
supracondylar fracture
73. เด็กชายอายุ 6 ปี แม่พา
มาเนื่องจากอ้วนมี truncal
obesity , Buffalo hump ,
moon face c acne , ตัวเตี้ย
จะตรวจเพิ่มเติมพบอะไร
A. Hypocalcemia
B. Hyperkalemia
C. Advance bone age
D. Increase serum
cortisol
E. Increase activity of
rennin
74. เด็กหญิงอายุ 6 ปี มีจำ้า
เลือดตามแขนขามา 2 สัปดาห์
ตรวจร่างกายพบ petechiae &
ecchymosis at extremities
CBC : Hb 12 Hct 36 WBC
6500 Plt.150,000 ผล Lab ใด
จะผิดปกติ
A. Bleeding time
B. PT
C. PTT
D. Thromin time
E. Venous clotting time
75. เด็กอายุ 2 ปี ผ่าตัด VSD
หลังผ่าคลำา pulse ไม่ได้ วัด
BP ไม่ได้ทันที ได้บีบ AMbu
ผ่าน tube และให้ Amiodalone
แล้วมี EKG เป็น ventricular
tachycardia จง management
A. Defibilation
B. Ca gluconate
C. NaHCO3
D. Adenosine
E. Direct – current
synchronized
cardioversion
76. เด็กชายอายุ 19 ปี มาพบ
แพทย์หลังจากไปถอนฟั นกับ
ทันตแพทย์ แล้วเลือดไหลซึม
ไม่หยุด ตรวจตับและม้ามไม่
โต ตรวจเลือด PTT 50 sec
(control 30 sec), PT 13 sec
(control 14 sec), no
prolonged bleeding time
สาเหตุของการเกิดโรคนี้คือ
อะไร
A. Lupus
coagulopathy
B. Factor VII
deficiency
C. Factor VIII
deficiency
D. von Willebrand
factor deficiency
E. Hereditary platelet
dysfunction
77. เด็ก 3 ปี เหนื่อยง่าย มี
mild pale other WNL หนัก
13.5 kg , blood smear :
hypochromic microcytic
anemia
A. Iron deficiency
Surgery
1. ผู้ป่ วยชายไทยอายุ 47 ปี มี
ก้อนยุบๆ บวมๆ ที่ขาหนีบ
เป็นๆ หายๆ มา 3 ปี ครั้งนี้
มาพบแพทย์ที่โรงพยาบาล ผู้
ป่ วยมาด้วย ก้อนบวมและโต
มากขึ้น กดเจ็บมากและดัน
ก้ อ น ก ลั บ ไ ม่ ไ ด้ ตั้ ง แ ต่ 5
ชั่วโมงก่อนมาโรงพยาบาล
การปฏิบัติในข้อใดเหมาะสม
ที่สุดในผู้ป่วยรายนี้
a. Analgesic
b. Try reducing
c. Observe and plan
admit
d. Surgery
e. Reassure and plan
discharge
2. ผู้ป่ วยชายไทยมีอาการปวด
ท้องที่บริเวณ epigastrium
ทันทีขณะที่พึ่งดื่มเหล้าไปได้
2 แก้ว มาพบแพทย์ที่โรง
พ ย า บ า ล แ พ ท ย์ เ ว ร ไ ด้
ทำา การตรวจร่างกายพบ
Abdomen : generalized
guarding, absent bowel
sound การส่งตรวจในข้อใด
เหมาะสมที่สุดในผู้ป่วยรายนี้
a. Plain abdomen
b. CT abdomen
c. Ultrasound
d. Abdomen series
e. MRI
3. ผู้ป่ วยหญิงอายุ 22 ปี คลำา
ได้ก้อนที่เต้านมขวา 3 เดือน
ก่อน ตรวจร่างกายพบก้อน
ข น า ด 2 cm oval shape,
firm, smooth and movable
การวินิจฉัยในข้อใดเป็นไปได้
มากที่สุด
a. Fat necrosis
b. Fibroadenoma
c. Fibrocystic disease
d. Intraductal carcinoma
e. Intraductal papilloma
4. ผู้ป่ วยชาย 56 ปี ประวัติ
ครอบครัวพี่ช ายเป็ นโรค
มะเร็งลำาไส้ใหญ่แล้วเสียชีวิต
จึ ง ม า ต ร ว จ ร่ า ง ก า ย ทำา
colonoscopy พบ polyps at
descending colon แพทย์ทำา
polypectomy การตรวจพบ
ในข้อใดมีโอกาสพัฒนาเป็ น
มะเร็งมากสุด
a. Villous adenoma
b. Tubulovillous adenoma
c. Tubulous adenoma
5. ผู้ ป่ ว ย ช า ย เ ป็ น hernia ที่
testis ด้ า น ข ว า โ ต ม า 1
ชั่วโมงก่อนมาโรงพยาบาล
การปฏิบัติในข้อใดเหมาะสม
ที่สุดในผู้ป่วยรายนี้
a. Reduction
b. Ultrasound
c. Surgery
d. Radiation
e. Reassure
6. ผู้ ป่ ว ย ช า ย อ า ยุ 30 ปี มี
อาการเจ็บและปวดบริเวณถุง
อัณฑะด้านซ้ายมา 5 วัน
ก่อนมาโรงพยาบาล การ
ตรวจร่างกายเบื้องต้นพบว่า
มีอาการปวด บวม แดง
อาการเจ็บลดลงเมื่อยกถุง
อัณฑะขึ้น แพทย์เจ้าของไข้
ส่ ง ต ร ว จ Ultrasound พ บ
increase vascular blood
flow การวินิจฉัยในข้อใด
เป็นไปได้มากที่สุด
a. Orchitis
b. Epididymitis
c. Incarcerated hernia
d. Acute torsion testis
e. Torsion of appendix
testis
7. ผู้ป่วยชายไทยเคยเป็นโรคตับ
อ่อนอักเสบ รักษาจนหาย
ดีแล้ว แต่ยังมีอาการจุกแน่น
ท้อง ท้องอืด เบื่ออาหาร นำ้า
ห นั ก ล ด ต ร ว จ พ บ
pseudocyst at the front of
pancreas ตั้งแต่ 4 สัปดาห์
ก่อน ก้อนโตขึ้นอย่างรวดเร็ว
การปฏิบัติในข้อใดเหมาะสม
ที่สุดในผู้ป่วยรายนี้
a. Partial Pancreatectomy
b. Total Pancreatectomy
c. Excision of pseudocyst
d. Internal drainage
e. Percutaneous drainage
8. ผู้ ป่ ว ย ช า ย โ ด น หิ น เ จี ย ร
กระเด็นทิ่มคอขณะกำา ลัง
ทำางาน ตำาแหน่งที่โดนใกล้
ลูกกระเดือก 30 นาทีก่อน
มาโรงพยาบาล ช่วงแรกที่
บาดแผลมีเลือดออกตาม
จังหวะการเต้นของหัวใจ, วัด
ค ว า ม ดั น โ ล หิ ต ไ ด้ 86/70
mmHg ขณะนี้เลือดหยุดไหล
แล้ว มีแผล laceration 3 cm
anterior to
sternocleidomastoid ก า ร
ปฏิบัติในข้อใดเหมาะสมที่สุด
ในผู้ป่วยรายนี้
a. Explore wound at ER
b. Explore wound at OR
c. CT
d. MRI
e. Angiography
9. ช า ย อ า ยุ 30 ปี ป ร ะ ส บ
อุ บั ติ เ ห ตุ ร ถ ช น ต้ อ ง ทำา
distal below knee
amputation แ พ ท ย์ พั น
stump ไว้ หลังจากนั้นแพทย์
ควรจะนัดผู้ป่ วยมาเพื่อใส่ขา
เทียมแบบถาวร นานเท่าไร
a. 7 วัน
b. 10 วัน
c. 15 วัน
d. 30 วัน
e. 45 วัน
10. หญิงอายุ 16 ปี มีก้อนที่
เต้านมด้านขวา ก้อนขนาด
ป ร ะ ม า ณ 2 cm. ป ว ด
สัมพันธ์กับรอบเดือน แพทย์
ตรวจร่างกายพบว่ามี well
define 2.5 centimeter
mass, firm consistency at
the right breast การปฏิบัติ
ในข้อใดเหมาะสมที่สุดในผู้
ป่วยรายนี้
a. ให้ตรวจ เต้าน มด้วย
ตนเอง
b. มาพบแพทย์ให้ตรวจเต้า
นม
c. Fine needle aspiration
d. Ultrasound หลังจากนั้น
นัดตรวจติดตามทุกๆ 6
เดือน
e. Mammogram หลังจาก
นั้นนัดตรวจติดตามทุกๆ
12 เดือน
11. ผู้ป่ วยชายอายุ 35 ปี ดื่ม
เ ห ล้ า ติ ด ต่ อ กั น 5 วั น มี
อาการปวดท้องมากมา 2
วัน ปวดร้าวไปหลัง ผู้ป่วยงอ
ตัวแล้วอาการปวดท้องดีขึ้น
แพทย์ทำาการตรวจร่างกาย
Abdomen - tender at
epigastrium, liver dullness,
decreased bowel sound
การวินิจฉัยในข้อใดเป็นไปได้
มากที่สุด
a. Alcoholic gastritis
b. Bowel perforation
c. Acute pancreatitis
d. Spontaneous Bacterial
Peritonitis
e. Cholecystitis
12. ผู้ป่ วยช าย อายุ 35 ปี
ถ่ า ย อุ จ จ า ร ะ มี เ ลื อ ด ป น
เป็นๆหายๆมา 6 เดือน โดย
3 วันก่อนมา รพ. คลำาก้อน
ได้เท่านิ้วหัวแม่มือ เอานิ้วดัน
กลับเข้าไปได้ การปฏิบัติใน
ข้อใดเหมาะสมที่สุดในผู้ป่ วย
รายนี้
a. Injected 5% phenol
b. Rubber band ligation
c. Hemorrhoidectomy
d. Cryosurgery
e. Reassure and follow
up
13. ผู้ป่วยชายอายุ 65 ปี ได้รับ
ก า ร วิ นิ จ ฉั ย ว่ า เ ป็ น
Colorectal Cancer เ มื่ อ 1
เดือนก่อน ครั้งนี้ผู้ป่วยมาโรง
พยาบาลด้วยอาการปวดท้อง
รุนแรงมา 3 ชั่วโมงก่อนมา
โรงพยาบาล แพทย์ทำาการ
ต ร ว จ ร่ า ง ก า ย พ บ ว่ า มี
restless, body temperature
= 39
O
C , Blood Pressure
=100/70 mmHg ,Pulse
Rate =110/min ,
Respiratory Rate = 26/min
generalized abdominal
tenderness, guarding,
rebound tenderness,
absent bowel sound ผ ล
จ า ก ก า ร ส่ ง Acute
abdominal series พ บ ว่ า มี
generalized bowel dilatation
with intraabdominal free air
แพทย์เจ้าของไข้ควรให้ยา
ปฏิชีวนะในข้อใดต่อไปนี้
อะไรก่อนทำาการส่งต่อผู้ป่วย
a. Ceftriazone +
Metronidazole
b. Clindamycin +
Gentamicin
c. Clindamycin
+Ceftazidime
d. Ampicillin + Gentamicin
e. Ceftazidime +
Gentamicin
14. ผู้ป่ วยชายอายุ 65 ปี มี
ป ร ะ วั ติ Car accident มี
อาการอ่อนแรงตั้งแต่คอลง
ไป มี Blood pressure drop,
Bradycardia การวินิจฉัยใน
ข้อใดเป็นไปได้มากที่สุด
a. Cardiogenic shock
b. Cushing response
c. Hypovolemic shock
d. Neurogenic shock
e. Septic shock
15. ผู้ป่ วยชายอายุ 35 ปี ได้
รับบาดเจ็บจากอุบัติเหตุ
เครื่องบินตก มีอาการปวดที่
กระดูกซี่โครงด้านขวาแพทย์
เวรประจำาห้องฉุกเฉินทำาการ
ต ร ว จ ร่ า ง ก า ย พ บ ว่ า มี
trachea shift ไปด้านซ้าย ,
Right pneumothorax (ใ ห้
ข้ อ มู ล เ ป็ น แ บ บ tension
pneumothorax) การปฏิบัติ
ในข้อใดเหมาะสมที่สุดในผู้
ป่วยรายนี้
a. Pleural tapping
b. Observe vital sign
c. Oxygen mask with bag
d. Emergency ORIF
16. ผู้ป่ วยชายอายุ 31 ปี ได้
รับบาดเจ็บจากการถูกแทงที่
left parasternal border 5
นาทีก่อนมาโรงพยาบาล มี
Blood Pressure drop แพทย์
เวรประจำาห้องฉุกเฉินทำาการ
ต ร ว จ ร่ า ง ก า ย พ บ ว่ า มี
distant heart sound ก า ร
ปฏิบัติในข้อใดเหมาะสมที่สุด
ในผู้ป่วยรายนี้
a. Echocardiogram
b. Pericardiocentesis
c. CT chest
d. MRI chest
e. Observe vital sign and
clinical sign and
symptom
17. ผู้ป่ วยชายอายุ 13 ปี ได้
รับบาดเจ็บจากอุบัติเหตุมอ
เตอร์ไซด์ล้มแพทย์เวรประจำา
ห้องฉุกเฉินทำา การตรวจ
ร่ า ง ก า ย พ บ ว่ า มี stupor
และได้ทำาการตรวจโดย CT
scan ผ ล พ บ ว่ า มี cresent
shape hemorrhage แ ล ะ
hyperdensity area ก า ร
วินิจฉัยในข้อใดเป็นไปได้มาก
ที่สุด
a. acute epidural
hemorrhage
b. acute subdural
hemorrhage
c. subacute epidural
hemorrhage
d. subacute subdural
hemorrhage
e. subacute intradural
hemorrhage
18. ผู้ป่ วยชายอายุ 14 ปี ได้
รับบาดเจ็บจากอุบัติเหตุมอ
เตอร์ไซด์ล้มแพทย์เวรประจำา
ห้องฉุกเฉินทำา การตรวจ
ร่ า ง ก า ย พ บ ว่ า Blood
pressure drop ป ร ะ เ มิ น
Glasgow Coma Score = 8
การปฏิบัติในข้อใดเหมาะสม
ที่สุดในผู้ป่วยรายนี้
a. Cervical Collar
b. ET intubations
c. CT and MRI
d. IV fluid and dopamine
if necessary
e. Refer to the nearest
hospial immediately
19. ผู้ป่ วยหญิงปั สสาวะไม่
ออก ต้องสวนปัสสาวะไป 2
ครั้งเมื่อวันที่ผ่านมา วันนี้ผู้
ป่ วยมาด้วยอาการขาอ่อน
แรงทั้งสองข้าง และมีอาการ
ชาถึงระดับสะดือ แพทย์เวร
ประจำาทำาการตรวจร่างกาย
พ บ ว่ า Loose sphincter
tone, areflexia both legs ผู้
ป่ วยน่าจะมีพยาธิสภาพถึง
ระดับใด
a. Parasagittal
b. Basal pons
c. Thoracic cord
d. Lumbosacral nerve
root
e. Conus medullaris
20. ผู้ป่ วยชายอายุ 14 ปี นำ้า
หนักเท่าไรก็ไม่รู้ไม่มีใครจด
ออกมาเลย burn 1
st
degree
burn = 4% ที่บริเวณหน้า,
2
nd
degree burn= 27% เมื่อ
15 นาทีก่อน ไม่มี inhalation
injury ต้ อ ง ใ ห้ ส า ร นำ้ า ใ น
ชั่ ว โ ม ง แ ร ก เ ท่ า ไ ร ต า ม
Parkland’s formula
a. 200 ml/hr
b. 250 ml/hr
c. 290 ml/hr
d. 340 ml/hr
e. 390 ml/hr
21. ผู้ ป่ ว ย ช า ย อ า ยุ 52 ปี
ปวดหน้าอกล่างซ้ายทันที
หลังอาเจียนรุนแรง แพทย์
เวรประจำาห้องฉุกเฉินทำาการ
ตรวจร่างกายพบว่า Leftt
lung decrease breath
sound, Abdomen: tender
with guarding การวินิจฉัย
ในข้อใดเป็นไปได้มากที่สุด
a. Hiatal hernia
b. Esophageal rupture
c. Tension pneumothorax
d. Mallory-Weiss
syndrome
e. Deep neck infection
22. ผู้ป่วยชายอายุ 65 ปี U/D
AAA มีจำ้าเลือดแขนขา PE:
ecchymosis ที่ขาแล ะแขน
CBC Hb: 10, Hct: 30,
WBC 8600 (N 75, L 25),
PT 15, PTT 33, fibrinogen
150 ส่ง lab การส่งตรวจใน
ข้อใดเหมาะสมที่สุดในผู้ป่ วย
รายนี้
a. D-dimer
b. Bleeding time
c. Platelet function test
d. Protein C, Protein S
e. Fibrinogen level
23. ชายอายุ 24 ปี ตาขวา
โ ป น Intracranial bruise,
chemosis ตาขวา, 2 ปี ก่อน
มี severe head injury ได้รับ
การรักษาที่โรงพยาบาล การ
วินิจฉัยในข้อใดเป็นไปได้มาก
ที่สุด
a. Intraorbital abscsee
b. Carotid cavernous
fistula
c. Chronic subdural
hematoma
d. Posttraumatic
hydrocephalus
e. Ruptured opthalmic
aneurysm
24. ชายอายุ 18 ปี ถูกหมา
กัดหน้าโรงภาพยนตร์ที่นิ้ว
มื อ ข ว า ข น า ด 2x1 cm
(โจทย์ไม่ได้บอกลักษ ณะ
บาดแผล) หลังจากนั้นล้าง
แผลและได้ tetanus toxoid
การปฏิบัติในข้อใดเหมาะสม
ที่สุดในผู้ป่วยรายนี้
a. Rabies vaccine
b. Rabies Ig
c. เ ฝ้ า ดู อ า ก า ร สุ นั ข 1
สัปดาห์
d. ตัดหัวสุนัขส่งตรวจและ
รอฟังผล
e. Rabies vaccine แ ล ะ
Rabies Ig
25. ชาย 70 ปี ปัสสาวะบ่อย
ปั สสาวะไม่สุด ปั สสาวะ
นานกว่าปกติ ปั สสาวะไม่
พุ่ง และมีปัสสาวะหยดหลัง
จ า ก ปั ส ส ว ะ เ ส ร็ จ ต ร ว จ
Rectal examination : firm ,
slightly enlarged and firm
prostate gland ,U.A. :
normal การวินิจฉัยในข้อใด
เป็นไปได้มากที่สุด
a. Urethral stricture
b. Acute prostatitis
c. Prostate cancer
d. Neurogenic bladder
e. BPH
สอบ-ศรว-มีนาคม-2551
สอบ-ศรว-มีนาคม-2551
สอบ-ศรว-มีนาคม-2551
สอบ-ศรว-มีนาคม-2551
สอบ-ศรว-มีนาคม-2551
สอบ-ศรว-มีนาคม-2551
สอบ-ศรว-มีนาคม-2551
สอบ-ศรว-มีนาคม-2551
สอบ-ศรว-มีนาคม-2551
สอบ-ศรว-มีนาคม-2551
สอบ-ศรว-มีนาคม-2551
สอบ-ศรว-มีนาคม-2551
สอบ-ศรว-มีนาคม-2551
สอบ-ศรว-มีนาคม-2551
สอบ-ศรว-มีนาคม-2551
สอบ-ศรว-มีนาคม-2551
สอบ-ศรว-มีนาคม-2551
สอบ-ศรว-มีนาคม-2551
สอบ-ศรว-มีนาคม-2551
สอบ-ศรว-มีนาคม-2551
สอบ-ศรว-มีนาคม-2551
สอบ-ศรว-มีนาคม-2551
สอบ-ศรว-มีนาคม-2551
สอบ-ศรว-มีนาคม-2551
สอบ-ศรว-มีนาคม-2551
สอบ-ศรว-มีนาคม-2551
สอบ-ศรว-มีนาคม-2551
สอบ-ศรว-มีนาคม-2551
สอบ-ศรว-มีนาคม-2551
สอบ-ศรว-มีนาคม-2551
สอบ-ศรว-มีนาคม-2551
สอบ-ศรว-มีนาคม-2551

More Related Content

What's hot

มะเร็งกล่องเสียงคอหอย
มะเร็งกล่องเสียงคอหอย มะเร็งกล่องเสียงคอหอย
มะเร็งกล่องเสียงคอหอย Sutthinee Sudchai
 
Urinary (นำเสนอข้อมูลจาก CD) Physiology by Jiradet
Urinary (นำเสนอข้อมูลจาก CD) Physiology by JiradetUrinary (นำเสนอข้อมูลจาก CD) Physiology by Jiradet
Urinary (นำเสนอข้อมูลจาก CD) Physiology by JiradetJiradet Dongroong
 
เฉลยแบบฝึกหัดหน่วยที่ 5
เฉลยแบบฝึกหัดหน่วยที่ 5เฉลยแบบฝึกหัดหน่วยที่ 5
เฉลยแบบฝึกหัดหน่วยที่ 5Thanawut Rattanadon
 
วิทยาศาตร์พื้นฐาน ม1เทอม1
วิทยาศาตร์พื้นฐาน ม1เทอม1วิทยาศาตร์พื้นฐาน ม1เทอม1
วิทยาศาตร์พื้นฐาน ม1เทอม1dnavaroj
 
การประเมินสมรรถภาพในเชิงปฏิบัติ การช่วยเหลือตนเองในกิจวัตรประจาวันขั้นพื้นฐาน...
การประเมินสมรรถภาพในเชิงปฏิบัติ การช่วยเหลือตนเองในกิจวัตรประจาวันขั้นพื้นฐาน...การประเมินสมรรถภาพในเชิงปฏิบัติ การช่วยเหลือตนเองในกิจวัตรประจาวันขั้นพื้นฐาน...
การประเมินสมรรถภาพในเชิงปฏิบัติ การช่วยเหลือตนเองในกิจวัตรประจาวันขั้นพื้นฐาน...Dr.Suradet Chawadet
 
teedanai,+{$userGroup},+14-บทความวิจัย-JMLD..pdf
teedanai,+{$userGroup},+14-บทความวิจัย-JMLD..pdfteedanai,+{$userGroup},+14-บทความวิจัย-JMLD..pdf
teedanai,+{$userGroup},+14-บทความวิจัย-JMLD..pdfssuserf1346a
 
แบบทดสอบ องค์ประกอบของพยางค์และคำ
แบบทดสอบ องค์ประกอบของพยางค์และคำแบบทดสอบ องค์ประกอบของพยางค์และคำ
แบบทดสอบ องค์ประกอบของพยางค์และคำPiyarerk Bunkoson
 
ข้อสอบโอลิมปิก ม.ต้น (Ijso) ปี พ.ศ.2550
ข้อสอบโอลิมปิก ม.ต้น (Ijso) ปี พ.ศ.2550ข้อสอบโอลิมปิก ม.ต้น (Ijso) ปี พ.ศ.2550
ข้อสอบโอลิมปิก ม.ต้น (Ijso) ปี พ.ศ.2550sawed kodnara
 
ชีววิทยาเรื่องระบบภูมิคุ้มกัน Immune system
ชีววิทยาเรื่องระบบภูมิคุ้มกัน Immune systemชีววิทยาเรื่องระบบภูมิคุ้มกัน Immune system
ชีววิทยาเรื่องระบบภูมิคุ้มกัน Immune systemkasidid20309
 
การประเมินนวัตกรรมสื่อออนไลน์อาชีวศึกษา
การประเมินนวัตกรรมสื่อออนไลน์อาชีวศึกษาการประเมินนวัตกรรมสื่อออนไลน์อาชีวศึกษา
การประเมินนวัตกรรมสื่อออนไลน์อาชีวศึกษาPrachyanun Nilsook
 
ข้อสอบวิชาช่าง
ข้อสอบวิชาช่างข้อสอบวิชาช่าง
ข้อสอบวิชาช่างkrupeak
 
ใบงานพอลิเมอร์
ใบงานพอลิเมอร์ใบงานพอลิเมอร์
ใบงานพอลิเมอร์Jariya Jaiyot
 
ข้อสอบ เคมี
ข้อสอบ เคมีข้อสอบ เคมี
ข้อสอบ เคมีzweetiiz
 
การพยาบาลผู้ป่วยที่มีอุปกรณ์ติดตัว
การพยาบาลผู้ป่วยที่มีอุปกรณ์ติดตัว การพยาบาลผู้ป่วยที่มีอุปกรณ์ติดตัว
การพยาบาลผู้ป่วยที่มีอุปกรณ์ติดตัว techno UCH
 
เล่ม 5 ปฏิกิริยาเคมีในชีวิตประจำวัน
เล่ม 5 ปฏิกิริยาเคมีในชีวิตประจำวันเล่ม 5 ปฏิกิริยาเคมีในชีวิตประจำวัน
เล่ม 5 ปฏิกิริยาเคมีในชีวิตประจำวันPreeyapat Lengrabam
 

What's hot (20)

มะเร็งกล่องเสียงคอหอย
มะเร็งกล่องเสียงคอหอย มะเร็งกล่องเสียงคอหอย
มะเร็งกล่องเสียงคอหอย
 
Urinary (นำเสนอข้อมูลจาก CD) Physiology by Jiradet
Urinary (นำเสนอข้อมูลจาก CD) Physiology by JiradetUrinary (นำเสนอข้อมูลจาก CD) Physiology by Jiradet
Urinary (นำเสนอข้อมูลจาก CD) Physiology by Jiradet
 
Transportation body
Transportation bodyTransportation body
Transportation body
 
5แบบทดสอบส่วนประกอบของเซลล์
5แบบทดสอบส่วนประกอบของเซลล์5แบบทดสอบส่วนประกอบของเซลล์
5แบบทดสอบส่วนประกอบของเซลล์
 
เฉลยแบบฝึกหัดหน่วยที่ 5
เฉลยแบบฝึกหัดหน่วยที่ 5เฉลยแบบฝึกหัดหน่วยที่ 5
เฉลยแบบฝึกหัดหน่วยที่ 5
 
วิทยาศาตร์พื้นฐาน ม1เทอม1
วิทยาศาตร์พื้นฐาน ม1เทอม1วิทยาศาตร์พื้นฐาน ม1เทอม1
วิทยาศาตร์พื้นฐาน ม1เทอม1
 
การประเมินสมรรถภาพในเชิงปฏิบัติ การช่วยเหลือตนเองในกิจวัตรประจาวันขั้นพื้นฐาน...
การประเมินสมรรถภาพในเชิงปฏิบัติ การช่วยเหลือตนเองในกิจวัตรประจาวันขั้นพื้นฐาน...การประเมินสมรรถภาพในเชิงปฏิบัติ การช่วยเหลือตนเองในกิจวัตรประจาวันขั้นพื้นฐาน...
การประเมินสมรรถภาพในเชิงปฏิบัติ การช่วยเหลือตนเองในกิจวัตรประจาวันขั้นพื้นฐาน...
 
teedanai,+{$userGroup},+14-บทความวิจัย-JMLD..pdf
teedanai,+{$userGroup},+14-บทความวิจัย-JMLD..pdfteedanai,+{$userGroup},+14-บทความวิจัย-JMLD..pdf
teedanai,+{$userGroup},+14-บทความวิจัย-JMLD..pdf
 
แผนBioม.4 1
แผนBioม.4 1แผนBioม.4 1
แผนBioม.4 1
 
แบบทดสอบ องค์ประกอบของพยางค์และคำ
แบบทดสอบ องค์ประกอบของพยางค์และคำแบบทดสอบ องค์ประกอบของพยางค์และคำ
แบบทดสอบ องค์ประกอบของพยางค์และคำ
 
ข้อสอบโอลิมปิก ม.ต้น (Ijso) ปี พ.ศ.2550
ข้อสอบโอลิมปิก ม.ต้น (Ijso) ปี พ.ศ.2550ข้อสอบโอลิมปิก ม.ต้น (Ijso) ปี พ.ศ.2550
ข้อสอบโอลิมปิก ม.ต้น (Ijso) ปี พ.ศ.2550
 
ชีววิทยาเรื่องระบบภูมิคุ้มกัน Immune system
ชีววิทยาเรื่องระบบภูมิคุ้มกัน Immune systemชีววิทยาเรื่องระบบภูมิคุ้มกัน Immune system
ชีววิทยาเรื่องระบบภูมิคุ้มกัน Immune system
 
การประเมินนวัตกรรมสื่อออนไลน์อาชีวศึกษา
การประเมินนวัตกรรมสื่อออนไลน์อาชีวศึกษาการประเมินนวัตกรรมสื่อออนไลน์อาชีวศึกษา
การประเมินนวัตกรรมสื่อออนไลน์อาชีวศึกษา
 
Example osce
Example osceExample osce
Example osce
 
ข้อสอบวิชาช่าง
ข้อสอบวิชาช่างข้อสอบวิชาช่าง
ข้อสอบวิชาช่าง
 
10ยูเนียน
10ยูเนียน10ยูเนียน
10ยูเนียน
 
ใบงานพอลิเมอร์
ใบงานพอลิเมอร์ใบงานพอลิเมอร์
ใบงานพอลิเมอร์
 
ข้อสอบ เคมี
ข้อสอบ เคมีข้อสอบ เคมี
ข้อสอบ เคมี
 
การพยาบาลผู้ป่วยที่มีอุปกรณ์ติดตัว
การพยาบาลผู้ป่วยที่มีอุปกรณ์ติดตัว การพยาบาลผู้ป่วยที่มีอุปกรณ์ติดตัว
การพยาบาลผู้ป่วยที่มีอุปกรณ์ติดตัว
 
เล่ม 5 ปฏิกิริยาเคมีในชีวิตประจำวัน
เล่ม 5 ปฏิกิริยาเคมีในชีวิตประจำวันเล่ม 5 ปฏิกิริยาเคมีในชีวิตประจำวัน
เล่ม 5 ปฏิกิริยาเคมีในชีวิตประจำวัน
 

Viewers also liked

Biochemistry tutorial
Biochemistry tutorialBiochemistry tutorial
Biochemistry tutorialrookiess
 
Nl 2010 nctms
Nl 2010 nctmsNl 2010 nctms
Nl 2010 nctmsNew Srsn
 
Comprehensive 2008 kku
Comprehensive 2008 kkuComprehensive 2008 kku
Comprehensive 2008 kkurookiess
 
ศรว 51 ANS By Cmu
ศรว 51 ANS By Cmuศรว 51 ANS By Cmu
ศรว 51 ANS By Cmuvora kun
 
152010010020
152010010020152010010020
152010010020Aaesah
 
9789740329213
97897403292139789740329213
9789740329213CUPress
 
แนวทางเวชปฏิบัติในการรักษาผู้ป่วยมาลาเรียในประเทศไทย พ.ศ.2557
แนวทางเวชปฏิบัติในการรักษาผู้ป่วยมาลาเรียในประเทศไทย พ.ศ.2557แนวทางเวชปฏิบัติในการรักษาผู้ป่วยมาลาเรียในประเทศไทย พ.ศ.2557
แนวทางเวชปฏิบัติในการรักษาผู้ป่วยมาลาเรียในประเทศไทย พ.ศ.2557Utai Sukviwatsirikul
 
แนวทางการวินิจฉัยและรักษาโรคไข้เลือดออก ระดับ รพ.ชุมชน 2004
แนวทางการวินิจฉัยและรักษาโรคไข้เลือดออก ระดับ รพ.ชุมชน 2004แนวทางการวินิจฉัยและรักษาโรคไข้เลือดออก ระดับ รพ.ชุมชน 2004
แนวทางการวินิจฉัยและรักษาโรคไข้เลือดออก ระดับ รพ.ชุมชน 2004Utai Sukviwatsirikul
 
DIP_ oświetlenie Gmina Miedźna
DIP_ oświetlenie Gmina MiedźnaDIP_ oświetlenie Gmina Miedźna
DIP_ oświetlenie Gmina Miedźnamarcingermanek
 
Skin dispensing1
Skin dispensing1Skin dispensing1
Skin dispensing1Or Chid
 
Leadership skills part one attributes attitude and mindset
Leadership skills part one attributes attitude and mindsetLeadership skills part one attributes attitude and mindset
Leadership skills part one attributes attitude and mindsetPeter Chen
 
แนวทางการดูแลเท้าในผู้ป่วยเบาหวาน
แนวทางการดูแลเท้าในผู้ป่วยเบาหวานแนวทางการดูแลเท้าในผู้ป่วยเบาหวาน
แนวทางการดูแลเท้าในผู้ป่วยเบาหวานUtai Sukviwatsirikul
 
2007821172158 466 6438_1
2007821172158 466 6438_12007821172158 466 6438_1
2007821172158 466 6438_1New Srsn
 
Skin part 2
Skin part 2Skin part 2
Skin part 2Or Chid
 
บทที่ 3 ยาสามัญประจำบ้าน
บทที่ 3 ยาสามัญประจำบ้านบทที่ 3 ยาสามัญประจำบ้าน
บทที่ 3 ยาสามัญประจำบ้านPa'rig Prig
 

Viewers also liked (20)

Biochemistry tutorial
Biochemistry tutorialBiochemistry tutorial
Biochemistry tutorial
 
Nl 2010 nctms
Nl 2010 nctmsNl 2010 nctms
Nl 2010 nctms
 
Comprehensive 2008 kku
Comprehensive 2008 kkuComprehensive 2008 kku
Comprehensive 2008 kku
 
ศรว 51 ANS By Cmu
ศรว 51 ANS By Cmuศรว 51 ANS By Cmu
ศรว 51 ANS By Cmu
 
รวมข้อสอบCompre nl
รวมข้อสอบCompre nlรวมข้อสอบCompre nl
รวมข้อสอบCompre nl
 
152010010020
152010010020152010010020
152010010020
 
9789740329213
97897403292139789740329213
9789740329213
 
แนวทางเวชปฏิบัติในการรักษาผู้ป่วยมาลาเรียในประเทศไทย พ.ศ.2557
แนวทางเวชปฏิบัติในการรักษาผู้ป่วยมาลาเรียในประเทศไทย พ.ศ.2557แนวทางเวชปฏิบัติในการรักษาผู้ป่วยมาลาเรียในประเทศไทย พ.ศ.2557
แนวทางเวชปฏิบัติในการรักษาผู้ป่วยมาลาเรียในประเทศไทย พ.ศ.2557
 
แนวทางการวินิจฉัยและรักษาโรคไข้เลือดออก ระดับ รพ.ชุมชน 2004
แนวทางการวินิจฉัยและรักษาโรคไข้เลือดออก ระดับ รพ.ชุมชน 2004แนวทางการวินิจฉัยและรักษาโรคไข้เลือดออก ระดับ รพ.ชุมชน 2004
แนวทางการวินิจฉัยและรักษาโรคไข้เลือดออก ระดับ รพ.ชุมชน 2004
 
Waddell & reed operation breakthrough-2055
Waddell & reed  operation breakthrough-2055Waddell & reed  operation breakthrough-2055
Waddell & reed operation breakthrough-2055
 
DIP_ oświetlenie Gmina Miedźna
DIP_ oświetlenie Gmina MiedźnaDIP_ oświetlenie Gmina Miedźna
DIP_ oświetlenie Gmina Miedźna
 
Skin dispensing1
Skin dispensing1Skin dispensing1
Skin dispensing1
 
National licensce ii 2554 print meng
National licensce ii 2554 print mengNational licensce ii 2554 print meng
National licensce ii 2554 print meng
 
Leadership skills part one attributes attitude and mindset
Leadership skills part one attributes attitude and mindsetLeadership skills part one attributes attitude and mindset
Leadership skills part one attributes attitude and mindset
 
แนวทางการดูแลเท้าในผู้ป่วยเบาหวาน
แนวทางการดูแลเท้าในผู้ป่วยเบาหวานแนวทางการดูแลเท้าในผู้ป่วยเบาหวาน
แนวทางการดูแลเท้าในผู้ป่วยเบาหวาน
 
2010_PMC Musculoskeleton
2010_PMC Musculoskeleton2010_PMC Musculoskeleton
2010_PMC Musculoskeleton
 
2007821172158 466 6438_1
2007821172158 466 6438_12007821172158 466 6438_1
2007821172158 466 6438_1
 
National test _2553_TU
National test _2553_TUNational test _2553_TU
National test _2553_TU
 
Skin part 2
Skin part 2Skin part 2
Skin part 2
 
บทที่ 3 ยาสามัญประจำบ้าน
บทที่ 3 ยาสามัญประจำบ้านบทที่ 3 ยาสามัญประจำบ้าน
บทที่ 3 ยาสามัญประจำบ้าน
 

Similar to สอบ-ศรว-มีนาคม-2551

แนวทางการดูแลรักษาลิ้มเลือดอุดตันในปอด ในผูปวยฉุกเฉิน
แนวทางการดูแลรักษาลิ้มเลือดอุดตันในปอด ในผูปวยฉุกเฉินแนวทางการดูแลรักษาลิ้มเลือดอุดตันในปอด ในผูปวยฉุกเฉิน
แนวทางการดูแลรักษาลิ้มเลือดอุดตันในปอด ในผูปวยฉุกเฉินSociety of Thai Emergency Physicians
 
Interhospital chest conference
Interhospital chest conferenceInterhospital chest conference
Interhospital chest conferenceMy Parents
 
Compre si 2010 l
Compre si 2010 lCompre si 2010 l
Compre si 2010 lvora kun
 
Case study surgery
Case study surgeryCase study surgery
Case study surgerysoftmail
 
Exercise national license_part_ii_march_2009_2
Exercise national license_part_ii_march_2009_2Exercise national license_part_ii_march_2009_2
Exercise national license_part_ii_march_2009_2Loveis1able Khumpuangdee
 
interesting case
interesting  caseinteresting  case
interesting caseSHAMONBEST1
 
Surgery of acquired heart disease อ.วรวงศ์ ศลิษฏ์อรรถกร
Surgery of acquired heart disease อ.วรวงศ์ ศลิษฏ์อรรถกรSurgery of acquired heart disease อ.วรวงศ์ ศลิษฏ์อรรถกร
Surgery of acquired heart disease อ.วรวงศ์ ศลิษฏ์อรรถกรpohgreen
 
Case-study-เพชรกร-อริศรา.docx
Case-study-เพชรกร-อริศรา.docxCase-study-เพชรกร-อริศรา.docx
Case-study-เพชรกร-อริศรา.docxERppk
 
CPG Thai Stroke infarct retrieved since 2555
CPG Thai Stroke infarct retrieved since 2555CPG Thai Stroke infarct retrieved since 2555
CPG Thai Stroke infarct retrieved since 2555Thorsang Chayovan
 
Common nutritional problems in pediatrics
Common nutritional problems in pediatricsCommon nutritional problems in pediatrics
Common nutritional problems in pediatricsPitiphong Sangsomrit
 
Nle step 2_2009 si115-116 and nle_step_2_2009 nctms editors cut key
Nle step 2_2009 si115-116 and nle_step_2_2009 nctms editors cut keyNle step 2_2009 si115-116 and nle_step_2_2009 nctms editors cut key
Nle step 2_2009 si115-116 and nle_step_2_2009 nctms editors cut keyLoveis1able Khumpuangdee
 
Nt2009 Complete Ans
Nt2009 Complete AnsNt2009 Complete Ans
Nt2009 Complete Ansvora kun
 

Similar to สอบ-ศรว-มีนาคม-2551 (20)

แนวทางการดูแลรักษาลิ้มเลือดอุดตันในปอด ในผูปวยฉุกเฉิน
แนวทางการดูแลรักษาลิ้มเลือดอุดตันในปอด ในผูปวยฉุกเฉินแนวทางการดูแลรักษาลิ้มเลือดอุดตันในปอด ในผูปวยฉุกเฉิน
แนวทางการดูแลรักษาลิ้มเลือดอุดตันในปอด ในผูปวยฉุกเฉิน
 
Interhospital chest conference
Interhospital chest conferenceInterhospital chest conference
Interhospital chest conference
 
Compre si 2010 l
Compre si 2010 lCompre si 2010 l
Compre si 2010 l
 
For extern
For externFor extern
For extern
 
National license 2010 by med tu 16
National license 2010 by med tu 16National license 2010 by med tu 16
National license 2010 by med tu 16
 
National license 2010 by med tu 16
National license 2010 by med tu 16National license 2010 by med tu 16
National license 2010 by med tu 16
 
Case study surgery
Case study surgeryCase study surgery
Case study surgery
 
Exercise national license_part_ii_march_2009_2
Exercise national license_part_ii_march_2009_2Exercise national license_part_ii_march_2009_2
Exercise national license_part_ii_march_2009_2
 
Nl part ii march 2009
Nl part ii march 2009Nl part ii march 2009
Nl part ii march 2009
 
Nl part ii march 2009
Nl part ii march 2009Nl part ii march 2009
Nl part ii march 2009
 
Spinal injury
Spinal injurySpinal injury
Spinal injury
 
interesting case
interesting  caseinteresting  case
interesting case
 
Surgery of acquired heart disease อ.วรวงศ์ ศลิษฏ์อรรถกร
Surgery of acquired heart disease อ.วรวงศ์ ศลิษฏ์อรรถกรSurgery of acquired heart disease อ.วรวงศ์ ศลิษฏ์อรรถกร
Surgery of acquired heart disease อ.วรวงศ์ ศลิษฏ์อรรถกร
 
Case-study-เพชรกร-อริศรา.docx
Case-study-เพชรกร-อริศรา.docxCase-study-เพชรกร-อริศรา.docx
Case-study-เพชรกร-อริศรา.docx
 
CPG Thai Stroke infarct retrieved since 2555
CPG Thai Stroke infarct retrieved since 2555CPG Thai Stroke infarct retrieved since 2555
CPG Thai Stroke infarct retrieved since 2555
 
Pediatric pneumonia Thai
Pediatric pneumonia ThaiPediatric pneumonia Thai
Pediatric pneumonia Thai
 
Common nutritional problems in pediatrics
Common nutritional problems in pediatricsCommon nutritional problems in pediatrics
Common nutritional problems in pediatrics
 
X ray con
X ray conX ray con
X ray con
 
Nle step 2_2009 si115-116 and nle_step_2_2009 nctms editors cut key
Nle step 2_2009 si115-116 and nle_step_2_2009 nctms editors cut keyNle step 2_2009 si115-116 and nle_step_2_2009 nctms editors cut key
Nle step 2_2009 si115-116 and nle_step_2_2009 nctms editors cut key
 
Nt2009 Complete Ans
Nt2009 Complete AnsNt2009 Complete Ans
Nt2009 Complete Ans
 

สอบ-ศรว-มีนาคม-2551

  • 1. ข้อสอบศูนย์ประเมิน ความรู้ความสามารถ ของผู้ประกอบวิชาชีพ เวชกรรม (ศรว.) ขั้นตอนที่ 2 ครั้งที่ 1 วันที่ 9 มีนาคม 2551 สวัสดีครับ เพื่อนๆ พี่ๆ น้องๆ ข้อสอบ ศรว. ฉบับนี้เป็นฉบับ update ล่าสุด เมื่อเดือน กันยายน 2551 โดยเป็นข้อสอบ จากพี่ๆ มหาวิทยาลัยต่างๆ 4 สถาบันคือ ขอนแก่น ธรรมศาสตร์ เชียงใหม่ และ จุฬานะครับ ข้อสอบฉบับนี้ จะไม่มีโอกาส สำาเร็จได้เลย ถ้าปราศจาก แรงงานและแรงใจของพี่ๆ คณะต่างๆ ที่รวบรวมข้อสอบ เอาไว้ จึงขอเอ่ยนามขอบคุณไว้ ที่นี้ด้วยครับ ขอขอบคุณ - พี่กิ๊ก, ก้อย, ปุย, สา, นูดเดิล, แบงค์ MD 31 KKU - พี่ๆ มช. ที่ช่วยๆกันจำา ข้อสอบ - พี่ๆ Med 14 THAMMASAT University - พี่ๆ จุฬาโดยเฉพาะเฮีย peck และเจ๊กุ๊ก สุดท้ายนี้ ขอขอบคุณเพื่อนๆ ทุกคนไม่ว่าจะเป็นจุฬา วราลี กรวิกและเพื่อนๆคนอื่นที่สละ แรงกายมารวบรวมข้อสอบใน ครั้งนี้ด้วยครับ ป.ล. การรวบรวมข้อสอบครั้งนี้ อาจไม่ใช่ข้อสอบชุดที่สมบูรณ์ ที่สุดและยังเป็น version แรก เท่านั้น ถ้าคณะใดรวบรวม ข้อสอบเสร็จแล้วก็รบกวนส่งมา อีกนะครับ ยินดีจัดทำาให้เสมอ ครับ ถึงแม้ข้อสอบชุดนี้จะ คลอดออกมาช้าไปหน่อย แต่ก็ น่าจะทันอยู่นะ ^_^
  • 2. Medicine 1. ผู้ป่ วยหญิงไทยอายุ 32 ปี ปวดข้อเป็นๆ หายๆ และ มีผื่นขึ้นที่หน้า ผลการ ตรวจทางห้องปฏิบัติการ พบ Urine protein 1+, red blood cell cast ก า ร ส่ ง ตรวจเพิ่มเติมในข้อใดต่อ ไปนี้ specific สำาหรับการ วินิจฉัยในผู้ป่ วยรายนี้มาก ที่สุด a. Rheumatoid factor b. ANA c. Anti-histone antibody d. Anti ds-DNA antibody 2. ผู้ป่วยหญิงอายุ 35 ปี มีไข้ ตำ่าๆ ปวดข้อเข่าและข้อ มือมา 4 วันก่อนมาโรง พยาบาล มาพบแพทย์ เจาะนำ้าข้อเข่าตรวจพบ wbc = 89000, PMN = 90%, G/S: gram negative diplococci intracellular การรักษาข้อใดต่อไปนี้ เหมาะสมที่สุด a. Cloxacillin b. Gantamicin c. Ceftriazone d. Doxycycline e. Metronidazole 3. ผู้ป่ วยชายวัยกลางคน ชัก แ บ บ Generalized Tonic Clonic ป ฏิ เ ส ธ ป ร ะ วั ติ อุบัติเหตุหรือการใช้สาร เสพย์ติด ผู้ป่ วยไม่เคยมี อาการชักแบบนี้มาก่อน ไ ม่มีไ ข้ จากการต ร วจ ร่างกายเบื้องต้นไม่พบ neurological deficit ใ ด ๆ แ ต่ พ บ ว่ า มี papilledema ผ ล CT scan: multiple calcified cystic lesion เชื้อก่อโรคในข้อใดต่อไปนี้ น่าจะทำาให้เกิดอาการใน ชายผู้นี้มากที่สุด a. Taenia solium
  • 3. b. Taenia saginata c. Toxocara d. Toxoplasma=HIVinf ect มาด้วยชัก e. Gnathostoma 4. ผู้ป่ วยชายอายุ 55 ปี ไอ แห้งๆ บางครั้งมีเลือดปน ออกมาด้วย นำ้าหนักตัวลด ลง 4 กิโลกรัมใน 1 เดือน ผลการตรวจ AFB ได้ผล negative 3 ครั้งติดต่อกัน ผ ล CXR พ บ ว่ า มี reticulonodular infiltration with thickening cavity การปฏิบัติในข้อใดต่อไปนี้ น่าจะเหมาะสมกับผู้ป่ วย รายนี้มากที่สุด a. ร อ ผ ล culture for TB b. Start anti-TB drug <diag tb จ า ก film แ ล ะ clinical> c. Bronchoscopy d. Aspiration e. PCR for TB 5. ผู้ป่ วยชายอายุ 18 ปี ไป เยี่ยมสถานรับเลี้ยงเด็กมา เมื่อ 2 สัปดาห์ก่อนมาโรง พยาบาล ต่อมามีอาการ อ่อนแรงของแขนขา เป็ น ม า ก ขึ้น เ รื่อ ย ๆ แ พ ท ย์ ทำาการตรวจร่างกายพบว่า มี Upper motor power 4/5, lower 1/5, absent DTR ข้อใดคือการวินิจฉัย ในผู้ป่ วยรายนี้ที่เหมาะสม ที่สุด a. Transverse myelitis b. GBS c. MG d. Neurosyphilis 6. ผู้ชายอายุ 30 ปี เหนื่อย มา 1 เดือน แพทย์ทำาการ ต ร ว จ ร่ า ง ก า ย พ บ ว่ า มี neck vein engorgement, Hepatomegaly ฟั ง ป อ ด ปกติ ผลจากการตรวจ EKG พบว่ามี diffuse ST- T change, low voltage ข้อใดคือการวินิจฉัยในผู้ ป่วยรายนี้ที่เหมาะสมที่สุด
  • 4. a. Restrictive pericarditis แ ต่ เ ป ลี่ ย น เ ป็ น constrictive b. Beri beri c. Cardiac tamponade d. Dilated cardiomyopathy 7. เด็กชายอายุ 9 ขวบ มา โรงพยาบาล ด้ว ยไ ข้สูง หายใจหอบเหนื่อย ไอ แ พ ท ย์ ทำา ก า ร ต ร ว จ ร่างกายพบว่ามี BT= 39.0 o C, PR= 120 /min, RR = 40 /min, crepitation both lungs, decreased breath sound left lung ผู้ ป่ ว ย มี แผลตุ่มหนองและ cellulitis ที่ขาซ้าย เชื้อก่อโรคในข้อ ใดต่อไปนี้น่าจะทำาให้เกิด อาการในชายผู้นี้มากที่สุด a. Salmonella spp. b. S. aureus c. H. influenzae d. Mycoplasma pneumonia e. Mycoplasma tuberculosis 8. ผู้ชายอายุ 50 ปี กินเหล้า มานาน 5 วัน มาด้วยปวด ท้อ ง ลิ้น ปี่ ร้าวไปหลัง นอนงอตัวดีขึ้น แพทย์ ทำาการตรวจร่างกายพบ ว่ า มี tenderness and guarding at epigastrium, liver dullness +ve ข้อใด คือการวินิจฉัยในผู้ป่ วย รายนี้ที่เหมาะสมที่สุด a. acute pancreatitis 9. มี rash เป็ นแบบ vesicle ตามแนวซี่โครง มีแสบๆ เ จ็ บ ๆ ข้ อ ใ ด คื อ ก า ร วินิจฉัยในผู้ป่ วยรายนี้ที่ เหมาะสมที่สุด a. VZV 10. ผู้ป่ วยหญิงอายุ 40 ปี มี อาการปวดบวมต้นขาซ้าย และมีไข้ แพทย์ทำา การ ตรวจร่างกายพบว่ามี BT
  • 5. = 39 o C, erythema and tender at left upper thigh เจาะแล้วได้หนอง เชื้อก่อ โรคในข้อใดต่อไปนี้น่าจะ ทำาให้เกิดอาการใน ผู้ป่ วย รายนี้มากที่สุด a. S. aureus b. S. pyogenes เพราะได้หนอง c. H. influenzae d. P. aeruginosa 11. ผู้ป่ วยชายไทย สูงอายุ ประวัติโรคประจำาตัวเดิม เ ป็ น DM, HT ครั้งนี้ม า ด้วยอาการชาและอ่อน แ ร ง 1 ข้ า ง น า น 10 นาที จากนั้น หายเป็ น ปกติ ข้อใดคือการวินิจฉัย ในผู้ป่ วยรายนี้ที่เหมาะสม ที่สุด a. TIA b. Temporal lobe ischemia c. Cerebella ischemia d. Bell’s palsy 12. ผู้ป่ วยชายอายุ 18 ปี ก่อน หน้านี้สุขภาพร่างกายแข็ง แรงดีไม่มีโรคประจำา ตัว ค รั้ ง นี้มี อ า ก า ร Upper respiratory tract infection ผู้ป่ วยมาด้วยไข้ ไอ หอบ ผลจากการตรวจ CXR พ บ ว่ า มี Interstitial infiltration ยาปฏิชีวนะใด เหมาะสมที่สุดในผู้ป่ วย รายนี้ a. Ampicillin b. Ciprofloxacin c. Roxithromycin d. Gentamicin e. Augmentin Atypicalpneumonia=Inter stitial infiltration Typical จะเป็น alveolar 13. ใ น ก า ร รั บ น้ อ ง ที่ มหาวิทยาลัยแห่งหนึ่ง รุ่น พี่ได้ให้รุ่นน้องให้กินนำ้า เปล่าปริมาณมาก หลัง จากนั้นผู้ป่ วยมีอาการซึม ปวดศีรษะและ คลื่นไส้
  • 6. อาเจียน ข้อใดต่อไปนี้น่า จะทำาให้เกิดอาการใน ผู้ ป่ วยรายนี้มากที่สุด (หลัง จากนำา ข้อมูลจากแต่ละ สถาบันมารวมกันแล้ว พบ ว่ามีจำานวนตัวเลือกเกิน 5 ตัวเลือก) a. Hypokalemia b. Hyperkalemia c. Hyponatremia d. Hypernatremia e. Hypocalcemia f. Acute gastric dilation g. SIADH 14. สารพิษที่พบในสีทาบ้าน ทำา ใ ห้ เ กิ ด อ า ก า ร peripheral neuropathy (wrist drop) แ ละ ซี ด คื อ อะไร-ตะกั่ว 15. ผู้ ป่ ว ย CKD มี K = 7 EKG change จงบอกการ รักษาที่สำา คัญที่สุดในผู้ ป่ วยรายนี้ Cacium gluconate Beta 2 agonist 16. ผู้ป่วยชายอายุ 40 ปี มีไข้ และปวดเมื่อยตามเนื้อตัว มา 3 วัน ผู้ป่ วยให้ประวัติ ว่าที่ละแวกบ้านมีนำ้ าำ และผู้ป่ วยต้องลุยนำ้าออก จากบ้านทุกวัน แพทย์ ทำาการตรวจร่างกายพบว่า มี BP = 120/80 mmHg, PR = 110/min, RR = 24 /min, BT = 39 o C, mild jaundice, ling clear, liver 2 cm BRCM ข้ อ ใ ด คื อ การวินิจฉัยในผู้ป่ วยรายนี้ ที่เหมาะสมที่สุด a. Enteric fever b. Dengue fever c. Leptospirosis d. Scrub typhus e. Murine typhus 17. ผู้ป่ วยหญิงอายุ 50 ปี มา ด้วยอาการปวดแขนขา
  • 7. ขึ้นบันไดลำาบาก ลุกจาก เก้าอี้ลำา บาก 4 สัปดาห์ ก่ อ น ม า โ ร ง พ ย า บ า ล แ พ ท ย์ ทำา ก า ร ต ร ว จ ร่ า ง ก า ย พ บ ว่ า มี tenderness of quadriceps muscle, purple-red discoloration over the upper forehead, eyelids and cheeks ข้อใดคือการ วินิจฉัยในผู้ป่ วยรายนี้ที่ เหมาะสมที่สุด a. Polymyositis b. Dermatomyositis— proximal mus weak c. Psoriasis d. Mixed connective tissue disease e. SLE 18. ผู้ป่ วยชายอายุ 40 ปี มี อาการไอแห้งๆ มา 1 เดือนก่อนมาโรงพยาบาล อ า ก า ร เ ป็ น ม า ก ต อ น อากาศหนาว และตอน เย็น ไอมีเสมหะสีขาว ไม่มี เลือดปน ตรวจร่างกาย ปกติ ผลการตรวจ CXR พบว่า WNL การปฏิบัติใน ข้อใดต่อไปนี้น่าจะเหมาะ สมกับผู้ป่วยรายนี้มากที่สุด a. Skin test b. CT chest c. Sputum AFB d. Bronchoscopy e. PFT 19. ผู้ป่ วยชายอายุ 50 ปี กิน เหล้าขาวมานาน 25 ปี ครั้งนี้มีอาการเหนื่อย ขา บวม 1 สัปดาห์ก่อนมา โ ร ง พ ย า บ า ล แ พ ท ย์ ทำาการตรวจร่างกายพบว่า มี BP = 120/70 mmHg, fine crepitation both lower lungs, PMI 6 th ICS 1 cm lateral to MCL, S3 gallop ก า ร รักษาที่เหมาะสมที่สุดในผู้ ป่วยรายนี้คือข้อใด a. Digoxin
  • 8. b. Thiamine<cardiac beriberi> c. Atropine 20. ผู้ป่ วยชายหลังจากกลับ จากนำ้าตก มีไข้และแผล ดังแสดงในรูป เชื้อก่อ โรคในข้อใดต่อไปนี้น่าจะ ทำาให้เกิดอาการใน ผู้ป่ วย รายนี้มากที่สุด a. Chigger mite b. Louse c. Aedes egypti d. Tick e. Flea 21. ผู้ป่ วยชายไทย มาโรง พยาบาลด้วยอาการชาที่ บ ริ เ ว ณ ต้ น ค อ แ พ ท ย์ ทำาการตรวจร่างกายพบว่า มี BP 60/30 mmHg ไม่มี sacral reflex ข้อใดคือการ วินิจฉัยในผู้ป่ วยรายนี้ที่ เหมาะสมที่สุด a. Cardiogenic shock b. Spinal shock c. Neurogenic shock d. Hypovolemic shock e. Obstructive shock 22. วัยรุ่นชายอายุ 18 ปี แข็ง แ ร ง ดี มี อ า ก า ร ห อ บ หายใจลำาบาก ตอนออก กำาลังกาย และตอนกลาง คืน มีอาการ 2-3 ครั้งใน เ ดื อ น ที่ผ่ า น ม า แ พ ท ย์ ทำาการตรวจร่างกายเบื้อง ต้นพบว่าไม่มีความผิดปรก ติใดๆ การรักษาที่เหมาะ สมที่สุดในผู้ป่ วยรายนี้คือ ข้อใด a. inhale salbutamol prn + inhale corticosteroid b. alpha-agonist as need c. steroid ตลอด d. beta-agonist as need + steroid ตลอด e. Oral bronchodilator f. Salbutamol inhalation as needed
  • 9. g. Long acting beta2 agonist + corticosteroid h. Inhaled corticosteroid 23. Motor Cycle Accident มี lesion ที่ T2 ค า ส า ย Foley Catheter มีอาการ แน่นหน้าอก เหงื่อออก มากขึ้น BP 180/110 RR เพิ่ม ควรทำาอย่างไร a. off F/C b. Hydralazine c. ลุกนั่งปลดเสื้อผ้า d. นอนหัวตำ่าวัด BP ซำ้า e. ADVICE 24. ห ญิ ง ไ ท ย 70 ปี no Underlying Disease สบายดีมาตลอด มีอาการ แขนซ้ายอ่อนแรง 5 นาที EKG เป็น AF ใช้ยาอะไร ป้ องกันอาการอ่อนแรง a. ASA b. Warfarin c. Clopidogrel 25. ผู้หญิง อายุ .PE: HT + Cushing appearance Lab เจออะไร a. Increase cortisol level 26. ผู้ ห ญิ ง subacute thyroiditis ให้ยาใดเพื่อลด อาการ a. ASA+propanplol 27. T4 ปกติ TSH ปกติ Dx? a. Simple Goiter b. Iodine def c. Thyroid CA d. Subacute thyroiditis e. Hashimoto Thyroiditis 28. ผู้หญิงอายุ 28 ปี ฉี่ไม่ อ อ ก ต้ อ ง ใ ส่ ส า ย ส ว น ปั ส ส า วะ 2 ค รั้ง ต่อ มา ขาอ่อนแรง ชาถึงระดับ ส ะ ดื อ PE : DTR 3+ , loss of sphincter tone ผู้ ป่วยมี lesion อยู่ที่ไหน a. parasagital sinus b. Basal pons c. Thoracic area
  • 10. d. Lumbosacral area e. Clonus medularis 29. ผู้ ป่ ว ย ช า ย ไ ท ย เ ป็ น bronchial asthma พ่ น ย า ไ ม่ ดี ขึ้ น lung : expirat Wheezing ก า ร รั ก ษ า ที่ เหมาะสมที่สุดในผู้ป่ วย รายนี้คือข้อใด a. iv. Dexametazone 30. EKG : bradycardia ต่ อ มาตรวจ vital sign พบว่า ผู้ป่ วยมีความดันโลหิตตก (80/50 mmHg, HR = 30) การรักษาที่เหมาะสมที่สุด ในผู้ป่วยรายนี้คือข้อใด a. Atropine b. dobutamine c. pacemaker 31. ผู้ ป่ ว ย ห ญิ ง อ า ยุ 27 ปี เจ็บคอมา 2 สัปดาห์ PE: white patch เต็มคอและ ลิ้น ถามว่าจะส่งตรวจ อะไรเพื่อให้ได้เหตุผลของ โรคที่ดีที่สุด a. anti-HIV 32. ผู้ป่ วยชาย ให้ EKG อ่าน เอง (เป็ น Mobitz II) จะ ให้ยาอะไร a. Atropine b. Lidociane c. Digitalis d. Cardioversion 33. ผู้ ป่ ว ย ห ญิ ง อ า ยุ 68 ปี แ พ ท ย์ ทำา ก า ร ต ร ว จ ร่างกายเบื้องต้นทุกอย่าง ปกติดี จะให้วัคซีนใด a. Rabies vaccine b. Tetanus viccine c. HBV vaccine d. Influenza vaccine e. HPV vaccine 34. ผู้ป่ วยหญิงอายุ 18 ปี มี อาการเจ็บหน้าอก โน้มตัว ไปข้างหน้าอาการดีขึ้น PE: lung clear, มี เ สี ย ง rub ที่ chest คิดว่าเป็ น อะไร a. Endocarditis b. Pancarditis c. Myocarditis d. Pneumonia
  • 11. 35. ผู้ป่วยชายอายุ 16 ปี เป็น homozygous Beta-thal ต้องรับเลือดเป็นประจำา มี อาการอ่อนเพลียและบวม มา 1 เดือน แพทย์ทำาการ ตรวจร่างกายเบื้องต้น BP drop, irregular heart rate & rhythm, hepatosplenomegaly ถาม ว่ า ผู้ ป่ ว ย มี heart complication จากอะไร a. cardiac siderosis b. Cardiac hypertrophy c. Immune cardiomyositis d. Extramedullary hematopoiesis at heart 36. ผู้ ป่ ว ย ช า ย อ า ยุ 20 ปี Motorcycle Accident ทำา CT พ บ crescent shape at frontal region ในผู้ป่ วย รายนี้น่าจะคิดถึงโรคใด มากที่สุด a. Acute epidural hemorrhage b. Acute subdural hemorrhage c. Subacute epidural hemorrhage 37. clinical rheumatoid arthritis ยาใดลดปวดเร็ว สุด a. Ibuprofen b. Paracetamol c. MTX d. Sulfazalazine e. Chloroquine 38. ผู้ป่ วยสูงอายุมีโรคประจำา ตั ว เ ป็ น DM with HT มี แ พ ท ย์ ทำา ก า ร ต ร ว จ ร่ า ง ก า ย เ บื้ อ ง ต้ น Costovertebral angle tenderness ก าร รัก ษ า ที่ เหมาะสมที่สุดในผู้ป่ วย รายนี้คือข้อใด a. Ceftriaxone IV b. Ceftriaxone กิน c. Ofloxacin กิน
  • 12. 39. ชายอายุ 22 ปี หมดสติ ในห้องนำ้า ไม่มีไข้ ตรวจ ร่างกายพบ PR ตำ่า RR ตำ่า miosis การรักษาที่ เหมาะสมที่สุดในผู้ป่ วย รายนี้คือข้อใด a. Atropine b. Naloxone c. Diazepam 40. ผู้ป่ วยหญิง 40 ปี หนัก 80 kg สูง 160 cm. เพิ่ง Dx. DM, FBG 180 ก า ร รักษาที่เหมาะสมที่สุดในผู้ ป่วยรายนี้คือข้อใด a. ควบคุมอาหาร b. Metformin c. Acarbose d. Glibenclamide 41. ผู้ป่ วยหญิง 25 ปี ใจสั่น กินจุ นน.ลด มา 1 ปี , PE: mild exophthalmos, generalized enlarged thyroid both lobe, …. (Graves’), T4..,T3 400, TSH <0.001 การรักษาที่ เหมาะสมที่สุดในผู้ป่ วย รายนี้คือข้อใด a. Propanolol b. Lugol solution c. PTU d. Radioactive iodine e. Subtotal thyroidectomy 42. ชาย 32 ปี มีอาการใจสั่น นำ้าหนักลด BP 150/100 mmHg PR 120/min PE: neuro sensory normal, muscle weak gr. 1/5 both upper and lower, reflex 2+ all Na 136 mmol/l K 2.5 mmol/l Cl 102 mmol/l CO2 22 mmol/l ต้องส่งตรวจอะไรเพื่อ เป็นการหาสาเหตุของอาการ อ่อนแรง a. Free T4 b. Plasma glucose c. CT brain d. CSF exam
  • 13. 43. ผู้ป่ วยชายมีอาการใจสั่น มี muscle weakness ตรวจ DTR 1+, กล้ามเนื้อ แขนขาอ่อนแรง 1+, ลุก ไม่ขึ้น K 2.5 ถามว่าตรวจ อะไรเพิ่มเติม a. Free T4 b. 44. หญิง 15 ปี มี webbed neck ไม่มีเต้านม ข้อใดถูก a. Insulin ตำ่า b. GH ตำ่า c. Estrogen สูง d. Gonadrotropin สูง e. Parathyroid hormone สูง 45. ชาย 30 ปี ไอมา 1 เดือน เสมหะขาวมีเลือด ปนบางครั้ง ไข้ตำ่าๆ นำ้า หนักลด 3 kg, PE: normal, CXR: Reticulo- patchy infiltration with thick wall cavity, AFB sputum: neg X 3 ครั้ง ควรทำาอะไรต่อ? a. Bronchoscopy b. FNA c. Anti-TB drug d. รอผล culture for TB 46. ชายไทย 30 ปี อาชีพขับ รถ bus มีไข้ตำ่า ไอแห้ง นำ้าหนักตัวลดลง, ตรวจ พบ AFB 3+, แพทย์ให้ยา TB ควรเขียนให้ลาหยุด งานนานเท่าใด a. 2 wk. b. 1 mo. c. 2 mo. d. ทีละ 1 wk จนกว่า AFB จะ neg e. ไม่จำาเป็นต้องหยุด งาน 47. ช า ย อ า ยุ 40 ปี โ ร ค ป ร ะ จำา ตั ว เ ป็ น Asthma เป็ นหวัด 2 วัน PTA พ่น ย า แ ล้ ว อ า ก า ร ไ ม่ ดี ขึ้ น แ พ ท ย์ ทำา ก า ร ต ร ว จ ร่างกายเบื้องต้น T 37.4 o C BP 110/90 mmHg PR 100 bpm RR 28 /min
  • 14. Lung: wheezing at both lower lung with accessory muscle used การรักษาที่เหมาะสมที่สุด ในผู้ป่วยรายนี้คือข้อใด a. Terbutaline Sc b. IV steroid c. IV dexamethasone d. IV Theophyline e. NB salbutamol 48. ผู้ ป่ ว ย ห ญิ ง ไ ท ย มี hyperkeratosis onycholysis การรักษาที่ เหมาะสมที่สุดในผู้ป่ วย รายนี้คือข้อใด a. Topical cotrimoxazole b. Topical itraconazole c. Topical ketokonazole d. Oral itraconazole 49. ผู้ป่วยหญิง มาด้วยอาการ ผื่น มีสะเก็ด เป็นทั่วศีรษะ ข้อเข่า ข้อศอก เกาแล้วมี ผื่นขึ้น แกะสะเก็ดออกมี เลือดออกเล็กน้อย ในผู้ ป่วยรายนี้น่าจะคิดถึงโรค ใดมากที่สุด a. Psoriasis b. Lichen planus c. Discoid lupus erythematosus 50. Pt 50 yr มาด้วย แพทย์ ทำาการตรวจร่างกายเบื้อง ต้นตัวเหลืองตาเหลือง 1 wk, Cholesterol 350, TG 250 อาจจะพบอาการ แสดงทางผิวหนังอะไร a. Eczema b. Lipoma c. Xanthelasma d. Seborrheic dermatitis 51. หญิง 15 ปี มาด้วย อาการปวดศีรษะข้างเดียว มา 4 เดือน มีอาการ เดือนละ 2-3 ครั้ง จงให้ Prophylaxis a. Propanolol b. Sumatriptan คือ ทันที acute
  • 15. c. Flunazine d. A,B e. A,C 52. ผู้ป่วยชาย 30 yr ปวด ศีรษะมากทันที เวียนหัว คลื่นไส้อาเจียน drowsiness, BP 150/110, PR 80, RR 22, stiffness of neck, Babinski’s sign - neg, Retinal a. - absence of pulse, no papilledema ในผู้ป่วยราย นี้น่าจะคิดถึงโรคใดมาก ที่สุด a. Migraine b. Viral encephalitis c. Subarachnoid hemorrhage d. Bacterial meningitis 53. หญิง 35 yr หนังตาตกมา 4 wk เป็นมากขึ้นตอ นบ่ายๆ ตรวจตาซ้ายพบ ptosis มี mild proximal muscle weakness, normal DTR, normal sensory จง Dx a. Polymyositis b. Horner’s syndrome c. Hyperthyroidism d. Myasthenia gravis e. CN III palsy 54. ผู้ป่วยชายอายุ 21 yr อ่อนแรงขาทั้ง 2 ข้าง ต่อ มาอ่อนแรงไปทั้งตัว มา 3 วัน, Total ophthalmoplegia, bilateral facial palsy, paradoxical breathing, generalized decrease motor power gr II-III/V, areflexia, loss of vibration sense all toe and fingers ข้อใดคือการ รักษาที่เหมาะสมที่สุด diab GBS a. Azatiopine b. Interferon beta 1A c. Cyclophosphamide d. Methylprednisolone e. Intravenous Immunoglobulin
  • 16. 55. ผู้ป่วยหญิง 70 yr มีแขน ขาซ้ายอ่อนแรง และมี EKG ดังนี้ (มี ST elevate)ให้ยาใดป้ องกัน อาการดังกล่าว a. Aspirin b. Enoxaparin c. Clopidogrel d. Warfarin e. Dipyramidamole 56. Male วัยกลางคน มี อาการปวดบวมเข่าและนิ้ว โป้ งเท้าขวา เจาะข้อได้ WBC 20,000 และ Needle shaped crystal จะเจอ X-ray finding อะไร a. Osteoporosis b. Osteophyte c. Chondrocalcinosis d. Bone ankylosis e. Lumpy soft tissue 57. ผู้ป่วยชายอายุ 50 ปี ปวดเข่ามา 2 วัน มีไข้ ปวดบวมเข่าขวา ขยับข้อ เข่าไม่ได้ เนื่องจากมี อาการปวดมาก PE: BT 38.5 o C, Rt. knee swelling and marked tender. Lab ใดจะมีประโยชน์มากที่สุด a. Serum uric acid b. Rheumatoid factor c. Joint fluid analysis d. ESR e. Knee plain film X- ray 58. ผู้ ป่ ว ย ช า ย อ า ยุ 70 ปี ปวดเข่าขวามา 3 วัน ได้ ย า diclofenac ไ ม่ ดี ขึ้ น เ จ า ะ เ ข่ า ไ ด้ turbid yellow, WBC 20,000 (PMN 50 %, L 20 %) ESR 50 % gram stain negative (X-ray : no joint space narrowing ,sclerotic rim? no fracture, no osteolytic lesion)ในผู้ป่ วยรายนี้น่าจะ คิดถึงโรคใดมากที่สุด a. septic arthritis b. Pseugogout
  • 17. c. Gout d. Rheumatoid arthritis e. Osteoarthritis 59. ชายอายุ 49 ปี มีไข้ ไม่มี หนาวสั่น เจ็บที่ชายโครง ข้างขวาร้าวไปที่ไหล่ขวา 3 วันก่อนมาโรงพยาบาล อุจจาระปกติ ไม่เหลือง มี เบื่ออาหาร นำ้าหนักลด PE: Rt lobe liver enlarged with mild tenderness, HBsAg positive LAB: LFT: TB/DB = 1.0/0.2 AST = 60 ALT = 70 ALP = 112 Total protein = 5.0 Albumin = 2.3 U/S : Hyperechoic mass at right lobe liver diameter 6 cm, no intrahepatic duct dilatation ผลการวินิจฉัยเป็นอะไร a. Metastatic CA b. Amoebic liver abscess c. Hepatocellular carcinoma d. Peripheral Cholangiocarcinom a e. Postnecrotic cirrhosis with hepatic adenoma 60. ชายวัย 52 ปี มีอาการ เจ็บใต้ชายโครงซ้าย เฉียบพลัน หลังจาก อาเจียนมาก PE : profused sweating, decrease breath sound Lt. lung, Abd.: tender with guarding Dx? a. Hiatal hernia b. Ruptured esophagus c. Mallory-Weiss syndrome
  • 18. d. Tension pneumothorax e. Acute MI 61. ชายอายุ 40 ปี ดื่มเหล้า 3-4 ครั้ง/wk มีปวดถ่วงๆ ท้องน้อย อุจจาระมีเลือด ปน แต่เลือดไม่ปนเป็น เนื้อเดียวกับอุจจาระ ข้อ ใดเป็นคำาแนะนำาเพื่อลด อาการถ่ายเป็นเลือด a. เพิ่มอาหารไขมัน b. เพิ่มอาหารโปรตีน สูง c. เพิ่มอาหาร fiber สูง d. ลดเครื่องดื่ม Alcohol e. ฝึกเบ่งถ่ายอุจจาระ ให้ถูกวิธีทุกวัน 62. ผู้ป่วยอายุ 45 ปี ดื่มเหล้า ขาววันละ 1 ขวด มา 25 ปี ถ่ายเหลาวมา 3-4 ครั้ง/d เป็นเวลา 2 วัน มา ด้วยอาการสับสน จำาญาติ ไม่ได้ ตรวจร่างกาย V/S ปกติ พบ scaling lesion บริเวณหน้า คอ และ แขน 2 ข้าง ถาม Dx a. Pellagra B3 def dementia diarrhea dermatitis b. Subdural hematoma c. Wernicke’s encephalopathy d. Electrolyte imbalance e. Delirium tremens 63. ผู้ป่วยชาย อายุ 33 ปี dyspnea, engorged neck vein, no murmur, EKG: low voltage, generalized ST change with inverted T ในผู้ป่วยรายนี้น่าจะ คิดถึงโรคใดมากที่สุด a. Constrictive pericarditis b. Cardiac tamponade c. RBBB d. Lateral wall infarction
  • 19. e. Pulmonary embolism 64. ผู้ป่ วยชายอายุ 18 ปี มี อาการเหนื่อยขึ้นมาทันที ไอเป็นฟองสีชมพู แพทย์ ทำาการตรวจร่างกายตรวจ ร่ า ง ก า ย พ บ ว่ า มี pink frothy sputum, crepitation both lower lungs, BP 160/100 mmHg ไ ด้ ใ ห้ furosemide และ O2 ไป การรักษาเพิ่มเติมที่เหมาะ สมที่สุดในผู้ป่ วยรายนี้คือ ข้อใด a. Digoxin b. Propanolol c. Dobutamine d. Nitropusside e. Morphine f. Mg 65. ผู้ป่วยหญิงอายุ 32 ปี Underlying Rheumatic heart disease with Mitral valve stenosis ขณะนี้ได้ ยา Digoxin(0.25mg) 1 tab OD , Furosemide(40 mg) 1 tab OD โดย 3 วัน ก่อนมา รพ. ผู้ป่วยมี อาการเหนื่อยมาก คลื่นไส้ อาเจียน เบื่ออาหาร เห็น แสงสีผิดปกติ PE: BP 100/70 mmHg, HR 50 bpm, Lung fine crepitation both lung, Heart loud S1, diastolic murmur at apex อะไร เป็นสาเหตุที่ทำาให้เกิด อาการมากขึ้น a. Hypokalemia b. Hyponatremia c. Drug interaction d. Volume depletion e. Metabolic alkalosis 66. ผู้ ป่ ว ย ห ญิ ง อ า ยุ 32 ปี Underlying Rheumatic heart disease with Mitral valve stenosis ขณะนี้ได้ ย า Digoxin(0.25mg) 1 tab OD , Furosemide(40 mg) 1 tab OD โดย 3 วัน ก่ อ น ม า ร พ . ผู้ ป่ ว ย มี อาการเหนื่อยมาก คลื่นไส้
  • 20. อาเจียน เบื่ออาหาร เห็น แ ส ง สี ผิ ด ป ก ติ PE: BP 100/70 mmHg, HR 50 bpm, Lung fine crepitation both lung, Heart loud S1, diastolic murmur at apex ก า ร รักษาที่เหมาะสมที่สุดในผู้ ป่วยรายนี้คือข้อใด a. เพิ่มยา Atropine b. เพิ่มยา beta Blocker c. หยุดทานยา Digoxin d. เพิ่มยา Steroid e. เพิ่มยา Furosemide 67. ผู้หญิงอายุ 60 ปี สูบบุหรี่ มา 30 ปี ไอมาก X-ray พบ Right upper lung mass และมี Na ในเลือด ตำ่า (urine osm 50) เกิด จากอะไร a. SIADH b. Polydipsia 68. patient CHF … Bx: lymphocyte infiltration, most common virus? a. Adenovirus b. Coxsackie B c. EBV d. CMV 69. ชายอายุ 30 ปี นำ้าท่วม บ้าน มาด้วยปวดกล้าม เนื้อ ไข้ Dx? a. Leptospirosis 70.ชาย 65 ปี U/D AAA มีจำ้า เลือดตามตัว PE: multiple ecchymoses, อื่นๆปกติ, CBC: Hb 10, WBC 7,200 (N 75, L 25), Plt 30,000, PT,PTT,TT : prolong, fibrinogen ↓, จะ ตรวจอะไรเป็นประโยชน์ ที่สุดในการบอกสาเหตุ ? a. D-dimer b. Euglobulin lysis time c. Blood smear 71. ลูกคนแรกเป็น HbH จง หา Typing ของแม่และพ่อ
  • 21. Hb MCV พ่อ 13 76 แม่ 13 87 a. พ่อเป็น alpha alpha, - - b. แม่เป็น alpha -, alpha - 72. กินเหล้าเถื่อน แล้วมี อาการอาเจียน หายใจ หอบ Na? K? a. Methanol b. Ethylene glycol c. Salicylate d. Paraquat 73. ผู้ชายอายุ 20 ปี ถูกสุนัข หน้าโรงหนังกัด แผล ขนาด 2 x 1 cm หลัง จากทำาแผลให้ TT แล้ว จะป้ องกัน Rabies ยังไง a. ให้ Rabies vaccine b. ให้ Rabies Ig c. แนะนำาให้สังเกต อาการสุนัขอีก 1 สัปดาห์ d. ตัดหัวสุนัขส่งตรวจ e. ให้ Rabies vaccine และ Rabies Ig 74. ผู้ป่ วยชายหมดสติมาที่ ER ผลการทำา EKG เป็ น ดังรูป (VF)การรักษ าที่ เหมาะสมที่สุดในผู้ป่ วย รายนี้คือข้อใด a. Defibrillation b. Atropine injection c. Adenosine injection d. Adrenaline injection 75. หญิงอายุ 35 ปี มาที่ ER ประวัติว่ากินส้มตำาปู ปลาร้า พริกขี้หนู 10 เม็ด หลังจากนั้น 2 ชั่วโมง มี อาการชารอบปาก น้อง สาวที่ทานด้วยกันก็มี อาการเช่นเดียวกับผู้ป่วย PE: BT 37 o C , BP 140/90, HR 120, RR 14/min with shallow breathing, motor gradeII/V, reflex 1+ ถาม ว่าเกิดจากพิษอะไร a. Saxitoxin b. Tetradotoxin
  • 22. c. Botulinum toxin 76. ผู้ ป่ ว ย ใ น ICU เ ป็ น Subarachnoid Hemorrhage ใส่ ET tube แ ล ะ Foley’s cath 1 wk ต่อมาปัสสาวะมีสีขุ่นย้อม เป็น candida albican การ รักษาที่เหมาะสมที่สุดในผู้ ป่วยรายนี้คือข้อใด a. Itraconazole b. Clotrimazole c. Fluconazole d. Amphoterin B e. ถอดสาย Foley’s cath 77. ทหารเกณฑ์ออกฝึกมี อาการหน้ามืด เป็นลม และมีอาการชักแพทย์ ทำาการตรวจร่างกายเบื้อง ต้น BT 41.1 O C , BP 80/60 mmHg , PR 140 /min RR 25/min ในผู้ ป่วยรายนี้น่าจะคิดถึงโรค ใดมากที่สุด a. Heat stroke b. Subdural hematoma c. Dehydrate d. Encephalitis 78. ผู้ป่วยชาย 75 ปี อัมพาต bed sore ไข้ ซึม ฉี่ไม่ ออก 2 วัน Stupor , mild pale , no Jx , no hepatosplenomegaly , Hb 9 , MCV 98 , WBC 14000 , N90 , L10 , Plt 20000 , PT18 , aPTT 36 , มี PBS เป็น MAHA Blood picture ในผู้ป่วย รายนี้น่าจะคิดถึงโรคใด มากที่สุด a. primary fibrinolysis b. Vit K def c. Thalassemia with sepsis d. DIC e. TTP (MAHA พบได้ในโรคต่อไปนี้ นะครับ TTP, HUS, Malignant HT, Prostatic heart vale, DIC,etc)
  • 23. 79. ผู้ป่ วยชายไทยรายหนึ่ง ตรวจร่างกายประจำาปี มี ผลเลือดดังต่อไปนี้ TG = 210, LDL = 200, HDL = 30 ยาในข้อใดน่าจะเหมาะ สมกับผู้ป่ วยรายนี้มากที่สุด ที่สุด a. Gemfibrozil b. Simvastatin c. Fibric acid d. Niacin e. Cholestyramine f. Fenofibrate 80. ผู้ป่วยชายอายุ 56 ปี มา ด้วยเจ็บแน่นหน้าอกด้าน ซ้าย 2 ชม เป็น DM & HT มา 10 yrs ให้ EKG มาเป็น STEMI , V1-V4 หลังจากให้ ASA ไปแล้ว การรักษาใด เหมาะสม ที่สุด a. Enoxaparin b. Warfarin c. Streptokinase d. Balloon e. PCI 81. ชาย 56 ปี เป็นโรคตับ แข็งมา 2 ปี ถ่ายเหลว 3- 4 ครั้งต่อวัน มา 3 วัน ก่อนมารพ. วันนี้ซึมลง BT= 37.3 PR=98 RR=18 BP104/80 stupor , mild jaundice , mild ascites , spider nevi + palmar erythema จะส่ง Investigation ใดเพื่อช่วย ในการวินิจฉัยมากที่สุด a. CBC b. Stool exam c. Coagulogram d. LFT e. Abdominal paracentesis 82. ผู้ป่วยชายอายุ 42 ปี สูบ บุหรี่ 2 ซองต่อปี มานาน 20 ปี ตรวจพบ cyanosis at finger tips , radial pulse 1+,brachial pulse normal Dx? a. Raynaud’s disease
  • 24. b. Takayasu’s disease c. Polyarteritis nodosa d. Thromboangiitis obliterans e. Systemic lupus erythematosus 83. ผู้ป่วยชายอายุ 60 ปี ปวดบวมแดงร้อนเข่าขวา มา 1 สัปดาห์ มีไข้ ตรวจ เข่าขวาพบ บวม อุ่น มีนำ้า ในข้อ พบ WBC 50,000 , PMN 80%, sugar 22 (plasma glucose 105) ไม่ พบ crystal , ทำา G/S & AFB negative การรักษา ที่ เหมาะสม คือ a. Colchicine b. Ceftriaxone c. Indomethacin d. Intraarticular steroid injection e. Anti-TB drug 84. เด็กหญิง 6 ขวบ มีจำ้า เลือดตามตัวเป็น ๆ หายๆ มา 1 เดือน PE: generalized ecchymosis CBC:Hb 12,Hct 36,WBC=8,500,PMN=55 %,Lym=30%,Eo=15%,Plt= 154,000 with pale stain ตรวจ LAB อะไรผิดปกติ a. PT b. aPTT c. Bleeding time d. VCT e. TT 85. ผู้ป่วยหญิงอายุ 50 ปี ปวดแขนขามา 4 สัปดาห์ นั่งแล้วลุกขึ้นยืนลำาบาก ขึ้นลงบันไดลำาบาก ตรวจ ร่างกายพบ purple-red discoloration ที่หน้าผาก แก้มและเปลือกตา purple nodule ที่ศอกและเข่า คิด ว่าคนนี้เป็นอะไร a. Psoriasis b. Dermatomyositis c. Discoid LE d. SLE
  • 25. e. Mixed connective tissue 86. ชายอายุ 30 ปี ท้องเสีย 3 เดือน Cachexia, muscular wasting รูปไข่ พยาธิคล้ายถั่วลิสง (Capillaria philipinensis) ควรมีวิธีการป้ องกัน อย่างไร a. งดกินปู b. งดกินหอย c. งดกินผักสด d. งดกินปลานำ้าจืด ดิบๆ e. งดเดินเท้าเปล่าตาม ทุ่งนา f. งดดื่มนำ้าที่ยังไม่ได้ ผ่านการต้มสุก 87. หญิงอายุ 42 ปี เดินทาง มาจากเชียงใหม่ มาตรวจ FU มะเร็งเต้านมที่ รพ.กรุงเทพฯ ขณะกำาลัง ลงจากรถทัวร์ มีอาการ เจ็บอกด้ายขวา และ หายใจหอบเหนื่อย PE: PR 128 /min RR 30 /min lung-clear, Rt leg-Swollen & tenderness เข้ากับโรค ใดมากที่สุด a. Pleuodynia b. Pneumonia c. Pulmonary embolism d. Acute coronary syndrome e. Dissected aortic aneurism 88. ชายอายุ 18 ปี ไข้สูง เจ็บ คอ 3 day ซีด เหลือง moderate pale, mild Jaundice, liver 2 cm below costal margin, spleen 3 cm below costal margin, Hb 7.9, Hct 28, MCV 65, WBC 9500, N 80%, L 20%, Plt 450000 ในผู้ป่วยรายนี้ น่าจะคิดถึงโรคใดมาก ที่สุด a. AIHA b. HbH with crisis c. PNH
  • 26. d. G6PD e. DIC 89. ชายอายุ 50 ปี ปวดเข่า ขวามาก มีไข้ P.E. : Rt. Knee swelling , Tender ปวดมาก เดินไม่ได้ ต้อง อะไรต่อ a. X-ray b. Synovial analysis 90. ผู้ป่วย Cirrhosis มีท้อง เสีย ถ่ายเหลว 3-4 ครั้ง ต่อวัน มา 3 วัน มี อาการซึม ตรวจพบ Sign ของ Chronic liver disease และ Moderate ascites ต้อง investigation ใดเพิ่มเพื่อช่วย Dx a. Abdominal paracentesis b. Stool exam c. LFT d. CBC 91. ชาย อายุ … ปี มีไข้เจ็บ คอ P.E. : Tonsil enlargement , greyish patch at pharynx and tonsils gl. , anterior cervical L.N. enlargement both side , ตับม้ามไม่โต ในผู้ป่วยรายนี้น่าจะคิดถึง โรคใดมากที่สุ a. Diptheria 92. ชายอายุ 23 ปี มาด้วยเจ็บ อก หอบเหนื่อย นอนราบ ไม่ได้ Dx : CHF ต่อมา เสียชีวิต ตรวจศพพบ Lymphocyte infiltrate ที่ กล้ามเนื้อหัวใจ ถามหา เชื้อ a. Influenza b. Adeno virus c. Coxakie B d. EBV 93. ผู้ป่วยชายอายุ 45 ปี ปวดหัวมาก 1 วัน โดยมี อาการมาประมาณ 1-2 เดือน ใจสั่นเป็นพักๆ วัน ละ 1-2 ครั้ง เคยไปพบ แพทย์ตรวจพบว่าความดัน 200/120 mmHg ได้ยามา ทานแต่ทานไม่สมำ่าเสมอ ตรวจความดันได้ 210/130
  • 27. mmHg PR 120 /min RR 22/min ตรวจตา Fundus พบ Flame shape hemorrhage with early papilledema นอกจาก Furosemide แล้วจะให้ยา ลดความดัน a. Enalapril b. Propanolol c. Prazosin d. Nitroprusside e. Spironolactone 94. ชาย 45 ปี กินเหล้ามา นาน มาร.พ.ด้วยเลือด ออกใน stomach ขณะนี้ เลือดหยุดแล้ว ตอนนี้มี อาการกระสับกระส่าย กระวนกระวาย มี restlessness , sweating , BP = 150/100 , PR = 120 , Tremor both hands จะ Treatment อย่างไร a. Chlorpromazine IM b. Diazepam IV c. Haloperidol IM d. Thiamine IM 95. ชายอายุ 60 ปี มีตุ่มนำ้า ขึ้นเป็นกลุ่มบริเวณหลังไป ถึงใต้ชายโครง ปวดแสบ ร้อน ถามว่าเกิดจากโรค อะไร a. Cellulitis b. Contact dermatitis c. Dermatitis dermatiformis d. Herpes simplex infection e. Vericella zoster infection 96. ผู้ป่วยมี anaphylaxis shock ต้อง treatment อย่างไร a. IM adrenaline ดี กว่า Subcutaneous adrenaline confirm 97. ผู้ป่วยหญิงอายุ 50 ปี ขณะดูโทรทัศน์ ขณะดู โทรทัศน์ที่โซฟา มีอาการ ปวดท้ายทอย ลุกเดินขึ้น มีเดินเซขวา PE : pupil
  • 28. 1mm แขนขาอ่อนแรง ถามหา lesion a. Pontine hemorrhage b. Cellebellar hemorrhage c. Subarchnoid hemorrhage d. Basal ganglion hemorrhage e. Intraventricular hemorrhage 98. ผู้ป่วยชาย 30 ปี มาด้วย ผื่นขาว ขอบเขตไม่ชัดเจน บริเวณซีกเดียวของ ร่างกายลามช้าๆ ไม่คัน แต่มีอาการชาบริเวณผื่น ในผู้ป่วยรายนี้น่าจะคิดถึง โรคใดมากที่สุด a. Vitiligo b. Pityriasis versicolor c. Tinea corporis d. Leprosy e. Syphilis 99. หญิงอายุ 45 ปี มาด้วย เหนื่อย 2 เดือน ตรวจ ร่างกาย PR เร็ว มี heave, totally irregular มี pitting edema , JVP up to mandible , tapping apex , tapping parasternum, loud P2 (แต่มีอีกสถาบันหนึ่งบอก ว่าเป็น loud S1 ???), opening snap , มี Diastolic rumbling murmur ที่ apex ในผู้ ป่วยรายนี้น่าจะคิดถึงโรค ใดมากที่สุด a. Rheumatic carditis b. Mitral valve prolapsed c. Infective endocarditis d. Mitral annulus calcification e. Rupture chordate tendinae 100. ชายวัยกลางคน มี ไข้ ไอ เหนื่อย ลม หายใจมีกลิ่นเหม็น CXR :cavitary lesion with
  • 29. air fluid level ในผู้ป่วย รายนี้น่าจะคิดถึงโรคใด มากที่สุด a. Lung abscess 101. ผู้ป่ วย CRF ,urine ออกน้อย, BP สูง , K 7 , EKG change การรักษาที่ เหมาะสมที่สุดในผู้ป่ วย รายนี้คือข้อใด a. 10% calcium gluconate 102. Pt. มี ไ ข้ ตำ่ า ไ อ เรื้อรัง นำ้าหนักลด CXR :cavitary lesion + infiltration at RUL , AFB negative 3 ค รั้ ง ก า ร รักษาที่เหมาะสมที่สุดในผู้ ป่วยรายนี้คือข้อใด a. Tx. TB เลย 103. Pt. มีอาการปวดหัว เป็นๆหายๆ มี HT ตรวจ พบ Na 140, K 3.0, สาเหตุของ HT คืออะไร a. Pheochromocytoma b. Primary hyperaldosteronism c. Renal a. stenosis 104. Pt. ,มี tinia ungium การรักษาที่เหมาะสมที่สุด ในผู้ป่วยรายนี้คือข้อใด a. Topical ketoconazole b. Oral ketoconazole c. Oral itraconazole d. Topical itraconazole Topical 105. ผู้ป่ วยได้ penicillin มานาน ต่อมา diarrhea ต ร ว จ proctoscope พ บ ลักษณะ เข้าได้กับ AAC การรักษาที่เหมาะสมที่สุด ในผู้ป่วยรายนี้คือข้อใด a. Metronidazole 106. pt. post arrest มี EKG ให้ดูเป็ น VT การ รักษาที่เหมาะสมที่สุดในผู้ ป่วยรายนี้คือข้อใด a. Defibrilation 107. pt pale + jx ตับ ม้ามไม่โต จะส่งตรวจ อะไร:
  • 30. a. Hb typing b. inclusion body c. BMA d. G-6 PD screening e. Coomb’s test 108. ผู้ป่ วย Clinical DKA การรักษาที่เหมาะสมที่สุด ในผู้ป่วยรายนี้คือข้อใด a. i.v. insulin b. i.v. fluid 109. ผู้ป่ วย clinical DIC การรักษาที่เหมาะสมที่สุด ในผู้ป่วยรายนี้คือข้อใด a. ATB b. Plt. c. FFP 110. รูป EKG เป็น PVC 3 ตัวติดกัน ในผู้ป่วยรายนี้ น่าจะคิดถึงโรคใดมาก ที่สุด a. Ventricular tachycardia 111. รู ป EKG เ ป็ น complete heart block การรักษาที่เหมาะสมที่สุด ในผู้ป่วยรายนี้คือข้อใด a. Atropine Pediatrics 1. เด็กอายุ 5 ปี มาด้วยไข้ เจ็บ คอ ตรวจร่างกายพบ cervical lymphadenopathy, Splenomegaly and gray patch at tonsil เชื้อใดต่อไปนี้เป็นเชื้อ ก่อโรคในผู้ป่วยรายนี้ A) EBV B) Diptheria C) GAS D) Measles 2. เด็กหญิงอายุ 6 ปี มีผื่นขาว ขุยเล็กๆ ที่ใบหน้า เห็นชัดเวลา ว่ายนำ้า เป็ นมานาน 3 เดือน ไม่คัน PE: Multiple ill-defined, fine scaly patch, hypopigmented macule and patch ที่ห น้ า แ ล ะ ค อ KOH: negative จงให้การวินิจฉัย A) Vitiligo B) Pityriasis alba
  • 31. C) Pityriasis vesicolor D) Tinea facialis E) Contact leukoderm 3. ผู้ ป่ ว ย เ ด็ ก ช า ย อ า ยุ 3 ปี สบายดีมาตลอด หลังเล่นกับพี่ มีไอ Lung: rhonchi right lung ควรส่ง investigation อะไร A) AP, lateral B) PA, lateral C) Inspiration and exspiration D) Right lateral decubitus-3yr can’t do C E) Left lateral decubitus 4. Term newborn BW = 4000 g มารดาเป็ น DM ชักเกร็งทั้ง ตัว สั่น และมือเท้าเย็น Blood sugar = 26 mg% ให้การรักษา ด้ ว ย ส า ร นำ้า ช นิ ด แ ล ะ อั ต ร า เท่าใดจึงจะเหมาะสมที่สุด A) Oral 10% D/W 10 ml/kg B) IV 10% D/W 2 ml/kg+maintain GFR6- 8mg/kg/min C) IV 10% D/W 4 ml/kg D) IV fluid with GPR 4- 6 mg/kg/min E) IV fluid with GPR 6- 8 mg/kg/min F) IV hydrocortisone 10 mg/kg/day และนมแม่ 5. เด็กอายุ 4 ปี มาด้วยไข้และ ซึม BP = 90/60 mmHg, PR= 120 /min ตรวจร่างกายพบจุด ตามตัว Stiff neck positive เชื้อ ที่เป็ นสาเหตุการเกิดโรคในผู้ ป่วยรายนี้มากที่สุดคือ Neisseria meningitis 6. เด็กหญิงไม่มีประจำา เดือน ต ร ว จ ร่ า ง ก า ย พ บ webbed neck, wide nipple distance and low hair line ค ว า ม ผิ ด ป ก ติ ข อ ง ผู้ ป่ ว ย ร า ย นี้อ ยู่ ที่ ฮอร์โมนชนิดใด A) Decrease Insulin B) Decrease GH C) Increase estrogen D) Increase GnRH E) Decrease PTH 7. เด็กหญิงอายุ 14 ปี ไม่มี ประจำา เดือนมานาน 8 เดือน
  • 32. ไม่มีก้อนที่คอ ไม่มีใจสั่น/ เหงื่อ ออกมากผิดปกติ ปฏิเสธนำ้านม ไหลผิดปกติ ไม่ได้ลดนำ้าหนัก หรือออกกำา ลังอย่างหักโหม ตรวจร่างกายไม่พบความผิดปก ติ ใ ด ๆ ทำา progesterone challenge test: negative แ ล ะ estrogen-progesterone challenge test: negative ก า ร วินิจฉัยใดเหมาะสมที่สุด A) Ovarian failure B) PCOS C) Exercise amenorrhea D) Asherman’s syndrome 8. เด็กอายุ 5 ปี ปวดท้อง ปวด ข้อ และพบผื่นขึ้นที่ขาทั้งสอง ข้าง การส่งตรวจในข้อใดต่อไป นี้มีประโยชน์ในการประเมิน prognosis มากที่สุด A) CBC B) ESR C) Skin biopsy D) UA 9. ปวดข้อและพบผื่นกดเจ็บสี แดงที่หน้าแข้งทั้งสองข้าง จง ให้การวินิจฉัย Erythema nodusum asso TB 10. NB 35 wk, 2700 g แ ม่ Blood group O, Rh positive นำ้าเดิน 4 ชั่วโมง หลังคลอด เ ห ลื อ ง Hct 40% TB = 10 เหลืองจากข้อใดต่อไปนี้เป็นไป ได้มากที่สุด A) Sepsis B) Prematurity C) Thalassemia D) G6PD deficiency E) ABO incompatibility 11. เด็กอายุ 2 ปี สุขภาพแข็ง แรงดี เมื่ออายุ 1 ปี อยู่สถานรับ เลี้ยง และเป็ นหวัดบ่อย 1-2 ครั้งต่อสัปดาห์ ครั้งละ 3-5 วัน ควรแนะนำาอย่างไร A) Pneumococcal vaccine B) ติดเครื่องฟอกอากาศ ที่สถานเลี้ยงเด็ก
  • 33. C) ใ ห้ กิ น antihistamine ป้ องกันอาการ D) ทำา skin test E) ไม่ทำาอะไร เป็น normal ของเด็ก 12. เด็กอายุ 2 ปี ไข้สูง 1 wk เจ็บคอ หายใจลำาบาก ตรวจ ร่ า ง ก า ย พ บ retraction, greenish patch at posterior pharynx and tonsils จงให้การ วินิจฉัย A) Croup B) Diptheria C) Candidiasis D) Streptococcus tonsillitis E) Infectious mononucleosis 13. เด็กอายุ 6 ปี U/D Asthma ไข้ ไอ นำ้ามูก 2d หอบมากขึ้น PE: ไข้ คอแดง suprasternal, intercostal, subcostal retraction, inspiratory and exspiratory wheezing, RR 40/min ใ ห้ salbutamol 3 dose ทุก 20 min หอบน้อยลง แต่ยังมี exp. wheezing อยู่ ใน ขณะนั้นให้การรักษาอะไรเพิ่มดี ที่สุด A) Oral mucolytic B) Oral antibiotic C) IV Aminophylline D) IV dexamethasone E) NB. Ipratropium bromide 14. เด็ก 10 ปี U/D Bthal/HbH ต้องรับเลือดทุกเดือน แพทย์ พิจารณาทำา splenectomy ก่อน ที่จะ Sx ควรได้รับ Vaccine ใด เพื่อ prophylaxis A. Typhoid vaccine B. Influenza vaccine C. Pneumococcal vaccine D. Meningococal vaccine 15. ผู้ป่วยหญิง 13 yr คอโต แบบ generalize มา 3 mo ไม่มีอาการอื่น FT4 1.25, TSH 3.5 Dx 1. Simple goiter 2. Hashimoto thyroiditis
  • 34. 3. Subacute thyroiditis 4. Iodine deficiency 5. CA thyroid 16. หญิง 2 ปี ขาอ่อนแรง เดิน ไม่ไหว PE: mild pale, no Jx มี nodule ที่หัว มีก้อนที่ท้อง ขนาด 6x8 cm cross midline hard consistency Dx A) Neuroblasoma ป ว ด ท้อง B) Wilm’s tumor มี hematuria มีก้อน C) ANLL D) Non-Hodkin lymphoma E) Retroperitoneal teratoma 17 เด็กหญิงอายุ 4 ปี มี RUQ pain with mass urine:RBC 10- 20,WBC 1-2 จงให้การวินิจฉัย โรค a. Wilm’s tumor b. Neuroblastoma c. Lymphoma d. Hydronephrosis e. .............. 18. เด็กชาย 3 ปี ไข้ ซีด เหลือง reticulocyte 7% ให้ตรวจอะไร ต่อ A) G6PD screening 19. เด็กชาย 9 ปี เรียนป.3 เขียนหนังสือไม่ค่อยได้ ต้อง สะกดทีละคำา IQ test ได้ 109 คะแนนการอ่าน 79 การเขียน 80 การคำานวณ 97 ถาม เด็กเป็นอะไร A) dyslexia B) dyscalculia C) mental retardation D) Attention deficit disorder E) Autism 20. เด็กอายุ 3 ขวบ ไข้มา 3 วัน แล้วซีด PE : Moderately pale , mild icteric sclera , No hepatosplenomegaly + ตรวจ ร่างกายอย่างอื่นปกติ Lab : Hb 7.7 , WBC 4,800 , Lym เด่น ,plt 380,000 , RC 7.5 , MCV 87 ต้องตรวจอะไรเพิ่ม A. Coomb’s test
  • 35. B. Inclusion test C. Hemoglobin typing D. G6PD screening E. Bone marrow aspiration 21. เด็กกินปลาทอด มีผื่นขึ้น ตามตัว หายใจมีเสียง wheeze ประมาณ anaphylaxis ถามว่า first med คืออะไร ตอบ Adrenaline (Epinephrine) 22. เด็ก 1 ปี มีไข้ซึม 1 วัน มี ไ ข้ V/S T 39.8 , PR 140, RR 40, BP 90/60, CBC : Hb 11, WBC 20,000 N 93% , Plt 50,000 Coagulogram : prolong , Petechiae and Necrotic purpura at legs ถาม ว่า การ manage ที่สำาคัญ คือ A) fluid therapy B) antimicrobial C) Plt concentration D) FFP E) vasopressin 23. เด็กคลอดที่บ้าน กินแต่นม แ ม่ มี ecchymosis ต า ม ตั ว ตอบ Vit K deficiency ผู้ป่วยเด็ก มีหนังตาบวม ตัว บวมทั่วทั้งตัวมา 2 เดือน ตรวจ ร่างกายมี Ascites UA : Proteinuria 3+ , RBC 0-1/HPF , Fine granular cast 0-1 , Fat oval cast 0-1 อะไรทำาให้เกิด การบวม A. Decrease Plasma oncotic pressure B. Increase Plasma Hydrostatic pressure C. Oncotic pressure ใน cavity เพิ่มขึ้น D. Hydrostatic pressure ใน cavity ลดลง E. Increase Vascular permeability 24. เด็กหญิงอายุ 12 ปี เป็นลม หมดสติ 1 นาที หลังฟื้ นปกติ
  • 36. ไ ม่ เ ค ย เ ป็ น แ บ บ นี้ม า ก่ อ น management อะไรต่อ A. EKG B. Blood glucose C. 24 – hr holter monitoring D. EEG E. No further investigation 25. ผู้ป่วยหญิงเป็น TB AFB 2+ CXR พบ cavity มีลูกอายุ 6 เดือน ตรวจ PPD 4 mm ถามว่าจะ manage ลูกอย่างไร 26. เด็กชาย 8 ปี มีปวดท้อง คลื่นไส้อาเจียน ปัสสาวะบ่อย มา 3 วัน มีไข้ BP 90/70, PR 120, RR 30, poor perfusion, Lab: Na 145 Cl 110 HCO3 10, urine ketone 4+ Management ที่ต้องทำาเป็น อันดับแรกคืออะไร 1. Insulin 2. 0.9%NaCl 3. Potassium 4. HCO3 27. ผู้ป่วยเด็กอายุ 8 ปี มีค่า PT, PTT prolong เจาะเลือด พบ Platelet 50,000 ตรวจ ร่างกายพบ purpura, petechiae จะ management อย่างไร A) FFP B) Cryo C) Platelet conc. 28. ผู้ป่วย newborn มีประวัติ แม่ตั้งครรภ์ 41 wk with meconium stain AF + fetal distress จะทำาอย่างไร A) เช็ดตัว + ดีดเท้า B) ET tube + suction 29. เด็ก term อายุ 2 เดือน คลอดที่บ้าน BW 4,600 gm PE: BT = 37, PR = 150 AF bulging, mod pale, no jx, CBC: WBC 8,500 platelet 190,000 คิดว่าเป็นอะไร A) sepsis B) SDH C) Idiopathic vitamin K deficiency of infant 30. ผู้ป่วย newborn ตรวจ ร่างกายพบ heart murmur,
  • 37. cataract, ตับม้ามโต คิดว่า congenital infection เชื้อใด (Rubella) 31. ผู้ป่วยเด็กอายุ 6 เดือน มี อาการเขียวมาตั้งแต่อายุ 4 เดือน ครั้งนี้มีเขียวมากขึ้นเมื่อ ร้องไห้ PE: tachycardia, tachypnea, pansystolic murmur 2/6 at left parasternal border, cyanosis คิดว่าเป็น ภาวะใด 1. Hypoxic spell 2. Asthma 3. Pulmonary... 32. ผู้ป่วยเด็กอายุ 2 ปี 4 วัน PTA มีไข้ 38 องศา PE: drooling กลืนลำาบาก, posterior pharynx บวม Dx A) Acute tracheitis B) Acute epiglottitis C) Viral croup D) Laryngitis E) Retropharyngeal abscess 33. ผู้ป่วยเด็กอายุ 3 ปี มารับ วัคซีน เคยรับวัคซีนครบจนถึง อายุ 1 ปี จากนั้นไม่ได้รับวัคซีน อีกเลย จะให้วัคซีนใดใน visit นี้ A) OPV, DPT B) Hib C) MMR, OPV, DPT D) HBV E) OPV, DPT, JE 34. ผู้ป่วย newborn preterm GA 32 wk นำ้าหนักแรกคลอด 1,500 gm หลังคลอด 12 ชั่วโมงต่อมามีอาการหายใจ ลำาบาก ถามว่าขาดสารตัวใด ในปอด A. Spyringomyelin B. Lacitin C. Phosphatidyl inositol D. Phosphatidyl glycerol 35. ผู้ป่วยเด็กอายุ 4 ปี มีไข้ 39 C PE: milky patch at tonsil, cervical LN โต 1-2 node ขนาด 2 cm 2 ข้าง, นอนกรน หายใจลำาบาก liver
  • 38. 2 cm BRCM, spleen 2 cm BLCM จงให้การวินิจฉัย 1) Infectious mononucleosis 2) TB lymphadenitis 3) Streptococcal pharyngitis 4) Dipthelia 5) Lymphoma 36. ผู้ป่วยเด็กอายุ 4 ปี ปู่เป็น TB ตรวจร่างกายไม่พบ BCG scar CXR ปรกติ ทำา tuberculin test 13 mm จะทำา อย่างไรต่อไป A) INH prophylaxis B) start ยา TB C) repeat tuberculin test อีกที 3 เดือนต่อมา D) ฉีด BCG E) AFB from gastric lavage 37. ผู้ป่วยเด็กอายุ 9 เดือน กินนมแม่ มีไข้ตำ่า ๆ ได้รับยา para, amoxy มา 3 วัน ต่อมา ถ่ายเหลวเป็นนำ้า PE: sunken eyeball, dry lips, perianal redness ท่านคิดว่าเกิดจาก สาเหตุใดมากที่สุด A) Salmonella B) Shigella C) Rotavirus D) Lactose deficiency E) Antibiotic related diarrhea 38.เด็กชาย 1 ขวบ มีไอ มี นำ้ามูก barking cough, inspiratory stridor, expiratory rhonchi จะทำาอย่างไรนอกจาก ให้ IV 1) Ampicilin IV 2) Cloxacilin IV 3) พ่น beta-agonist 4) พ่น adrenaline 5) ETT 39. เด็กอายุ 3 ปี ไอแบบ Barking cough จง Mx 1. Adrenaline NB stat 2. Salbutamol NB stat 3. Dexamethasone IV 4. Prednisolone po 40. เด็กอายุ 7 วัน ไข้ ซึม ร้อง เวลาจับเต้านมขวา มีนำ้านมไหล
  • 39. ตั้งแต่อายุ 2 เดือน มารดาบีบ ออกทุกวัน , PE: lethargy, moderate jaundice, erythematous indurated mass at Rt. Breast, ควรให้ ATB ? 1. Ceftriaxone 2. Ampicillin +Gentamicin 3. Cloxacillin + Gentamicin 4. Fortum + Amikacin 5. Fortum + Vancomycin 41. ผู้ป่วยเด็กชาย ได้ยา ampicillin มา 14 วัน หลังจาก นั้นมีไข้ ท้องเสีย ถ่ายเหลว ทำา endoscope พบว่ามี hyperemia Tx? 1. Metronidazole 2. Ceftriaxone 42. เด็กชาย อายุ 8 ปี เมื่อ ก่อนแข็งแรงดี มีประวัติเป็น หวัด 1 PTA, มีจำ้าเลือดตามตัว และเลือดออกตามไรฟัน 2 wk PTA, vital sign ปกติ, PE : Oozing per gum, petichiae & ecchymosis ที่ trunk & extremities, no lymphadenopathy, no hepatosplenomegaly, Dx? 1. Hemophilia 2. Acute leukemia 3. DHF 4. ITP 5. APDE 43. Female Infant 5 hr APGAR 8,9 ที่ 1,5 min BW 4,200 gm. คลอดมามีอาการ ชักเกร็ง ตัวยาว 52 cm. Hc 35 cm. The most appropriate investigation for Dx this Patience’s Disease is? 1. Plasma Glucose 2. Urine Ketone 3. Serum Ammonia 44.(ชุด A) Case preterm 32 wk. BW 1,500 g ดูหายใจ เร็วๆ ถามว่าเกิดจากภาวะ อะไร 1. Respiratory distress syndrome เด็กชายอายุ 4 ปี จมนำ้าในสระ ว่ายนำ้า หลังจาก resuscitation แล้วหายใจได้เอง แต่ยังไม่ค่อย
  • 40. รู้สึกตัว นำาส่ง ER P.E. : T = 36.5 , PR = 140 , RR = 55 ,BP = 90/60 , drowsy , mild cyanosis , diffuse fine crepitation and expiratory wheezing , capillary refill 4 sec , O2 sat 85% On O2 mask with bag 5 LPM ไอมี frothy pink จะทำาอะไร A. IV dopamine B. Nebulized adrenaline C. Nebulized bronchodilator D. On O2 mask with bag 10 LPM E. Positive pressure ventilation 45. เด็ก 11 ปี , 1 เดือนก่อน ไม่สบาย มีจำ้าเลือดตามตัว, CBC : Plt ตำ่า อย่างเดียว ,Dx? 1. ITP 46. เด็กชาย อายุ 15 ปี ไป สถานกักกันมา 2 ปี ตรวจ ร่างกาย พบ Upper/Lower motor weak gr IV/II DTR no impaired pain and touch, glottic stroking pattern 1. Beri Beri 2. GBS 47. เด็กอายุ 7 ปี มีอาการคัน ตาเป็นๆหายๆ มา 1 ปี ตรวจ ตา พบ Giant papillae ลักษณะ เป็น Cubblestone การวินิจฉัย ที่เป็นไปได้มากที่สุด 1. Trachoma 2. Inclusion Conjunctivitis 3. Hay fever Conjunctivitis 4. Vernal karatoconjunctivitis 5. Epidemic Conjunctivitis 48. เด็กมาด้วยอาการ Drooling, miosis, dysphoresis, profuse sweating ถามว่า เกิด จากสารใด 1. Ethanol 2. Organophosphate 3. Opioid 4. Ephedrine 5. Cocaine
  • 41. 49. ทารกอายุ 6 ชั่วโมง แรก คลอดไม่ดูดนม หายใจหอบ เหนื่อย ฟัง Breath sound ได้ ข้างขวาข้างเดียว เสียงหัวใจอยู่ ทางขวา X-ray พบ heart ที่ Rt.chest ท้องยุบ จง Dx. 1. Situs inversus 2. TE fistula 3. Diaphragmatic hernia 50. เด็กหญิงอายุ 10 ปี เป็นโรค ผนังกั้นหัวใจรั่วแต่กำาเนิด เคย เข้ารับการรักษาด้วยหัวใจล้ม เหลวหลายๆครั้งตอนเล็กๆ เมื่ออายุ 3 ปี อาการเหนื่อย น้อยลง ตอนนี้มาด้วยเหนื่อย มากขึ้นมา 2 เดือน นอนราบได้ PE: central cyanosis, clubbing finger, Loud P2, diastolic rumbing murmur 3/6 at Lt upper parasternal border lung: clear ข้อใด น่าจะเป็นสาเหตุ ของการเหนื่อยในเด็กรายนี้ 1. Pulmonary embolism 2. Aortic regurgitation 3. Hypoxic spell 4. Eisenmenger complex 5. Congestive heart failure 51. เด็กชายอายุ 12 ปี มาด้วย อาการเป็นลม ล้มหมดสติไป 1 นาที ตื่นมาปกติดี เกิดขณะ เข้าแถวตอนเช้า ตรวจร่างกาย พบว่าปกติ, neurologic exam normal ต้องทำาอะไรต่อหรือไม่ 1. EKG 2. Blood sugar 3. 24hr Holter monitor 4. EEG 5. ไม่ต้องทำาอะไรเพิ่มเติม 52. เด็กอายุ 7 วัน นำ้าหนักแรก คลอด 2,800 gm กินนมแล้ว อาเจียนเป็นนำ้าสีเหลือง หลัง จากนั้นซึม ดูดนมน้อยลง นำ้า หนักลดเหลือ 2,400 gm จง Dx. เบื้องต้น 1. Duodenal atresia 2. Pyloric stenosis 3. Inborn error metabolism 4. Sepsis 5. Hirchsprung disease 53. ผู้ป่วยเด็กหญิงอายุ 9 ปี มี อาการหงุดหงิดง่าย การเรียน ตกตำ่าลง PE: Thyroid diffuse
  • 42. enlarged, exophthalmos, smooth skin. PR120 FT4 เพิ่ม TSH ลด Antityroglobulin 1:1600 จะรักษาอย่างไร 1. Iodized oil 2. Iodized salt 3. Lugol solution 4. Methimazole 54. เด็ก 6 yr ได้ vaccine ครบ ตอน 5 yr ถูกแมวข่วนขนาด 2 cm ต้องได้รับ vaccine อะไร บ้าง 1. Rabies vaccine 2. TT 3. Rabies vaccine + Tetanus toxoid 4. Rabies vaccine และ Rabies Ig 5. TT + Rabies vaccine และ Rabies Ig 55. เด็ก 7 ปี Dx ว่าเป็น Chickenpox ควรแนะนำาแม่ว่า จะให้เด็กไป รร ได้เมื่อไร 1. หลังจากไข้ลด 2. เมื่อแผลตกสะเก็ดหมด 3. เมื่อตุ่มนำ้าใสเม็ดสุดท้าย ลอก 4. หลังกิน Acyclovir 48 hr 5. ไปได้เลย แต่ต้องสวมเสื้อ ให้มิดชิด 56. เด็ก 9 ปี มีอาการปวดหัว ปวดท้องก่อนไป รร ไม่อยากไป รร กลัวมีเหตุการณ์ลักพาตัว แม่บอกว่ามีฝันร้ายตอนกลาง คืน ไม่ไป รร 3 wk ถาม Dx 1. Generalized anxiety 2. Separation anxiety 3. Simple phobia 57. เด็ก หายใจหอบเหนื่อย มี กลิ่นเหม็น hyperpnea, +ve urine ketone 4+ Dx อะไร 1. DKA 2. Sepsis 3. meningitis 58. ผู้ป่วยเด็กชายอายุ 9 mo มาด้วย otitis media เคยเป็น โรคติดเชื้อเป็นๆหายๆ ตั้งแต่ 3 mo ก่อน พวก pneumonia, S.pneumo, Hib, ถามว่าน่ามี defect ที่ใด 1. B cell
  • 43. 2. T cell 3. Phagocytic defect 4. Complement 59. เด็ก 4 yr ไข้สูง 1 d มีผื่น แดงตามตัว วันนี้ซึมลงจึงมา รพ. PE: BP 90/60, RR 40, PR 120, stiff neck +ve, Purpura??? ถาม Dx 1. Salmonella 2. N. meningitides 3. Listeria monocytogenes 4. S. pneumonia 5. H. influenza 60. เด็ก 10 mo มีไข้ ไอ นำ้า มุก มา 3 mo วันนี้ไอมาก หายใจหอบเหนื่อย มี inspiratory stridor, chest drawning ภายหลังให้ epinephrine มีหอบน้อยลง epinephrine มีผลอย่างไร 1. ขยายหลอดลม 2. ขับเสมหะ 3. ขับนำ้าที่ค้างในปอด 4. ลดบวมเยื่อบุทางเดิน หายใจ 61. มารดาเป็น DM เด็กเกิดมา มีอาการสั่น มี sign ของ hypoglycemia DTX 30 Hct 63 ถาม Management 1. Early feeding 2. Bolus 10%D/W 2ml/kg 3. Plasma exchange 62. เด็กชายอายุ 4 mo ซึมลง WBC 18000 (N 80%), Hct 6.9%, Dx อะไร 1. Subdural hematoma 63. เด็กอายุ 4-5 ขวบ มีนำ้ามูก ไหลกลิ่นเหม็น รักษาด้วย Amoxy 2 วัน อาการไม่ดีขึ้น มาพบแพทย์ตรวจพบหนองใน ช่องจมูกมาก ทำาอะไรต่อไป a. ส่ง X-ray PNS b. ตรวจด้วย nasal speculum ซำ้าอีกครั้ง c. Amoxicillin/Clavulinic acid d. ใช้ Amoxycillin เดิมต่อ ไป e. Consult โสต ศอ นอ สิกแพทย์ 64. เด็กในข้อใดมีพัฒนาการช้า
  • 44. a. อายุ 10 เดือน ตั้งไข่ ได้ b. อายุ 12 เดือน เดินได้ c. อายุ 2 ปี เดินก้าวขึ้น บันได้สลับข้างได้ d. อายุ 3 ปี พูดคำาโดด ได้ วลี 10-20 คำา e. อายุ 5 ปี วาดรูป สามเหลี่ยมได้ 65. เด็กหญิง เป็น UTI รักษา ด้วย Ampicillin ครบ 14 วัน จากนั้นปวดท้อง ไข้ขึ้น ตัว เหลือง อุจจาระมี WBC=1- 5,RBC>100 ส่องกล้อง scope ดู เจอ Patch? ให้ยาอะไร a. Cotrimoxazole b. Norfloxacin c. Metronidazole 66. เด็กผู้ชาย มีอาการเหมือน Cushing syndrome จะตรวจ อะไรผิดปกติ A. Hypocalcemia B. Hyperkalemia C. Increased cortisol level D. Advance bone age E. Increased renin plasma activity 67.เด็กหญิงอายุ 4 yr ซีด 2 wk ก่อนหน้านี้ปกติ PE: Pallor, petichiae บริเวณแขนขา CBC Hb 7, Hct 22, WBC 3200(N 35, E 3, L 62), Platelet 26000, MCV 90 จะส่ง Investigation อะไรเพื่อการ วินิจฉัย A. Serum ferritin B. Bleeding time C. Reticulocyte count D. Screening coagulogram E. Bone marrow aspiration 68. เด็กหญิง 9 เดือน กินนม แล้วสำาลักบ่อย เป็นปอดบวม 3 ครั้ง BW 7 kg PR 120 lung: wheezing both lung, CXR: perihilar infiltration, PPD 4 mm หลังได้ brochodilator แล้ว ดีขึ้น Dx A. Retian FB B. Diaphragmatic hernia C. TB D. GERD
  • 45. E. Asthma 69. เด็กมีไข้ มีผื่นตาม ตัว(Purpura fulminan) Stiff neck : positive ถามว่าเกิดจาก เชื้ออะไร - N.meningitidis 70. เด็ก asthma มีอาการ เฉพาะเวลาออกกำาลังกายและ ตอนกลางคืน 2-3 ครั้งต่อเดือน ถามว่าต้องให้ยาอะไร A. Oral salbutamol B. Inhale sulbutamol C. Inhale steroid D. Inhale ipratropium + Salbutamol 71. เด็กอายุ 6 เดือนกระตุก ตอนเช้าเป็นมา 2 สัปดาห์ โดย เริ่มจากผงกศีรษะก่อนแล้ว ทำาท่ามือโอบกอด จง วินิจฉัย---infantile spasm 72. เด็กชาย อายุ 10 ปี MCA มาที่ ER ด้วยอาการปวดบวม และขยับข้อศอกขวาไม่ได้ PE: swelling at Rt elbow, normal triangle landmark of Rt elbow, notmal neurovascular จง Dx A. Elbow dislocation B. Displaced olecranon fracture C. Displaced supracondylar fracture D. Displaced lateral condylar fracture E. Displaced medial supracondylar fracture 73. เด็กชายอายุ 6 ปี แม่พา มาเนื่องจากอ้วนมี truncal obesity , Buffalo hump , moon face c acne , ตัวเตี้ย จะตรวจเพิ่มเติมพบอะไร A. Hypocalcemia B. Hyperkalemia C. Advance bone age D. Increase serum cortisol
  • 46. E. Increase activity of rennin 74. เด็กหญิงอายุ 6 ปี มีจำ้า เลือดตามแขนขามา 2 สัปดาห์ ตรวจร่างกายพบ petechiae & ecchymosis at extremities CBC : Hb 12 Hct 36 WBC 6500 Plt.150,000 ผล Lab ใด จะผิดปกติ A. Bleeding time B. PT C. PTT D. Thromin time E. Venous clotting time 75. เด็กอายุ 2 ปี ผ่าตัด VSD หลังผ่าคลำา pulse ไม่ได้ วัด BP ไม่ได้ทันที ได้บีบ AMbu ผ่าน tube และให้ Amiodalone แล้วมี EKG เป็น ventricular tachycardia จง management A. Defibilation B. Ca gluconate C. NaHCO3 D. Adenosine E. Direct – current synchronized cardioversion 76. เด็กชายอายุ 19 ปี มาพบ แพทย์หลังจากไปถอนฟั นกับ ทันตแพทย์ แล้วเลือดไหลซึม ไม่หยุด ตรวจตับและม้ามไม่ โต ตรวจเลือด PTT 50 sec (control 30 sec), PT 13 sec (control 14 sec), no prolonged bleeding time สาเหตุของการเกิดโรคนี้คือ อะไร A. Lupus coagulopathy B. Factor VII deficiency C. Factor VIII deficiency D. von Willebrand factor deficiency E. Hereditary platelet dysfunction
  • 47. 77. เด็ก 3 ปี เหนื่อยง่าย มี mild pale other WNL หนัก 13.5 kg , blood smear : hypochromic microcytic anemia A. Iron deficiency Surgery 1. ผู้ป่ วยชายไทยอายุ 47 ปี มี ก้อนยุบๆ บวมๆ ที่ขาหนีบ เป็นๆ หายๆ มา 3 ปี ครั้งนี้ มาพบแพทย์ที่โรงพยาบาล ผู้ ป่ วยมาด้วย ก้อนบวมและโต มากขึ้น กดเจ็บมากและดัน ก้ อ น ก ลั บ ไ ม่ ไ ด้ ตั้ ง แ ต่ 5 ชั่วโมงก่อนมาโรงพยาบาล การปฏิบัติในข้อใดเหมาะสม ที่สุดในผู้ป่วยรายนี้ a. Analgesic b. Try reducing c. Observe and plan admit d. Surgery e. Reassure and plan discharge 2. ผู้ป่ วยชายไทยมีอาการปวด ท้องที่บริเวณ epigastrium ทันทีขณะที่พึ่งดื่มเหล้าไปได้ 2 แก้ว มาพบแพทย์ที่โรง พ ย า บ า ล แ พ ท ย์ เ ว ร ไ ด้ ทำา การตรวจร่างกายพบ Abdomen : generalized guarding, absent bowel sound การส่งตรวจในข้อใด เหมาะสมที่สุดในผู้ป่วยรายนี้ a. Plain abdomen b. CT abdomen c. Ultrasound d. Abdomen series e. MRI 3. ผู้ป่ วยหญิงอายุ 22 ปี คลำา ได้ก้อนที่เต้านมขวา 3 เดือน ก่อน ตรวจร่างกายพบก้อน ข น า ด 2 cm oval shape, firm, smooth and movable การวินิจฉัยในข้อใดเป็นไปได้ มากที่สุด a. Fat necrosis b. Fibroadenoma c. Fibrocystic disease d. Intraductal carcinoma
  • 48. e. Intraductal papilloma 4. ผู้ป่ วยชาย 56 ปี ประวัติ ครอบครัวพี่ช ายเป็ นโรค มะเร็งลำาไส้ใหญ่แล้วเสียชีวิต จึ ง ม า ต ร ว จ ร่ า ง ก า ย ทำา colonoscopy พบ polyps at descending colon แพทย์ทำา polypectomy การตรวจพบ ในข้อใดมีโอกาสพัฒนาเป็ น มะเร็งมากสุด a. Villous adenoma b. Tubulovillous adenoma c. Tubulous adenoma 5. ผู้ ป่ ว ย ช า ย เ ป็ น hernia ที่ testis ด้ า น ข ว า โ ต ม า 1 ชั่วโมงก่อนมาโรงพยาบาล การปฏิบัติในข้อใดเหมาะสม ที่สุดในผู้ป่วยรายนี้ a. Reduction b. Ultrasound c. Surgery d. Radiation e. Reassure 6. ผู้ ป่ ว ย ช า ย อ า ยุ 30 ปี มี อาการเจ็บและปวดบริเวณถุง อัณฑะด้านซ้ายมา 5 วัน ก่อนมาโรงพยาบาล การ ตรวจร่างกายเบื้องต้นพบว่า มีอาการปวด บวม แดง อาการเจ็บลดลงเมื่อยกถุง อัณฑะขึ้น แพทย์เจ้าของไข้ ส่ ง ต ร ว จ Ultrasound พ บ increase vascular blood flow การวินิจฉัยในข้อใด เป็นไปได้มากที่สุด a. Orchitis b. Epididymitis c. Incarcerated hernia d. Acute torsion testis e. Torsion of appendix testis 7. ผู้ป่วยชายไทยเคยเป็นโรคตับ อ่อนอักเสบ รักษาจนหาย ดีแล้ว แต่ยังมีอาการจุกแน่น ท้อง ท้องอืด เบื่ออาหาร นำ้า ห นั ก ล ด ต ร ว จ พ บ pseudocyst at the front of pancreas ตั้งแต่ 4 สัปดาห์ ก่อน ก้อนโตขึ้นอย่างรวดเร็ว การปฏิบัติในข้อใดเหมาะสม ที่สุดในผู้ป่วยรายนี้ a. Partial Pancreatectomy
  • 49. b. Total Pancreatectomy c. Excision of pseudocyst d. Internal drainage e. Percutaneous drainage 8. ผู้ ป่ ว ย ช า ย โ ด น หิ น เ จี ย ร กระเด็นทิ่มคอขณะกำา ลัง ทำางาน ตำาแหน่งที่โดนใกล้ ลูกกระเดือก 30 นาทีก่อน มาโรงพยาบาล ช่วงแรกที่ บาดแผลมีเลือดออกตาม จังหวะการเต้นของหัวใจ, วัด ค ว า ม ดั น โ ล หิ ต ไ ด้ 86/70 mmHg ขณะนี้เลือดหยุดไหล แล้ว มีแผล laceration 3 cm anterior to sternocleidomastoid ก า ร ปฏิบัติในข้อใดเหมาะสมที่สุด ในผู้ป่วยรายนี้ a. Explore wound at ER b. Explore wound at OR c. CT d. MRI e. Angiography 9. ช า ย อ า ยุ 30 ปี ป ร ะ ส บ อุ บั ติ เ ห ตุ ร ถ ช น ต้ อ ง ทำา distal below knee amputation แ พ ท ย์ พั น stump ไว้ หลังจากนั้นแพทย์ ควรจะนัดผู้ป่ วยมาเพื่อใส่ขา เทียมแบบถาวร นานเท่าไร a. 7 วัน b. 10 วัน c. 15 วัน d. 30 วัน e. 45 วัน 10. หญิงอายุ 16 ปี มีก้อนที่ เต้านมด้านขวา ก้อนขนาด ป ร ะ ม า ณ 2 cm. ป ว ด สัมพันธ์กับรอบเดือน แพทย์ ตรวจร่างกายพบว่ามี well define 2.5 centimeter mass, firm consistency at the right breast การปฏิบัติ ในข้อใดเหมาะสมที่สุดในผู้ ป่วยรายนี้ a. ให้ตรวจ เต้าน มด้วย ตนเอง b. มาพบแพทย์ให้ตรวจเต้า นม c. Fine needle aspiration
  • 50. d. Ultrasound หลังจากนั้น นัดตรวจติดตามทุกๆ 6 เดือน e. Mammogram หลังจาก นั้นนัดตรวจติดตามทุกๆ 12 เดือน 11. ผู้ป่ วยชายอายุ 35 ปี ดื่ม เ ห ล้ า ติ ด ต่ อ กั น 5 วั น มี อาการปวดท้องมากมา 2 วัน ปวดร้าวไปหลัง ผู้ป่วยงอ ตัวแล้วอาการปวดท้องดีขึ้น แพทย์ทำาการตรวจร่างกาย Abdomen - tender at epigastrium, liver dullness, decreased bowel sound การวินิจฉัยในข้อใดเป็นไปได้ มากที่สุด a. Alcoholic gastritis b. Bowel perforation c. Acute pancreatitis d. Spontaneous Bacterial Peritonitis e. Cholecystitis 12. ผู้ป่ วยช าย อายุ 35 ปี ถ่ า ย อุ จ จ า ร ะ มี เ ลื อ ด ป น เป็นๆหายๆมา 6 เดือน โดย 3 วันก่อนมา รพ. คลำาก้อน ได้เท่านิ้วหัวแม่มือ เอานิ้วดัน กลับเข้าไปได้ การปฏิบัติใน ข้อใดเหมาะสมที่สุดในผู้ป่ วย รายนี้ a. Injected 5% phenol b. Rubber band ligation c. Hemorrhoidectomy d. Cryosurgery e. Reassure and follow up 13. ผู้ป่วยชายอายุ 65 ปี ได้รับ ก า ร วิ นิ จ ฉั ย ว่ า เ ป็ น Colorectal Cancer เ มื่ อ 1 เดือนก่อน ครั้งนี้ผู้ป่วยมาโรง พยาบาลด้วยอาการปวดท้อง รุนแรงมา 3 ชั่วโมงก่อนมา โรงพยาบาล แพทย์ทำาการ ต ร ว จ ร่ า ง ก า ย พ บ ว่ า มี restless, body temperature = 39 O C , Blood Pressure =100/70 mmHg ,Pulse Rate =110/min , Respiratory Rate = 26/min generalized abdominal tenderness, guarding,
  • 51. rebound tenderness, absent bowel sound ผ ล จ า ก ก า ร ส่ ง Acute abdominal series พ บ ว่ า มี generalized bowel dilatation with intraabdominal free air แพทย์เจ้าของไข้ควรให้ยา ปฏิชีวนะในข้อใดต่อไปนี้ อะไรก่อนทำาการส่งต่อผู้ป่วย a. Ceftriazone + Metronidazole b. Clindamycin + Gentamicin c. Clindamycin +Ceftazidime d. Ampicillin + Gentamicin e. Ceftazidime + Gentamicin 14. ผู้ป่ วยชายอายุ 65 ปี มี ป ร ะ วั ติ Car accident มี อาการอ่อนแรงตั้งแต่คอลง ไป มี Blood pressure drop, Bradycardia การวินิจฉัยใน ข้อใดเป็นไปได้มากที่สุด a. Cardiogenic shock b. Cushing response c. Hypovolemic shock d. Neurogenic shock e. Septic shock 15. ผู้ป่ วยชายอายุ 35 ปี ได้ รับบาดเจ็บจากอุบัติเหตุ เครื่องบินตก มีอาการปวดที่ กระดูกซี่โครงด้านขวาแพทย์ เวรประจำาห้องฉุกเฉินทำาการ ต ร ว จ ร่ า ง ก า ย พ บ ว่ า มี trachea shift ไปด้านซ้าย , Right pneumothorax (ใ ห้ ข้ อ มู ล เ ป็ น แ บ บ tension pneumothorax) การปฏิบัติ ในข้อใดเหมาะสมที่สุดในผู้ ป่วยรายนี้ a. Pleural tapping b. Observe vital sign c. Oxygen mask with bag d. Emergency ORIF 16. ผู้ป่ วยชายอายุ 31 ปี ได้ รับบาดเจ็บจากการถูกแทงที่ left parasternal border 5 นาทีก่อนมาโรงพยาบาล มี Blood Pressure drop แพทย์ เวรประจำาห้องฉุกเฉินทำาการ ต ร ว จ ร่ า ง ก า ย พ บ ว่ า มี
  • 52. distant heart sound ก า ร ปฏิบัติในข้อใดเหมาะสมที่สุด ในผู้ป่วยรายนี้ a. Echocardiogram b. Pericardiocentesis c. CT chest d. MRI chest e. Observe vital sign and clinical sign and symptom 17. ผู้ป่ วยชายอายุ 13 ปี ได้ รับบาดเจ็บจากอุบัติเหตุมอ เตอร์ไซด์ล้มแพทย์เวรประจำา ห้องฉุกเฉินทำา การตรวจ ร่ า ง ก า ย พ บ ว่ า มี stupor และได้ทำาการตรวจโดย CT scan ผ ล พ บ ว่ า มี cresent shape hemorrhage แ ล ะ hyperdensity area ก า ร วินิจฉัยในข้อใดเป็นไปได้มาก ที่สุด a. acute epidural hemorrhage b. acute subdural hemorrhage c. subacute epidural hemorrhage d. subacute subdural hemorrhage e. subacute intradural hemorrhage 18. ผู้ป่ วยชายอายุ 14 ปี ได้ รับบาดเจ็บจากอุบัติเหตุมอ เตอร์ไซด์ล้มแพทย์เวรประจำา ห้องฉุกเฉินทำา การตรวจ ร่ า ง ก า ย พ บ ว่ า Blood pressure drop ป ร ะ เ มิ น Glasgow Coma Score = 8 การปฏิบัติในข้อใดเหมาะสม ที่สุดในผู้ป่วยรายนี้ a. Cervical Collar b. ET intubations c. CT and MRI d. IV fluid and dopamine if necessary e. Refer to the nearest hospial immediately 19. ผู้ป่ วยหญิงปั สสาวะไม่ ออก ต้องสวนปัสสาวะไป 2 ครั้งเมื่อวันที่ผ่านมา วันนี้ผู้ ป่ วยมาด้วยอาการขาอ่อน
  • 53. แรงทั้งสองข้าง และมีอาการ ชาถึงระดับสะดือ แพทย์เวร ประจำาทำาการตรวจร่างกาย พ บ ว่ า Loose sphincter tone, areflexia both legs ผู้ ป่ วยน่าจะมีพยาธิสภาพถึง ระดับใด a. Parasagittal b. Basal pons c. Thoracic cord d. Lumbosacral nerve root e. Conus medullaris 20. ผู้ป่ วยชายอายุ 14 ปี นำ้า หนักเท่าไรก็ไม่รู้ไม่มีใครจด ออกมาเลย burn 1 st degree burn = 4% ที่บริเวณหน้า, 2 nd degree burn= 27% เมื่อ 15 นาทีก่อน ไม่มี inhalation injury ต้ อ ง ใ ห้ ส า ร นำ้ า ใ น ชั่ ว โ ม ง แ ร ก เ ท่ า ไ ร ต า ม Parkland’s formula a. 200 ml/hr b. 250 ml/hr c. 290 ml/hr d. 340 ml/hr e. 390 ml/hr 21. ผู้ ป่ ว ย ช า ย อ า ยุ 52 ปี ปวดหน้าอกล่างซ้ายทันที หลังอาเจียนรุนแรง แพทย์ เวรประจำาห้องฉุกเฉินทำาการ ตรวจร่างกายพบว่า Leftt lung decrease breath sound, Abdomen: tender with guarding การวินิจฉัย ในข้อใดเป็นไปได้มากที่สุด a. Hiatal hernia b. Esophageal rupture c. Tension pneumothorax d. Mallory-Weiss syndrome e. Deep neck infection 22. ผู้ป่วยชายอายุ 65 ปี U/D AAA มีจำ้าเลือดแขนขา PE: ecchymosis ที่ขาแล ะแขน CBC Hb: 10, Hct: 30, WBC 8600 (N 75, L 25), PT 15, PTT 33, fibrinogen 150 ส่ง lab การส่งตรวจใน ข้อใดเหมาะสมที่สุดในผู้ป่ วย รายนี้ a. D-dimer
  • 54. b. Bleeding time c. Platelet function test d. Protein C, Protein S e. Fibrinogen level 23. ชายอายุ 24 ปี ตาขวา โ ป น Intracranial bruise, chemosis ตาขวา, 2 ปี ก่อน มี severe head injury ได้รับ การรักษาที่โรงพยาบาล การ วินิจฉัยในข้อใดเป็นไปได้มาก ที่สุด a. Intraorbital abscsee b. Carotid cavernous fistula c. Chronic subdural hematoma d. Posttraumatic hydrocephalus e. Ruptured opthalmic aneurysm 24. ชายอายุ 18 ปี ถูกหมา กัดหน้าโรงภาพยนตร์ที่นิ้ว มื อ ข ว า ข น า ด 2x1 cm (โจทย์ไม่ได้บอกลักษ ณะ บาดแผล) หลังจากนั้นล้าง แผลและได้ tetanus toxoid การปฏิบัติในข้อใดเหมาะสม ที่สุดในผู้ป่วยรายนี้ a. Rabies vaccine b. Rabies Ig c. เ ฝ้ า ดู อ า ก า ร สุ นั ข 1 สัปดาห์ d. ตัดหัวสุนัขส่งตรวจและ รอฟังผล e. Rabies vaccine แ ล ะ Rabies Ig 25. ชาย 70 ปี ปัสสาวะบ่อย ปั สสาวะไม่สุด ปั สสาวะ นานกว่าปกติ ปั สสาวะไม่ พุ่ง และมีปัสสาวะหยดหลัง จ า ก ปั ส ส ว ะ เ ส ร็ จ ต ร ว จ Rectal examination : firm , slightly enlarged and firm prostate gland ,U.A. : normal การวินิจฉัยในข้อใด เป็นไปได้มากที่สุด a. Urethral stricture b. Acute prostatitis c. Prostate cancer d. Neurogenic bladder e. BPH